You are on page 1of 293

7

CBSE

MATHEMATICS
Prepare • Practice • Perform

Kuber 

(Progressive Educational Publishers)

Full Marks Pvt Ltd


New Delhi-110002
Published by:

4238 A, 1-Ansari Road, Daryaganj, New Delhi-110002


Phone: 40556600 (100 Lines)
Fax: 40556688
Website: www.fullcircleeducation.in
E-mail: info@fcepl.in

© Publishers

All rights reserved. No part of this publication may be reproduced or transmitted, in any form or by any means,
without permission. Any person who does any unauthorised act in relation to this publication may be liable to criminal
prosecution and civil claims for damages.

Branches:
• Ahmedabad • Bengaluru • Chennai • Dehradun • Guwahati • Hyderabad • Jaipur • Kochi
• Kolkata • Lucknow • Mumbai • Ranchi

NEW EDITION

“This book is meant for educational and learning purposes. The author(s) of the book has/have taken all reasonable care
to ensure that the contents of the book do not violate any existing copyright or other intellectual property rights of any
person in any manner whatsoever. In the event the author(s) has/have been unable to track any source and if any copyright
has been inadvertently infringed, please notify the publisher in writing for corrective action.”
Preface

Mathematics-VII is based on the latest curriculum guidelines specified by the CBSE.


The book will certainly prove to be a torch-bearer for those who toil hard to achieve their goal.
Salient Features of the Book:
●● The whole book is well designed to aim at total and easy learning. This will not only build
up students’ morale but their confidence also.
●● Each chapter is designed in Topicwise manner where every topic is briefly explained with
sufficient solved examples and exercise.
●● All topicwise exercises incorporate VSA, SA-I, SA-II and LA for indepth practice and
learning.
●● Most important questions from NCERT Textbook and NCERT Exemplar are included.
●● HOTS and Value Based Questions have been given to assess students understanding
beyond the text and its application in the real world.
●● Termwise Periodic Test Papers, Model Examination Papers and Chapterwise worksheets
provided at the end of the book prepare the student from examination point of view.
Suggestions for further improvement of the book, pointing out printing errors/mistakes which
might have crept in, in spite of all efforts, will be thankfully received and incorporated in the
next edition.
CBSE Circular No.: Acad-14/2017 Dated: 21/03/2017
Scholastic Area: The assessment structure and examination for classes VI to VIII have been prepared in
view of the provisions of RTE-Act 2009 and comprises of two terms i.e. Term-1 and 2 as explained below:
Subjects Term-1 (100 marks) Term-2 (100 marks)
(1st half of the session) (2nd half of the session)
20 marks Periodic Assessment + 80 20 marks Periodic Assessment + 80 marks for
marks for Half Yearly Exam Half Yearly Exam
Language - 1 PA 20 Marks Half Yearly PA 20 Marks Yearly Exam
• Periodic Test Exam • Periodic Test • Written exam for 80 marks
Language - 2
10 marks with • Written exam 10 marks with with syllabus coverage as
Language - 3 syllabus covered for 80 marks syllabus covered below:
till announcement with syllabus till announcement Class VI: 10% of 1st term
Mathematics of test dates by covered till of test dates by covering significant topics +
school announcement school entire syllabus of 2nd term
Science • Note Book of Half Yearly • Note Book Class VII: 20% of 1st term
Submission 5 exam dates by Submission 5 covering significant topics +
Social
marks at term school marks at term end entire syllabus of 2nd term
Science
end • Sub Enrichment Class VIII: 30% of 1st term
Any other • Sub Enrichment 5 marks at term covering significant topics +
Subjects 5 marks at term end entire syllabus of 2nd term
end

(iii)
CONTENTS
1. Integers.............................................................................................................. 5

2. Fractions and Decimals................................................................................... 22

3. Data Handling................................................................................................. 49

4. Simple Equations............................................................................................ 69

5. Lines and Angles............................................................................................. 81

6. The Triangle and Its Properties........................................................................ 97

7. Congruence of Triangles................................................................................ 122

8. Comparing Quantities................................................................................... 135

9. Rational Numbers......................................................................................... 154

10. Practical Geometry........................................................................................ 165

11. Perimeter and Area........................................................................................ 177

12. Algebraic Expression..................................................................................... 191

13. Exponents and Powers.................................................................................. 204

14. Symmetry...................................................................................................... 214

15. Visualising Solid Shapes............................................................................... 225

• Term–1: Periodic Test Papers 1 & 2 ��������������������������������������������������������������� 239

• Term–1: Half-yearly Test Papers 1 & 2����������������������������������������������������������� 242

• Term–2: Periodic Test Papers 1 & 2���������������������������������������������������������������� 248

• Term–2: Annual Test Papers 1 & 2����������������������������������������������������������������� 251

• Chapterwise Worksheets���������������������������������������������������������������������������������� 257

Answers to Test Papers and Chapterwise Worksheets������������������������������������� 289

(iv)
1 Integers
Topics Covered
1.1  Properties of Addition and Subtraction of Integers 1.2  Multiplication of Integers
1.3  Division of Integers

Let’s Remember
• The collection of natural numbers, zero and negatives of natural numbers is called Integers.
I = {..., –3, –2, –1, 0, 1, 2, 3, ...}

Recall
• Integers can be represented on a number line.

• An integer is greater than any integer if it lies to the right of the other on the number line
and smaller if it lies to the left of the other.
• If a number is greater than the other numbers then its opposite (i.e., negative) is lesser than
the negative of other number, e.g., –4 < –1 and 2 > –1
i.e., if a > b, then –a < –b
• There is no smallest or greatest integer. 1 is the smallest positive integer and –1 is the greatest
negative integer.
• The integer 0 is neither positive nor negative.
• Zero is greater than every negative integer and smaller than every positive integer.
• The absolute value of an integer is its numerical value regardless of its sign, e.g., |9| = 9 and
|–9| = 9.
• Addition and subtraction of integers
(i) Rules for the number line:
On the number line, when we
(a) add a positive integer, we move to the right.
(b) add a negative integer, we move to the left.
(c) subtract a positive integer, we move to the left
(d) subtract a negative integer, we move to the right
(ii) Rules for addition of two integers:
(a) If two integers are with like signs, we add their absolute values and give the sum with
their common sign.

5
(b) If the two integers are with unlike signs, we find the difference of their absolute values,
and put the sign of the integer with greater absolute value.
(iii) To subtract an integer b from a, we change the sign of b and add it to a.
Example 1: Arrange the following integers in ascending order: –4, 1, –6, 3, –2 and 2
Solution. The given integers when arranged in ascending order are –6, –4, –2, 1, 2 and 3.
Example 2: Find the value of: (a) | –30 | (b) |– 40 | + | –3|
Solution. (a) | –30 | = 30
(b) | –40 | + | – 3| = 40 + 3 = 43
Example 3: Simplify: 1 + (–3) – (–6)
Solution. 1 + (–3) – (–6) = 1 – 3 + 6 = 1 + 3 = 4

Exercise (RECALL)
I. Very Short Answer Questions (1 Mark)
A. Answer the following.
1. Which temperature is lower: +3°C or –3°C?
2. What is the absolute value of –25 + 10?
3. What is the negative of 0?
4. What is the sum of an integer and its negative?
5. 0 is greater than which integers? 6. Is –1 the smallest negative integer?
7. Which temperature is higher +8°C or –8°C?
8. Write a pair of integers whose absolute value is 27.
9. Which is the smallest integer? 10. Write the largest negative integer.
B. Fill in the blanks.
1. Sum of two positive integers is ............... .
2. –52 + ............... = 0 3. 35 and –35 are ............... of each other.

II.  Short Answer Type Questions–I (2 Marks)


1. Arrange the following integers in ascending order: –11, –16, 8, 3, –9, 4.
2. Is 10 – (–10) = –10 – 10? Why?  [HOTS]
3. At midnight the temperature was –8°C. One hour later it was 5°C warmer. What was the
temperature then?
4. Find:
(i) –207 + (–207) (ii) the additive inverse of | –20 |
(iii) the value of |–40| – |–3|

III.  Short Answer Type Questions–II (3 Marks)


1. GPS School organised an apple race. Jaya and Mona participated in this race. The race
was conducted in 6 parts. In the first part Jaya won by 20 seconds. In the second part Jaya

6 Mathematics–7
lost by 2 minutes, then she won by 40 seconds in the third part and lost by 50 seconds
in the fourth part, she lost by 74 seconds in the fifth part and won by 24 seconds in the
last part. Who won the race finally? What values is the school inculcating in the students
by organising this kind of competition? [VBQ]
2. Neena has `1520 in her savings account at the beginning of the month. The page of the
passbook shown below shows all her transactions during the month. How much money
is left in her account after these transactions?
Date Transactions Cheque No. Withdrawal Deposit Balance
`
7/4 MTNL 965102 `490
12/4 Deposit 83216 `400
23/4 Insurance payment 965103 `650
28/4 Deposit 731269 `390

IV.  Long Answer Type Questions (4 Marks)


1. Simplify: 9 – [4 + {3 – (5 – 8 – 6) – (–3 + 2)}]
2. A water tank has steps inside it. A monkey is sitting on the topmost step (i.e., the first
step). The water level is at the ninth step.
(i) The monkey jumps 3 steps down and then jumps back 2 steps
up. In how many jumps will he reach the water level?
(ii) After drinking water, the monkey wants to go back. For this,
he jumps 4 steps up and then jumps back 2 steps down in
every move. In how many jumps will he reach back to the top
step?
(iii) If the number of steps moved down is represented by
negative integers and the number of steps moved up by
positive integers, represent his moves in part (i) and (ii) by completing the following:
(a) –3 + 2 – ..... = –8 (b) 4 – 2 + ..... = 8.
In (a) the sum (–8) represents going down by eight steps. So, what will the sum 8 in
(b) represent?  [NCERT]
3. Place A is 1600 m above sea level. Another place B is 900 m below sea level. What is
the difference between the levels of these two places?
4. The water level in a well was 30 m below ground level. During the rainy season, rain
water collected in different water tanks was drained into the well and the water level in
the well rises by 10 m above the previous level. The wall of the well is 1 m 10 cm high
and a pulley is fixed at a height of 90 cm. Jaina wants to draw water from the well. What
is the minimum length of the rope that she can use? [HOTS]
5. History. Immediately after 1 BC, 1 AD came. Solve the problems given below by taking
1 BC as –1 and 1 AD as +1.
(i) The Greco-Roman era, when Greece and Rome ruled Egypt, started in the year
330 BC and ended in the year 395 AD. How long did this era last?
(ii) Bhaskaracharya was born in the year 1114 AD and died in the year 1185 AD. What
was his age when he died?

Integers  7
(iii) Turks ruled Egypt in the year 1517 AD and Queen Nefertiti ruled Egypt about 2900
years before the Turks. In what year did she rule?
(iv) Greek mathematician Archimedes lived between 287 BC and 212 BC and Aristotle
lived between 380 BC and 322 BC. Who lived during an earlier period?
 [Multidisciplinary Question]

Answers and Hints


I. A. 1. –3° C 2. 15 3. 0 2. `1170
4. 0 Hint: Starting balance = `1520 (+)
5. 0 is greater than all negative integers. Date Withdrawal Deposit Balance `
6. No 7. +8°C 8. 27, –27 7/4 `490 — `1030
9. None 10. –1 12/4 — `400 `1430
B. 1. positive 2. 52 23/4 `650 — `780
3. additive inverse 28/4 — `390 `1170
IV. 1. –8
II. 1. –16, –11, –9, 3, 4, 8
Hint: 9 – [4 + {3 – (5 – 8 – 6) – (–3 + 2}]
2. No, as 10 – (–10) = 10 + 10 = 20
= 9 – [4 + {3 + 9 + 1}] = 9 – [4 + 13]
and –10 – 10 = –20 = 9 – 17 = –8
Clearly, 20 π –20 2. (i) 11 Jumps
3. –3°C [Hint: –8°C + 5°C = –3°C] Hint: Monkey is at 1st step and the water
4. (i) –414 (ii) –20 (iii) 37 level is at 9th step. The monkey will reach
III. 1. Mona the water level in the following way.
Hint: Result of 1st round = +20 Number of Position of
Result of 2nd round = –120 jumps Monkey
[2 minutes = 120 seconds] 1st 1 + 3 = 4th step
Result of 3rd round = +40 2nd 4 – 2 = 2nd step
Result of 4th round = –50 3rd 2 + 3 = 5th step
Result of 5th round = –74 4th 5 – 2 = 3rd step
Result of 6th round = +24 5th 3 + 3 = 6th step
So, Final result = 2 0 + ( – 1 2 0 ) + 4 0 6th 6 – 2 = 4th step
+ (–50) + (–74) + 24 7th 4 + 3 = 7th step
= 20 + 40 + 24 + (–120) + (–50) + (–74) 8th 7 – 2 = 5th step
= 84 – (120 + 50 + 74) 9th 5 + 3 = 8th step
= 84 – 244 = –160 10th 8 – 2 = 6th step
Thus, Mona won the race by 160 seconds 11th 6 + 3 = 9th step which
= 2 minutes and 40 seconds. is the water level.
Value: Schools organise such events to (ii) 5 jumps
inculcate the spirit of sportsmanship among Hint: After drinking water, the monkey’s
the students, i.e., cooperation and team position is at 9th step (water level) The
spirit. monkey will reach the top in the following
way.

8 Mathematics–7
Number of Position of Monkey Height of the wall from ground level
jumps = 1 m 10 cm = 1.10 m.
Height of the pulley from the top of the
1st 9 – 4 = 5th step wall = 90 cm = 0.9 m.
2nd 5 + 2 = 7th step So, the length of rope required to draw
3rd 7 – 4 = 3rd step water from the wall
4th 3 + 2 = 5th step = 0.9 m + 1.10 m + 20 m = 22 m.
5th 5 – 4 = 1 which is the 5. (i) 725 years
top most step. H int:  330 BC = –330
(iii) (a) –3 + 2 – 3 + 2 – 3 + 2 – 3 and 395 AD = +395
+ 2 – 3 + 2 – 3 = –8 Length of the era = 395 – (–330)
(b) 4 – 2 + 4 – 2 + 4 = 8 = 725 years
The sum (–8) in (a) represents going (ii) 71 years
down by 8 steps. Therefore, the sum (+8) Hint: Age of Bhaskaracharya
in (b) represents going up by 8 steps. 1185 – 1114 = 71 years
3. 2500 m (iii) 1383 BC
Hint: Height of place is 1600 m above Hint: Year in which Turks ruled = 1517 AD
sea level which represents +1600. Year in which Queen Nefertiti ruled
Height of place B is 900 m below sea = 1517 – 2900 = –1383 = 1383 BC
level which represents –900. (iv) Aristotle
Difference between the level of these Hint: Archimedes lived in-between 287 BC
two places and 212 BC.
= 1600 – (–900) = 1600 + 900 In integer form, period of Archimedes was
= 2500 m –287 to –212.
4. 22 m Aristotle lived between 380 BC and
Hint: Initial distance between water level 322 BC.
and ground level = 30 m. In integer form, period of Aristotle was
Distance between water level and ground –380 and –322.
level after draining water So, Aristotle lived in the earlier period.
= 30 m – 10 m = 20 m.

1.1  Properties of Addition and Subtraction of Integers

Closure Property
• Sum of two integers is always an integer, i.e., for any two integers a and b, a + b is an
integer.
• Difference of two integers is always an integer, i.e., for any two integers a and b, a – b is
an integer.
Examples:
Statement Observation
(a) 15 + 25 = 40 Answer is an integer
(b) –10 + 4 = –6 Answer is an integer
(c) –80 + 18 = –62 Answer is an integer

Integers  9
(d) –30 + 0 = –30 Answer is an integer
(e) 8 – 10 = –2 Answer is an integer
(f) 15 – (–21) = 36 Answer is an integer
(g) –31 – ( –10) = –21 Answer is an integer
(h) –28 – 0 = –28 Answer is an integer
Commutative Property
• Addition is commutative for integers, i.e., for any two integers a and b, a + b = b + a.
• Just like whole numbers, subtraction of integers is not commutative, i.e., for any two integers
a and b, a – b π b – a.
Examples: (a) –5 + (–14) = –5 – 14 = –19
and (–14) + (–5) = –14 – 5 = –19
Thus, –5 + (–14) = (–14) + (–5)
(b) (–6) – (–20) = –6 + 20 = 14
and (–20) – ( –6) = – 20 + 6 = –14
Thus, –6 – (–20) π (–20) – (–6)
Associative Property
• Addition of integers is associative. If a, b and c are any three integers, then
(a + b) + c =a + (b + c)
• Subtraction of integers is not associative. If a, b and c are any three integers, then
(a – b) – c π a – (b – c)
Examples: (a) [(–4) + (–7)] + 18 = (–11) + 18 = 7
and (–4) + [(–7) + 18] = (–4) + (–7 + 18)
= (–4) + 11 = –4 + 11 = 7
Thus, [(–4) + (–7)] + 18 = (–4) + [(–7) + 18].
(b) [(–4) – (–10)] – 16 = [–4 + 10] – 16 = 6 – 16 = –10
and (–4) – [(–10) – 16] = (–4) – [–10 – 16] = (–4) – (–26)
= –4 + 26 = 22
Thus, [(–4) – (–10)] – 16 π (–4) – [(–10) – 16].
Additive Identity
• Zero (0) is an additive identity for integers. If a is any integer, then a + 0 = 0 + a = a
Examples: (a) 6 + ( –6) = 6 – 6 = 0
(b) (–6) + (6) = –6 + 6 = 0
Thus, 6 + (–6) = (–6) + 6 = 0
Additive Inverse
• For every integer, there exists its additive inverse (opposite or negative) such that their sum
is zero.
If a is any integer, then a + (–a) = (–a) + a = 0
Examples: (a) 7 + (–7) = 0 or (–7) + 7 = 0
(b) (–14) + 14 = 0 or 14 + (–14) = 0

10 Mathematics–7
Property of Zero
• When zero (0) is subtracted from an integer, we get the same integer. If a is any integer,
then a – 0 = a.
Examples: (a) 18 – 0 = 18 (b) –15 – 0 = –15
(c) –37 – 0 = –37 (d) 173 – 0 = 173

Exercise 1.1
I. Very Short Answer Questions (1 Mark)
A. Answer the following.
1. What is the additive inverse of 0? 2. What is the additive identity of integers?
3. Are integers closed under addition and subtraction?
4. Is 8 + (–7) an integer? 5. Find (–8) + 0
6. Is 6 + (–6) = 0?
7. What is the sum of a number and its additive inverse?
B. Fill in the blanks.
1. The additive inverse of ............... is 1. 2. Additive inverse of –7 is ............... .
3. Additive inverse of 8 is .............. . 4. 3 + (–7) = ...............+ 3
5. [6 + (–3)] + 2 = 6 + [............... + 2]
C. Find the value of x in the following.
1. (–6) + 4 = x + (–6). 2. x + 0 = 0 + (–7).
3. 4 + (5 + 6) = (4 + x) + 6. 4. (–5) + 5 = 0 = x + (–5).
II.  Short Answer Type Questions–I (2 Marks)
A. State whether the following statements are True or False. Give justification.
1. Sum of any two integers is an integer.
2. x – 0 = x for any integer ‘x’. This means 0 is the identity of subtraction for integers.
B. Answer the following.
1. Write a pair of integers whose
(i) sum is greater than only one of the integers. (ii) sum is zero.
(iii) difference is greater than both the integers.
2. Find the sum by suitable rearrangement: 425 + (–207) + 75.
3. Verify a + b = b + a by taking a = 5 and b  =  –6.
4. Is |–10| + | –5 | = –5 + (–10)? Explain.
5. The sum of two integers is 410. If one of the integers is –90, find the other integer.
III.  Short Answer Questions–II (3 Marks)
1. Verify x + (y + z) = (x + y) + z for x = –3, y  = –4 and z = 7.
2. Find the following.
(i) a pair of negative integers whose difference gives 8.
(ii) a negative and a positive integer whose sum is –5.
(iii) a negative and a positive integer whose difference is –3.

Integers  11
3. Chemistry. An atom consists of two types of charged particles called electrons and
protons. Each proton has a charge of +1 and each electron has a charge of –1. Note that
the number of electrons is equal to the number of protons. Now answer the following
questions:
(i) What is the charge on an atom?
(ii) What will be the charge on an atom if it loses 1 proton?
(iii) What will be the charge on an atom if it gains 1 electron? 
 [Multidisciplinary Questions]
IV.  Long Answer Type Questions (4 Marks)
1. In a quiz, team A scored –40, 10, 0 and team B scored 10, 0, –40 in
three successive rounds. Which team scored more? Can we say that we
can add integers in any order?  [NCERT]
2. Fill in the blanks to make the following statements true:
(i) (–5) + (–8) = (–8) + (............) (ii) –53 + ............ = –53
(iii) 17 + ............ = 0
(iv) [13 + (–12)] + (............) = 13 + [(–12) + (–7)]

Answers and Hints


I. A . 1. 0 2. 0 3. Yes 4. No, as | –10 |  +  | –5 | = 10 + 5 = 15
4. Yes 5. –8 6. Yes and –5 + (–10) = –5 –10 = –15
7. 0 5. 500
B. 1. –1 2. 7 3. –8 Hint: Sum of two integers = 410
4. –7 5. –3 One integer = –90
C. 1. x = 4 2. x = –7 3. x = 5 Other integer = 410 – (–90)
4. x = 5 = 410 + 90 = 500
II. A. 1. True; e.g., –2 + (–3) = –5, –2 + 3 = 1, III. 1. Hint: LHS = x + (y + z)
2 + (–3) = –1, etc. = –3 + (–4 + 7) = –3 + 3 = 0
2. True; 3 – 0 = 3, –4 – 0 = –4 RHS = (x + y) + z
B. 1. (i) –2, 1 (ii) –4, 4 (iii) –3, –6 = [–3 + (–4)] + 7 = (–3 – 4) + 7
2. 293 = (–7) + 7 = –7 + 7 =  0
Hint: 425 + (–207) + 75
\ LHS = RHS
= (425 + 75) + (–207) = 500 – 207
Hence, verified.
= 293
2. (i) –2, –10
3. Hint: When a = 5, b = –6
Hint: –2 – (–10) = 8
a + b = 5 + (–6) = –1 ...(1)
(ii) –6, 1
b + a = –6 + 5 = –1 ...(2)
Hint: –6 + 1 = –5
From (1) and (2)
(iii) –1, 2
a + b = b + a = –1
Hint: –1 – 2 = –3
Hence verified.

12 Mathematics–7
3. (i) 0, (ii) –1 (iii) –1 Hint: Total score of team A
Hint: Charge on proton = +1 = (–40) + 10 + 0 = –40 + 10 = –30
Charge on electron = –1 Total score of team B
Number of electrons = Number of protons = 10 + 0 + (–40) = 10 + (–40) = –30
(i) Charge on an atom = 0 2. (i) –5
(ii) Charge on an atom if it loses 1 proton H int: \ (–5) + (–8) = (–8) + (–5)
= 0 – (+1) = –1 (ii) 0 [Hint: \ –53 + 0 = –53]
(iii) Charge on an atom if it gains 1 electron (iii) –17
= 0 + (–1) = 0 – 1 = –1 \ 17 + (–17) = 0
IV. 1. Scores of both the teams are same, i.e., (iv) –7
–30.
Hint: [13 + (–12)] + ( –7)
Yes, we can add integers in any order.
= 13 + [(–12) + (–7)]

1.2  Multiplication of Integers


• To multiply a positive integer and a negative integer, we multiply them as whole numbers
and put a minus sign (–) before the product.
i.e., a × (–b) = – (a × b) and (– a) × b = – (a × b), where a and b are positive integers.
Examples: (a) 4 × (–1) = – 4 (b) (–1) × 4 = – 4
Note that the product of a positive and a negative integer is a negative integer.
• To multiply two positive integers, we multiply them as whole number, i.e., if a and b are
two positive integers, then
(a) × (b) = a × b
Examples: (a) 5 × 6 = 30 (b) 8 × 15 = 120
Note that the product of positive integers is a positive integer.
• To multiply two negative integers, we multiply them as whole numbers and write the product
with a positive sign or without any sign.
i.e., if a and b are two positive integers, then (–a) × (–b) = a × b
Examples: (a) –6 × (–6) = 36 (b) (–4) × (–1) = 4
N ote that the product of two negative integers is a positive integer.
• To find the product of three or more negative integers, we multiply them as whole numbers
and put the
(i) positive sign (+) if the number of integers in a product is even.
(ii) negative sign (–) if the number of integers in a product is odd.
Examples: (a) (–3) × (–2) × (–4) × (–5) = 3 × 2 × 4 × 5 = 120
(b) (–1) × (–5) × (–7) × (–3) × (–10) = – (1 × 5 × 7 × 3 × 10) = –1050
Properties of Multiplication of Integers
• The collection of integers is closed under multiplication, i.e., if we multiply two integers,
we always get an integer as their product. For any integer a and b, a × b is an integer.
Examples: (a) 4 × (–6) = –24 (an integer) (b) –8 × (–3) = 24 (an integer)

Integers  13
• The collection of integers is commutative under multiplication, i.e., if a and b are any two
integers, then a × b = b × a.
Example: 18 × (–3) = –54 and (–3) × 18 = –54
So, 18 × (–3) = (–3) × 18
• The collection of integers is associative under multiplication, i.e., if a, b and c are any three
integers, then a × (b × c) = (a × b) × c = (a × c) × b.
Example: [(4 × –3)] × 2 = (–12) × 2 = –24 and 4 × [(–3) × 2] = 4 × (–6) = –24
Thus, [4 × (–3)] × 2 = 4 × [(–3) × 2] = –24
• Multiplication of integers is distributive over addition and subtraction, i.e., if a, b and c
are any three integers, then a × (b + c) = a × b + a × c and a × (b – c) = a × b – a × c.
Example: [18 + (–15)] × 2 = (18 – 15) × 2 = 3 × 2 = 6
Also, [18 × 2 + (–15) × 2 = 36 – 30 = 6
Thus, [18 + (–15)] × 2 = 18 × 2 + (–15) × 2
• 1 is the multiplicative identity of integers, i.e., if x is an integer, then × 1 = 1 × x = x
Example: –5 × 1 = 1 × (–5) = –5
• The product of any integer and (–1) is the additive inverse of that integer, i.e., if x is an
integer, then x × (–1) = –x
Example: –8 × (–1) = 8, 7 × (–1) = –7
• Product of any integer with zero is zero, i.e., if x is any integer, then x × 0 = 0.
Example: –8 × 0 = 0 × (–8) = 0

Exercise 1.2
I. Very Short Answer Type Questions (1 Mark)
A. Answer the following.
1. What is the sign of the product of a negative integer with a negative integer?
2. What is the product if (–1) is multiplied by itself 10 times?
3. What is the product if (–1) is multiplied by itself 15 times?
4. Are integers closed under multiplication?
5. What are the possible values of integer ‘a’ for which a × a = a?
6. Which integer multiplied by (–1), gives 20?
B. Fill in the blanks.
1. If the number –1 is multiplied by itself .......... number of times, the product is (–1).
2. The product of any integer with zero is ............... .
3. (–8) × 7 = 7 × (–8) by ............... property of multiplication.
C. Say True or False.
1. Associative property of multiplication states that (a × b) × c = a × (b × c) for integers
a, b and c.
2. If we multiply (–1), 3m times, where m is an odd natural number, we get 1.
3. The product of a positive integer and a negative integer may be 0.

14 Mathematics–7
D. Find the following products.
1. 3 × (–15) = 2. (–8) × (–7) =
3. –6 × 5 = 4. 9 × (–3) =
5. (–3) × (5) × (–2) = 6. (–1) × (10) × (–2) =
7. (–5) × (4) × 0 = 8. (–2) × (–4) × (–6) × 1 =
9. 25 × (–4) × (–3) × 2 =
II.  Short Answer Type Questions–I (2 Marks)
1. Multiply by suitable rearrangement: –250 × 782 × (–4).
2. Find (–50) × (105) using distributive property.
3. Verify a × b = b × a by taking a = –4, b = –6.
4. Verify a × 1 = a by taking a = –10.
5. Simplify 5 × (–3) + 5 × 13 using distributive property.
6. What will be the sign of the product if we multiply together:
(i) 8 negative integers and 3 positive integers?
(ii) 5 negative integers and 4 positive integers?
7. Check whether multiplication of integers is commutative. Justify your answer.
8. Multiply (–603) by 1. Is the product additive inverse of –603?
III.  Short Answer Type Questions–II (3 Marks)
1. Simplify 4 – (6 × 4 – 11 × 2).
2. Verify the property x × (y × z) = (x × y) × z by taking x = 4, y = –5 and z = 8.
3. A shopkeeper earns a profit of `1 by selling one pen and incurs a loss of 40 paise per
pencil while selling pencils of her old stock.
(i) In a particular month, she incurs a loss of `5. In this period, she sold 45 pens. How many
pencils did she sell in this period?
(ii) In the next month, she earns neither profit nor loss. If she sold 70 pens, how many pencils
did she sell?
4. Suppose we represent the distance above the sea level by a positive integer and that of
below the sea level by a negative integer. A diver descends into the sea at the rate of
5  m per minute. If he begins to descend from 15 m above the ground, what will be his
position after 18 minutes?
5. In a test containing 20 questions, 5 marks are given for every correct answer and (–4)
marks are given for every incorrect answer. A student attempts all questions but only 10
of his answers are correct. What is his total score?
IV.  Long Answer Type Questions (4 Marks)
1. Verify the property x × (y + z) = (x × y) + (x × z) by taking x = 5, y = –6 and z = 4.
2. Verify the property x × (y – z) = (x × y) – (x × z) by taking x = 5, y = –6 and z = –3.
3. Rachit starts from a point A and goes 8 km due South and then from there 10 km due
North. If the end point of the first round becomes the starting point of the second round,
and so on, how far and where will he be from point A after 5 rounds?

Integers  15
Answers and Hints
I. A. 1. Positive 2. +1 3. –1 Hint: 4 – (6 × 4 – 11 × 2)
4. Yes 5. 1 and 0 6. –20 = 4 – (24 – 22) = 4 – (2) = 2
B. 1. odd 2. zero 2. Hint: LHS = x × (y × z) = 4 × (–5 × 8)
3. commutative = 4 × (–40) = –160
C. 1. True 2. False 3. False RHS = (x × y) × z = [4 × (–5)] × 8
D. 1. –45 2. 56 3. –30 = (–20) × 8 = –160
4. –27 5. 30 6. 20 \ LHS = RHS
7. 0 8. –48 9. 600 Hence, verified.
II. 1. 782000 3. 125
Hint: –250 × 782 × (–4) Hint: (i) Profit on 45 pens = 45 × `1 = `45
= –250 × (–4) × 782 = 1000 × 782 Let the number of pencils sold be x
= 782000 Loss on one pencil = 40 paise = – 40
2. –5250  40 
Total loss = x × ( − 40) = `  − x
Hint: –50 × 105 = –50 × (100 + 5) 100 
= –50 × 100 + (–50) × 5
= –5000 – 250 = –5250 But, total loss = `5 = – 5
– 40
3. Hint: When a = –4, b = –6 \ 45 + d n x = –5
a × b = (–4) × (–6) = 24 ...(1) 100
b × a = –6 × (–4) = 24 ...(2) + 40 +4
⇒ 45 + 5 = x ⇒ 50 = x
From (1) and (2), a × b = b × a 100 10
Hence verified. 50 × 10
⇒ = x ⇒ 125 = x
4. Hint: When a = –10 4
a × 1 = –10 × 1 = –10 = a Number of pencils sold = 125
Hence verified. (ii) 175
5 . 50 Hint: Profit on 70 pens = 70 × `1 = `70
Hint: 5 × (–3) + 5 × 13 = 5 × [(–3) + 13] Let number of pencils sold be y
= 5 × (10) = 5 × 10 =  50  −40  –4
Loss on y = y × ( −40) = ` y ×   =` y
6. (i) Positive  100  10
Hint: 8 negative integers × 3 positive As there is neither profit nor loss
−4
integers \ ` 70 + `   y = 0
= (+) × (+) = positive  10 
4 2
(ii) Negative ` 70 = ` y = ` y
10 5 5
Hint: 5 negative integers × 4 positive ⇒ y = 70 × = 35 × 5 = 175
integers 2
= (–) × (+) = negative Total number of pencils sold = 175
7. Yes, e.g., –3 × 4 = –12 = 4 × (–3) 4. 75 m below sea level
8 . No Hint: Rate at which the diver descends
Hint: –603 × 1 = –603 = +5 km per min
III. 1. 2 Total distance in 18 minutes

16 Mathematics–7
= +5 × 18 = +90 m IV. 1. Hint: For x = 5, y = –6, z = 4
Diver starts from 15 m above ground. So, LHS = x × (y + z) = 5 × (–6 + 4)
after 18 minutes his position will be = 5 × (–2) = –10
15 – 90 = –75 m RHS = x × y + x × z
Thus, diver will be 75 m below sea level. = 5 × (–6) + 5 × 4 = –30 + 20 = –10
5. 10 \ LHS = RHS. Hence verified.
Hint: Total number of questions = 20 2. Hint: For x = 5, y = –6 and z = –3
No. of questions answered correctly = 10 LHS = x × (y – z) = 5 × (–6 – (–3))
Marks per correct answer = 5 = 5 × (–6 + 3) = 5 × (–3) = –15
Total marks for correct answers RHS = ( x × y) – (x × z)
= 10 × 5 = 50 = [5 × (–6)] – [5(–3)] = –30 –(–15)
Number of incorrect answers = –30 + 15 = –15
= 20 – 10 = 10 \ LHS = RHS. Hence verified.
Marks given for every incorrect answer = –4 3. 10 km North
Total marks for incorrect answers Hint: Result of the first round is positive
= 10 × (–4) = –40 and 10 – 8 = 2 km North.
His total marks = 50 + (–40) Result of 5 such rounds
= 50 – 40 = 10 = 2 × 5 = 10 km North
1.3 Division of Integers
• To divide a negative integer by a positive integer, we divide them as whole numbers and
then put a minus sign (–) before the quotient i.e., for any two positive integers a and b,
(–a) ÷ b = – (a ÷ b) where b π 0.
Examples: (a) (–12) ÷ 2 = – 6 (b) (–20) ÷ 5 = – 4
Note: Quotient of a negative and positive integer is always negative.
• To divide a positive integer by a negative integer, we divide them as whole numbers and
then put a minus sign (–) before the quotient, i.e., for any two positive integers a and b,
a ÷ (–b) = –(a ÷ b); b π 0.
Examples: (a) 72 ÷ (–8) = –9 (b) 50 ÷ (–5) = –10
Note: Quotient of a positive and negative integer is always negative.
i.e. if two integers are of unlike signs, their quotient is negative.
• To divide a positive integer by a positive integer, we divide them as whole numbers and
write the quotient with plus sign (+) or without any sign, i.e., for any two positive integers
a and b, a ÷ b = a ÷ b; b π 0.
Examples: (a) 15 ÷ 3 = 5 (b) 20 ÷ 4 = 5
Note: Quotient of two positive integers is positive.
• To divide a negative integer by a negative integer, we first divide them as whole numbers
and then write the quotient with plus sign (+) or without any sign, i.e., for any two positive
integers a and b, (– a) ÷ (–b) = (a ÷ b).
Examples: (a) –20 ÷ (–4) = 5 (b) –51 ÷ (–17) = 3
Note: Quotient of two negative integers is positive.
i.e. if two integers are of like signs, the quotient is positive.

Integers  17
Properties of Division of Integers
• The collection of integers is not closed under division.
Example: –5 ÷ 3 is not an integer so division of integers is not closed.
• The collection of integers is not commutative under division.
Example: 8 ÷ 4 = 2
4 1
but 4 ÷ 8 = = ≠ 2
8 2
Thus, division of integers is not commutative.
• Division of any integer by zero is meaningless, i.e., for any integer a, a ÷ 0 is not defined.
• When 0 is divided by any integer other than 0, the quotient is 0, i.e., for any integer
a, 0 ÷ a = 0 when a π 0
Examples: (a) 0 ÷ 5 = 0 (b) 0 ÷ (–7) = 0
• Any integer divided by 1 gives the same integer, i.e., for any integer a, a ÷ 1 = a.
Examples: –5 ÷ 1 = –5 and 7 ÷ 1 = 7
• Any integer divided by –1, gives the additive inverse of that integer, i.e., for any integer
a, a ÷ (–1) = –a
Examples: 5 ÷ (–1) = –5 and –7 ÷ (–1) = 7
• Any integer divided by its additive inverse, gives the quotient –1, i.e., for any integer
a, a ÷ (–a) = –1, or – a ÷ (a) = –1.
Examples: 8 ÷ (–8) = –1 and –4 ÷ (4) = –1
• Any integer divided by itself gives the quotient 1, i.e., for any integer a, a ÷ a = 1, where a π 0.
Examples: 7 ÷ 7 = 1 and –13 ÷ (–13) = 1
• The collection of integers is not associative under division.
Example: 12 ÷ [6 ÷ (–3)] = 12 ÷ (–2) = –6
(12 ÷ 6) ÷ (–2) = 2 ÷ (– 2) = –1
12 ÷ [6 ÷ (– 3)] π (12 ÷ 6) ÷ (–2)
\
Thus, division of integers is not associative.
• The collection of integers is not distributive under division.
Example: [7 + (–5)] ÷ 2 = 2 ÷ 2 = 1
7 ÷ 2 + (–5) ÷ 2 is not an integer.
Thus, [7 + (–5)] ÷ 2 π 7 ÷ 2 + (–5) ÷ 2
Thus, division of integers is not distributive over addition.
• To simplify a numerical expression, we perform the operation strictly in order of the letters of
the word “BODMAS”.

Exercise 1.3
I. Very Short Answer Type Questions (1 Mark)
A. Answer the following.
1. If a is any non-zero integer, then what is 0 ÷ a?
2. If an integer is divided by its additive inverse, what is the result?
3. If the product of two integers is (–216) and one of them is –6, then find the other.

18 Mathematics–7
.
B Fill in the blanks.
1. Quotient of a positive integer and a negative integer is ............... .
2. Division by ............... is not defined.
3. –42 ÷ 7 is .............. .
C. Say True or False.
1. 0 ÷ (–30) = –30 2. –15 ÷ 1 = –15
D. Find the quotient of the following:
1. 36 ÷ 6 = 2. 24 ÷ (–3) =
3. (–15) ÷ 5 = 4. (–24) ÷ (–6) =
5. 27 ÷ (–1) = 6. 0 ÷ (–6) =
II.  Short Answer Type Questions–I (2 Marks)
1. Verify a ÷ a = 1 by taking a = –9.
2. Verify a ÷ b  b ÷ a by taking a = 9 and b = 3.
3. Comment on the statement:
“Integers are commutative under division.”
4. Write three pairs of integers (a, b) such that a ÷ b = –5.
III.  Short Answer Type Questions–II (3 Marks)
1. Verify that (x ÷ y) ÷ z  x ÷ (y ÷ z) by taking x = 15, y = 3 and z = –4.
2. Verify that (a ÷ b) × c  a ÷ (b × c) by taking a = 13, b = 4 and c = –4.
3. Simplify: (i) 4 – (6 × 4 – 11 × 2)
(ii) 7 – {6 – 12 ÷ (5 + 9 × 2 – 19)}
IV.  Long Answer Type Questions (4 Marks)
1. Write all possible multiplication and division statements for the integers with
5, –5, 7, –7 and 35, –35, e.g., 5 × 7 = 35.
2. Simplify: 14 – [3 + {8 ÷ (5 – 3 of 2 + 3)}]
3. Simplify: 5 – [3 – {2 – (2 × (–3) + 1) + (3 – 5)}]

Answers and Hints


I. A. 1. 0 2. –1 3. 36 From (1) and (2)
B. 1. Negative 2. 0 3. –6 a ÷ b ≠ b ÷ a. Hence verified.
C. 1. False [Hint: 0 ÷ (–30)= 0] 3. No, integers are not commutative under
2. True division.
1
D. 1. 6 2. –8 3. –3 e.g. 4 ÷ (–8) = – , –8 ÷ 4 = –2
1 2
4. 4 5. –27 6. 0 and – ≠ –2
II. 1. Hint: When a = –9, 2
4. (i) 15 ÷ (–3) = –5, (ii) –30 ÷ 6 = –5
L.H.S. = a ÷ a = – 9 ÷ (–9) = 1 = R.S.H.
(iii) 40 ÷ (–8) = –5
Hence verified. III. 1. Hint: LHS = (x ÷ y) ÷ z
2. Hint: L.H.S. = a ÷ b = 9 ÷ 3 = 3 ...(1) 15 5
3 1
= (15 ÷ 3) ÷ (–4) = ÷ ( −4) = 5 ÷ ( −4) = −
R.H.S. = b ÷ a = 3 ÷ 9 = = ...(2) 3 4
9 3 = RHS = x ÷ (y ÷ z) = 15 ÷ [3 ÷ (–4)]

Integers  19
 3  4 IV. 1. 5 × 7 = 35, 5 × (–7) = –35
= 15 ÷  −  = 15 ×  −  –5 × 7 = –35,
 4  3
= 5 × (–4) = –20 –5 × (–7) = 35, –7 × (–5) = 35,
LHS  RHS. Hence verified. and –7 × 5 = –35.
2. LHS = (a ÷ b) × c = (13 ÷ 4) × (–4) 35 ÷ 5 = 7, 35 ÷ (–5) = –7, 35 ÷ 7 = 5,
13 35 ÷ (–7) = –5, –35 ÷ 5 = –7,
= × ( −4) = 13 × ( −1) = −13
4 –35 ÷ (–5) = 7, –35 ÷ 7 = –5,
RHS = a ÷ (b × c) = 13 ÷ [4 × (–4)] –35 ÷ (–7) = 5
13 2. 7
= 13 ÷ ( −16) = −
16 Hint: 14 – [3 + {8 ÷ (5 – 6 + 3)}]
LHS  RHS
Hence verified. = 14 – [3 + {8 ÷ (8 – 6)}]
3. (i) 2 = 14 – [3 + {8 ÷ 2} = 14 – [3 + 4]
Hint: 4 – (24 – 22) = 4 – 2 = 2 = 14 – 7 = 7.
(ii) 4 3. 7
Hint: 7 – {6 – 12 ÷ (5 + 18 – 19)} Hint: 5 – [3 – {2 – (–6 + 1) + (–2)}]
= 7 – {6 – 12 ÷ (23 – 19)} = 5 – [3 – {2 – (–5) – 2}]
= 7 – {6 – 12 ÷ 4} = 5 – [3 – {2 + 5 – 2}]
= 7 – {6 – 3} = 7 – 3 = 4 = 5 – [3 – 5] = 5 – (– 2) = 5 + 2 = 7

Subject Enrichment Activities


I. Crossword
Complete the following crossword using the given clues.

Across Down
1. 1 is the ............... identity for integers. 4. Sign of the product of two negative integers.
2. ............... property of addition states 5. The operation under which integers are not
that a + b = b + a. closed.
3. .............. property of multiplication 6. –1 is the multiplicative ............... of –1.
states that (a × b) × c = a × (b × c). 7. Quotient of a positive and negative integer
is ............... integer.

20 Mathematics–7
II. Multiple Choice Questions
Choose the correct option.
1. The absolute value of (–8) is
(a) –8 (b) 8 (c) –1 (d) 1
2. The next term in the sequence is –1, 3, –9, 27, –81, ...
(a) 243 (b) 81 (c) –243 (d) 729
3. What should be added to –15 to get – 8?
(a) –7 (b) 7 (c) –23 (d) 23
4. On simplification 12 – 3[5 + 2{(5 – 7) ÷ 2}] gives
(a) 3 (b) –3 (c) 9 (d) 21
5. –5 ÷ (–5) is equal to
(a) 0 (b) 1 (c) –1 (d) –10
6. Sum of a positive and a negative integer is always
(a) positive (b) negative (c) zero (d) may be positive or negative
7. The product (–6) × (–5) × 4 is
(a) 150 (b) –150 (c) 120 (d) –120
8. (–80) × 392 + 80 × (–608)
(a) –80000 (b) 8000 (c) –8000 (d) 80000
III. Match the columns.
Column I Column II
(i) a × 1 (a) 0
(ii) –a ÷ a (b) 1
(iii) y + (–y) (c) –1
(iv) (–5) ÷ (–5) (d) y
(v) (–y) × (–1) (e) 6

Answers
I. II. 1. (b) 8 2. (a) 243 3. (b) 7
4. (a) 3 5. (b) 1
6. (d) may be positive or negative
7. (c) 120 8. (a) –80000
III. (i)—(e), (ii)—(c), (iii)—(a),
(iv)—(b), (v)—(d)


qqq

Integers  21
2 Fractions and Decimals
Topics Covered
2.1 Fractions 2.2  Multiplication of Fractions
2.3 Multiplication of a Fraction by a Fraction 2.4  Division of Fractions
2.5 Decimals 2.6 Multiplication of Decimals
2.7 Division of Decimals

Let’s Remember
• A fraction is a number which represents a part of the whole.
5
• In a fraction , 5 is called the numerator and 7 is called the denominator.
7
• A fraction whose denominator is a multiple of 10, is called a decimal fraction.
• In decimal numbers, we use a dot (•) known as a decimal point.
• The numbers before the decimal point is the whole part (also called integral part) and the
number after the decimal point is the fractional part.

2.1 Fractions
• In a fraction, the numerator represents the number of parts out of the whole whereas the
denominator represents the number of all parts in a whole.
• When the numerator is less than the denominator, the fraction is called a proper fraction.
• When the numerator is greater than the denominator, the fraction is called an improper
fraction.
• When a number has a whole number as well as a fraction, it is called the mixed number.
• Fractions having the same denominator, are called like fractions.
• Fractions with unequal denominators, are called unlike fractions.
• Fractions with numerator 1 are called unit fractions.
• When we multiply (or divide), both numerator and denominator, by the same non-zero number,
we get equivalent fractions.
• Each proper and improper fraction has as many equivalent fractions as we wish.
• A fraction is said to be in simplest form if its numerator and denominator have no common
factor except 1.
• A natural number is a fraction with denominator 1.

22
a
• A fraction is said to be in its lowest form if the HCF of a and b is 1.
b a c
• While comparing fractions, and , we have the following situations.
b d
a c a c a c
(i) If ad > bc,  then  > . (ii) If ad = bc,  then  = . (iii) If ad < bc,  then  < .
b d b d b d
• (i) For addition or subtraction of like fractions, numerators are added (or subtracted), but
denominator remaining the same. For example, 3 + 2 = 3 + 2 = 5 ; 6 − 3 = 6 − 3 = 3 .
7 7 7 7 7 7 7 7
(ii) For addition or subtraction of unlike fractions first change them to equivalent like fractions
(by finding the LCM of denominators) and then do as in (i).
2 3 8 15 23 3 3 1 15 7 8
For example, + = + = =1 ; – = – =
5 4 20 20 20 20 7 5 35 35 35
(iii) For addition or subtraction of mixed numbers, first convert them to improper fractions and
then proceed as in (i) and (ii) as the case may be.
1 3 3 11 12 11 23 7
For example, 1 +1 = + = + = =2 ;
2 8 2 8 8 8 8 8
1 5 41 29 123 58 65 17
5 –2 = – = – = =2
8 12 8 12 24 24 24 24

Exercise 2.1
I. Very Short Answer Type Questions (1 Mark)
A. Answer the following.
19
1. Express as a mixed fraction.
6
3
2. Find an equivalent fraction of with denominator 15.
5
17 14
3. Anu solved part of an exercise while Nidhi solved of it. Who solved lesser part?
25 25
7 5 5 2
4. Find the sum of and 5. Evaluate: – .
8 12 3 15
B. Say True or False.
1. Sum of two fractions is always a whole number.
1 3 5
2. When is subtracted from , we get .
3 4 12
34 5 1 3 7
3. is a fraction equivalent to 2 . 4. , , are arranged in descending order.
12 6 5 7 10
II.  Short Answer Type Questions–I (2 Marks)
1 2 3 1
1. Add 2 and 3 2. Subtract 1 from 2
4 5 4 2

Fractions and Decimals  23


11 5 4 7 1 11
3. Simplify: + – 4. Simplify: 1 + 1 – 1
12 18 9 8 4 12
5
5. I f 5 is added to both the numerator and the denominator of the fraction , will the value
9
of the fraction be changed? If so, will the value increase or decrease?

III.  Short Answer Type Questions–II (3 Marks)


5 1 2 7 3 5
1. Simplify: (i) + – (ii) 1 – +2
12 18 9 8 4 12
2. A square and an equilateral triangle have a side in common. If side of the
7
triangle is cm long, find the perimeter of the figure formed. 
3
 [Multidisciplinary Questions]

5 3
3. Sandhya finishes a work in hour whereas Soma finishes the same in hour. Who
6 4
finished the work in more time? By what fraction of an hour is it longer?
3 1 3
4. Ruma purchased 5 kg of potatoes, 2 kg of onions and 1 kg of tomatoes. What total
4 2 4
weight of vegetables did Ruma purchase?

IV.  Long Answer Type Questions (4 Marks)


2 1
1. Rachna weighed 18 kg in July but lost 1 kg in the month of August. If at the end of
3 5 9
September she weighs 19 kg, find the change in her weight in the month of September.
6
2. Find the perimeters of (i) DABE (ii) the rectangle BCDE in this figure. Whose perimeter
is greater and by how much?

Answers and Hints


1 9 7 5 7 # 3 5# 2
I. A. 1. 3 2. Hint: + = +
6 15 8 12 8 # 3 12 # 2
14 17 21 10 31
3. Nidhi [Hint: < ] = + = ]
25 25 24 24 24
31
4. 8
24 5. 1
15

24 Mathematics–7
5 2 5#5 2 25 2 7 3 5 15 3 29
Hint: – = – = – Hint: 1 – +2 = – +
3 15 3 # 5 15 15 15 8 4 12 8 4 12
23 8 45 – 18 + 58 45 + 40 85 13
= =1 = = = = 3
15 15 24 24 24 24
B. 1. False 2. True 3. True 2
4. False 2. 11 cm
3
13 Hint: Perimeter of the figure formed
II. 1. 5
20
7 7 7 7 7
Hint: 2 + 3 =
1 2 9 17
+ =
45 + 68 = d + n+ d + + n
4 5 4 5 20 3 3 3 3 3
113 13 14 21 35 2
= =5 . = + = = 11 cm
20 20 3 3 3 3
3 1
2. 3. Sandhya, hour
4 12
1 3 5 7 10 – 7 3
Hint: 2 – 1 = – = = 4. 10 kg
2 4 2 4 4 4
5
3 IV. 1. 2 kg
3. 18
4
11 5 4 33 + 10 – 16 Hint: Rachna’s weight in the month of July
Hint: + – = 2 56
12 18 9 36 = 18 kg = kg
27 3 3 3
= =
36 4 Weight loss in the month of August
5 1 10
4. 1 = 1 kg = kg
24 9 9
7 1 11 15 5 23 56 10
Hint: 1 + 1 – 1 = + – Weight in August = –
8 4 12 8 4 12 3 9
45 + 30 – 46 29 168 – 10 158 5
= = = = kg = 17 kg
24 24 9 9 9
5. Yes, increases 5
5 Weight in September = 19 kg
Hint: Original fraction = 5 5 6
9 Since, 19 > 17
5 + 5 10 5 6 9
New fraction = = =
9 + 5 14 7 So, she gained weight in the month of
Value of new fraction is September.
5 5 Weight gain in September
increased as >
7 9 5 5 119 158
= 19 – 17 = –
1 6 9 6 9
III. 1. (i)
4 357 – 316 41 5
5 1 2 15 + 2 – 8 9 = = =2 kg
Hint: + – = = 18 18 18
12 18 9 36 36
1 17
= 2. (i) 8 m
4 20
13 5 3 3
(ii) 3 Hint: Perimeter of DABE = + 2 + 3
24 2 4 5

Fractions and Decimals  25


5 11 18 50 + 55 + 72 3 7 3 7 11 7 11 7
= + + = = 2 + +2 + = + + +
2 4 5 20 4 6 4 6 4 6 4 6
177 17 33 + 14 + 33 + 14
= =8 cm =
20 20 12
(ii) Perimeter of rectangle BCDE
94 47 5
= = = 7 cm
12 6 6
2.2  Multiplication of Fractions
Multiplication of a Fraction by a Whole Number
• To multiply a proper or an improper fraction by a whole number, we multiply the whole
number with the numerator of the fraction keeping the denominator same.
1 1#7 7 5 5 # 2 10 1
For example, # 7 = = ; #2 = = =1
4 4 4 9 9 9 9
• To multiply a mixed fraction to a whole number, we first convert the mixed fraction to an
improper fraction and then multiply.
2 16 64 1 5 11 99 33 1
For example, 2 # 4 = #4 = = 9 ; 9 #1 = 9 # = = = 16
7 7 7 7 6 6 6 2 2
Fraction as an Operator ‘of’
1 1 1 1#2 2
• ‘of’ represents multiplication, e.g., of 2 means # 2 and #2 = = .
3 3 3 3 3
Exercise 2.2
I. Very Short Answer Type Questions (1 Mark)
A. Answer the following.
3
1. How much is of a day?
4
2. What would be the number, if one-fourth of it is 3?
3. What type of a fraction is obtained if we multiply a fraction by a whole number greater
than 1?
3
4. What is the value of of 32 kg?
4
B. Fill in the blanks.
part of a cake weighing 1 kg and her sister Rashi ate 1 of the same cake.
2
1. Mona ate
5 5
Weight of the cake left is ............... .
4 1
2. of 25 is ............... . 3. 4  6 is equal to ............... .
5 3
8
4. 8 × 3 = ............... . 5. × 0 = ............... .
4 33
1 3
6. of 24  ............... . 7. of 63  ............... .
8 7

26 Mathematics–7
C. Say True or False.
1 3
1. of `300 is `10. 2. One-fourth of fifteen makes 3 .
30 4
1 1 1 57
3. 7 × 0 = 7 . 4. 8 × 1 = .
6 6 7 7
1 1 1
5. 7 × 0 = 0. 6. 5 × 7 = 35 .
9 3 3
II.  Short Answer Type Questions–I (2 Marks)
3 2 1
1. Represent pictorially: 4 × . 2. Which is smaller of 36 or of 90?
4 3 5
1 1
3. One packet of biscuits requires 3 cups of flour and 2 cups of sugar. What is the
2 3
estimated total quantity of both ingredients used in 10 such packets of biscuits?

4. Renu completed 1 part of her homework in 1 hour. How much part of her homework
3
had she completed in 3 hours?

III.  Short Answer Type Questions–II (3 Marks)


1
1. In a class of 50 students, of the total number of students eat chapati only, 2 of the
5 5
total number of students like to eat rice only and the remaining students like to eat both.
Find the number of students who like to eat (i) chapati only (ii) rice only (iii) both chapati
and rice.
3
2. Represent 2  pictorially.
7
1 2 3
3. Shade (i) of the circles in box (a), (ii) of the triangles in box (b), (iii) of the
2 3 5
squares in box (c).


(a) (b) (c) [NCERT]
4. Vidya and Pratap went for a picnic. Their mother gave them a water bottle that contained
2
5 litres of water. Vidya consumed of the water. Pratap consumed the remaining water.
5
(i) How much water did Vidya drink?

(ii) What fraction of the total quantity of water did Pratap drink? [NCERT]

Fractions and Decimals  27


IV.  Long Answer Type Questions (4 Marks)
1. Use the graph of Manav’s day spent given below to frame a problem involving fractions.
How does Manav spend his day?

2. It takes 17 full specific type of trees to make one tonne of paper. If there are 221 such
trees in a forest, then
(i) What fraction of forest will be used to make 5 tonnes of paper?
7
(ii) To save part of the forest, how much of paper we have to save?
13
(iii) Why should we save our forests?  [VBQ]
3. In a survey, 400 students were asked, what influenced them most to buy their latest CD.
The results are shown in the pie chart.

(i) How many students said radio influenced them most?


(ii) How many more students were influenced by radio than by a music video channel?
(iii) How many said, a friend or a relative influenced them or they heard the CD in a shop?
Answers and Hints
I. A. 1. 18 hrs 3 3
3 Hint: of 32 kg = × 32 kg
Hint: × 24 = 18 hrs 4 4
4 = 3 × 8 kg = 24 kg
2. 12
1 B. 1. 400 g
Hint: of x = 3 , where x be the number.
4 Hint: Weight of cake eaten by Mona
1 2
× x = 3 ⇒ x = 3 × 4 = 12 = 1000 × = 400 g  [Q 1 kg = 1000 g]
4 5
3. May be proper or Improper fraction Weight of cake eaten by Rashi
4. 24 kg. 1
= 1000 × = 200 g
5

28 Mathematics–7
Weight of cake left = 1000 – (400 + 200) 7
1 = 10 × = 35 cups
= 400 g 2. 20 3. 25 2
3 3
4. 6 [Hint: 8 × = 2 × 3 = 6 ] For 1 packet, sugar required
4 1 7
5. 0 = 2 cups = cups
3 3
1 1 For 10 packets, sugar required
6. 3 [ Hint: of 24 = × 24 = 3 ]
8 8 7 70 1
3 = × 10 = cups = 23 cups
7. 27 [Hint: × 63 = 3 × 9 = 27] 3 3 3
7
4. 1
C. 1. True. Hint: Part of homework completed in
1 1
Hint: of ` 300 = × 300 = ` 10 1
30 30 1 hour =
3
2. True. Part of homework completed in 3 hours
1 1 15 3 1
Hint: of fifteen = ×15 = =3 = 3 × =1
4 4 4 4 3
1 III. 1. (i) 10 students
3. False. [Hint: 7 × 0 = 0 ]
6
1 1 57 Hint: Number of students who like chapati
4. True. [Hint: 8 ×1 = 8 = ]
7 7 7 1
1 only = 50 × = 10
5. True. [Hint: 7 × 0 = 0 ] 5
9 (ii) 20 students
6. False
Hint: Number of students who like rice
1 22 110 2 2
[Hint: 5 × 7 = 5 ×
= = 36 ] only = 50 × = 20
3 3 3 3 5
II. 1. (iii) 20

3 Hint: Number of students who like both


= + + = 4 × =3 chapati and rice = Total number of students
4
– (Number of students who like chapati
1 only + Number of students who like rice
2. of 90
5 only)
2 2 2 × 36
Hint: of 36 = × 36 = = 24 = 50 – (10 + 20) = 50 – 30 = 20
3 3 3
1 1 1 × 90
and of 90 = × 90 = = 18
2.
=
5 5 5
2 1
Q 24 > 18, so of 36 > of 90. 3 6
3 5 2 × =
1 7 7
3. Flour: 35 cups, sugar: 23 cups
3 3. (i)

Hint: For 1 packet, flour required
1 7
= 3 cups = cups
2 2
For 10 packets, flour required

Fractions and Decimals  29


1 Sleeping = 8 hrs
Hint: of the circles in box (a)
2 TV and sports = 4 hrs
1 1 School = 6 hrs
= of 12 = # 12 = 6 Meals = 2 hrs
2 2 1
So, we shade 6 circles. Hint: Self study: × 24 = 4 hrs,
6
1
(ii) Sleeping: × 24 = 8 hrs,
3
1
TV and sports: × 24 = 4 hrs
6
1
2 School: × 24 = 6 hrs ,
Hint: of the triangles in box (b) 4
3 1
2 2×9 Meals: × 24 = 2 hrs.
= of 9 = =6 12
3 3 5
2. (i)
13
So, we shade 6 triangles.
Hint: Number of trees required for 1 tonne
(iii) paper = 17
Number of trees required for 5 tonnes paper
= 5 × 17 = 85
85 5
Fraction of forest used = =
221 13
3 3 × 15
Hint: of 15 = =9 (Q No. of trees in forest = 221)
5 5
So, we shade 9 squares. (ii) 7 tonnes
4. (i) 2l
Hint: Saving 7 of forest
Hint: Amount of water consumed by Vidya 13
2 7
= of total quantity of water = saving × 221 trees
5 13
2 2 = 119 trees
= of 5 litres = # 5 litres = 2 litres
5 5
3 saving 17 trees, saves 1 tonne paper
(ii)
5 1
Hint: Remaining water = water consumed saving 1 tree, saves tonnes paper
17
by Pratap 119
= 5 litres – 2 litres = 3 litres saving 119 tree, saves tonnes paper
17
\ Fraction of water consumed by Pratap i.e. 7 tonnes of paper.
3 (iii) Forests are very important for ecological
=
5 balance. They reduce pollution, save
IV. 1. Given below is the account of how wild life and stop soil erosion.
Manav spends his day. Express each
3. (i) 180
as a fraction and pictorially using a pie
chart. Hint: Number of students influenced by
radio
Self study = 4 hrs

30 Mathematics–7
9 9 influenced by radio than by a music channel.
= of total = × 400 (iii) 100
20 20
Hint: No. of students influenced by
= 9 × 20 = 180 3
(ii) 148 friend or relative = × 400 = 60
20
Hint: Number of students influenced by No. of students heard CD in a shop
2 1
music video channel = 25 × 400 = 32 = × 400 = 40
10
So, 180 – 32 = 148 more students were Total = 60 + 40 = 100.

2.3  Multiplication of a Fraction by a Fraction


• To multiply a fraction by another fraction, we multiply the numerator of the first fraction
by the numerator of the second fraction and the denominator of the first fraction by the
denominator of the second fraction. The first product becomes the numerator and the second
product becomes the denominator of the required product (fraction).
Product of numerators
Thus, product of two fractions =
Product of denominators
1 2 1× 2 2 3 9 3 × 9 27 1 4 15 4 60 12 5
e.g. × = = ; × = = ; 2 × = × = = =1
5 7 5 × 7 35 5 7 5 × 7 35 7 5 7 5 35 7 7
Exercise 2.3
I. Very Short Answer Type Questions (1 Mark)
A. Answer the following.
1. What is the product of two proper fractions?
2. What is the product of two improper fractions?
1 1
3. What is of ?
4 4
B. Fill in the blanks.
2 1 12
1. 1 of 3 is ............... . 2. of is ............... .
2 7 9 5

3 4 21 2 4 8
3. × = 4. × =
5 8 40 4 9 45
C. Say True or False.
1. One third of one third is one ninth.
1 6 2
2. of is .
9 5 15
5 7 1
3. The lowest form of the product 4 × is 3 .
7 9 3

4. The product of two proper fraction is greater than each of the two fractions.

Fractions and Decimals  31


II.  Short Answer Type Questions–I (2 Marks)
1. Find the following.
1 1 4
(i) 6 8 ×1 4 (ii) 3 × 5 × 2
11 3 4 6 7
3 7 5 4
2. Multiply: (i) by (ii) ×
7 3 8 9
3 5 7 210 58
3. Find: × × 4. Find: ×1
8 7 9 256 70
4 85
5. What is the product of 3 and ?
5 33
2 4
6. Kanu ate part of a cake while her brother Manu ate of the remaining. What part
7 5
of the cake is left?
1
7. Renu finishes part of a work in 1 hour. How much part of the work will be finished
3
1
in 2 hours?
5
III. Short Answer Type Questions–II (3 Marks)
1 1
1. If the cost of 1 m cloth is ` 20 , then find the cost of 1 m of cloth?
4 3
7 13 5 7 15 12 16 5
2. Multiply (i) d – n×d – n (ii) d × n – d × n
5 15 12 18 8 5 15 8
1 1 13 15 2 1 4 1 2 4
(iii) d – n+ d × n (iv) d – + n × d + – n
4 12 14 52 3 2 5 3 5 15
1
3. In a hurdle race, Ridhi is over hurdle C and of the way through the race as shown in
2
the following figure.

2
(i) Where will Ridhi be, when she is of the way through the race?
3
5
(ii) W
here will Ridhi be, when she is of the way through the race?
6
IV.  Long Answer Type Questions (4 Marks)
2 4 3 2
1. Multiply the sum of and by the sum of and .
5 15 7 21
1 1
2. The speed of a car is 45 km/h. Find the distance covered in 3 hours.
2 7
3. Use the following fractions to answer the questions that follow.
1 1 2 3 5
, , , ,
3 2 5 4 6

32 Mathematics–7
1
(i) Which two fractions give the product ?
4
1
(ii) Which two fractions give the product ?
3
3
(iii) Which two fractions give the product ?
10
5
(iv) Which two fractions give the product ?
18
4 3
4. A rectangular field is 38 m long and 17 m wide. Find its area.
7 20
5. The dosage direction for a pain reliever recommends that an adult of 60 kg and above can take
1
4 tablets every 4 hours and an adult who weighs between 40 to 50 kg can take only 2
4 2
tablets every 4 hours. Each tablet weighs g.
25
(i) If a 72 kg adult takes 4 tablets, how many grams of pain reliever is he or she receiving?
(ii) How many grams of pain reliever is the recommended dose for an adult weighing
46 kg?
Answers and Hints
I. A. 1. A proper fraction. 1 5
2. An improper fraction 3. (ii)
16 18
5 4 5× 4 20 5
23 4 Hint: × = = =
B. 1. 2. 3. 7 8 9 8×9 72 18
14 15
5 3×5× 7 5
4. 5 3. [Hint: = ]
24 8 × 7 × 9 24
1 1 1 1 1
C. 1. True [Hint: of = × = ] 3 210 128 3
3 3 3 3 9 4. [Hint: × = ]
2 256 70 2
1 6 1× 6 2
2. True [Hint: × = = ] 26
9 5 9 × 5 15 5. 9
33
33 7 33 11 2 19 85 19 × 17 323 26
3. False [Hint: × = = =3 ]
7 9 9 3 3 Hint: × = = =9
5 33 33 33 33
4. False 1
6. part
23 7
74 7 518 23
II. 1. (i) 15 [Hint: × = = 15 ] Hint: Part of cake left after Kanu ate
33 11 3 33 33
5 1 1 4 2 5
(ii) 43 Hint: 3 × 5 × 2 = 1 – =
28 4 6 7 7 7 4 5 4 5
Part of cake eaten by Manu = of = ×
13 31 18 1209 5 5 7 5 7
= × × = = 43 4
4 6 7 28 28 =
7
2. (i) 1 Part of cake left after Manu ate
3 7 (3) × (7) 21
×d n =
5 4 1
Hint: = =1 = – =
7 3 7×3 21 7 7 7

Fractions and Decimals  33


11 29 7 203
7. part = × =
15 30 15 450
1
Hint: Part of work finished in 1 hour = 3. (i) D
1 3
Part of work finished in 2 hour Hint: Total number of parts = 6
1 1 1 11 11 5
= × 2 = × = 2 2
3 5 3 5 15 of 6 = × 6 = 2 × 2 = 4 parts
3 3
III. 1. `27
Ridhi will be at the hurdle D.
1
Hint: Cost of 1 m of cloth = `20 (ii) E
4
81 5
= ` Hint: of 6 = 5 parts
4 6
1 4 4 81
Cost of 1 m = m cloth = × = ` 27 Ridhi will be at the hurdle E.
3 3 3 4
2 22
2. (i) IV. 1.
135 63
2 4 3 2
Hint:
21 – 13 15 – 14
× Hint: d + n × d + n
15 36 5 15 7 21
8 1 2 6+4 9+2
= × = = d n×d n
15 36 135 15 21
5 10 11 2 11 22
(ii) 3 = × = × =
6 15 21 3 21 63
3 3 2 1
Hint: d × n – d × n 2. 143 km
2 1 3 1
9 2 9 2 27 – 4
= – d n = – = Hint: Distance covered = Speed × Time
2 3 2 3 6 1 1 91 22
23 5 = 45 × 3 = ×
= = 3 2 7 2 7
6 6 = 13 × 11 = 143 km
73
(iii) 1 3 2 5
168 3. (i) , (ii) ,
3 – 1 15 3 4 5 6
Hint: + 2 3 1 5
12 56 (iii) , (iv) ,
2 15 1 15 28 + 45 5 4 3 6
= + = + = 1
12 56 6 56 168 4. 661 m 2
73 2
= 4 270
168 Hint: l = 38 m = m
203 7 7
(iv) 3 343
450 b = 17 m = m
2 1 4 1 2 4 20 20
Hint: d – + n × d + – n
3 2 5 3 5 15 270 343
Area = l × b = ×
20 – 15 + 24 5+6– 4 7 20
= d n×d n 27 × 49 1323 1
30 15 = = = 661 m2
2 2 2

34 Mathematics–7
16 Hint: A 46 kg adult can take
5. g
25 1 5
Hint: (i) Weights of tablets taken by a 2 tablets = tablets
2 2
72  kg adult
5 5 4 2
4 16 Weight of tablets = × = g
= 4× = g 2 2 25 5
2 25 25
(ii) g
5
2.4 Division of Fractions
Reciprocal of a Fraction
a b 5 7
• Reciprocal of any fraction b is a . For example, reciprocal of is .
7 5
Division of Whole Number by a Fraction
• To divide a whole number by any fraction, multiply that whole number by the reciprocal of
3 4 8
that fraction. For example, 2 ' = 2× = .
4 3 3
Division of a Fraction by a Non-zero whole Number
• To divide a fraction by a non-zero whole number, multiply the fraction by the reciprocal of
that whole number.
3 3 1 1
For example. '9 = × =
5 5 9 15
Division of a Fraction by Another Fraction
• To divide a fraction by another fraction, multiply the first fraction by the reciprocal of second
a c a c a d
fraction, i.e., if and are two fractions, then ' = ×
b d b d b c
7 14 7 30 5
For example. ' = × =
18 30 18 14 6
Note: Before performing an operation, change mixed fractions to improper fractions.
2 8 3 2 22 22 1 22 7
For example. 16 ÷ 2 = 16 ÷ = 16 × = 6; 4 ÷ 3 = ÷3= × = =1
3 3 8 5 5 5 3 15 15
1 9 31 9 31 2 31 4 1 2 16 5 16 3 48 23
5 ÷ = ÷ = × = =1 ; 3 ÷ 1 = ' = × = =1
6 2 6 2 6 9 27 27 5 3 5 3 5 5 25 25

Simplification
• The sequence of operations followed while simplifying a numerical expression is
(i) Bracket (ii) Of (iii) Division
(iv) Multiplication (v) Addition (vi) Subtraction
• In case of brackets the sequence followed to solve an expression is —–, (  ), {  }, [  ], i.e.,
from the innermost to the outermost.

Fractions and Decimals  35


Exercise 2.4
I. Very Short Answer Type Questions (1 Mark)
A. Answer the following.
1. Reciprocal of a number greater than 1 is more than 1 or less than 1?
2. What is the product of a fraction and its reciprocal?
3. What is the reciprocal of a proper fraction?
2 1 2 1
4. What is the value of  ? 5. What is the reciprocal of ' ?
5 2 9 2
7
6. Find the reciprocal of .
8
B. Fill in the blanks.
4
1. The reciprocal of is ............... .
7
2. While dividing a fraction by another fraction, we ............... the first fraction by the ...............
of the another fraction.
3. Reciprocal of a unit fraction is a ............... number.
5
4. The reciprocal of is .............. .
3
5 20
5. 4 x = (Use + , –, × or ÷)
3 3
C. Say True or False.
1. The reciprocal of a fraction may not be a fraction.
6 9 1 7 1 9
2. ÷ 6 is equal to . 3.  ´ .
8 2 7 9 7 7
3 18
4. 3 ÷ 5 is equal to .
5 25
II.  Short Answer Type Questions–I (2 Marks)
A. Find the following.
4 1 2 2
1. 4 ÷ 2 2. 8 ÷ 1 3. 1 ' 5
3 7 3 3
1 2 1 2
4. 3'1 5. 7 ' 2
5 3 9 3
B. Say True or False. Justify your answer.
3
1. 10 pieces of each kg can be cut from a cake of weight 6 kg.
5
2. Reciprocal of 0 is 0.

3. The product of a fraction and its reciprocal is 1.

36 Mathematics–7
III.  Short Answer Type Questions–II (3 Marks)
1
1. How many kg boxes of milk cake can be made with 5 kg of milk cake?
8 1
2. Radhika is making bookmarks of length 20 cm each. How many bookmarks can she
make from a 41 cm long ribbon? 2

3. P.P. School organised a trekking trip for the students. Jayant said that they have gone
about 240 km or 2 of the way to the campsite. How much further do we have to go?
3
What are the benefits of such school trips?  [VBQ]
1 3
4. Richa bought 2 kg potatoes for ` 21 . How much does 1 kg potatoes cost?
2 4

IV.  Long Answer Type Questions (4 Marks) 1 1


2 1 2 2 + 1
1. Simplify: d1 ' n + d1 ' 3 n – d1 ' 2 n 2. Simplify: 2 5 –
9 5 3 1 1 5
2 '
2 5
3 1
3. The cost of 3 kg oranges is `124. How much will 4 kg oranges cost?
4 2
3 1
4. The product of two numbers is 68 . If one of the numbers is 12 , find the other.
4 2
3 7
5. The area of a rectangular room is 87 sq. m. If it is 4 m wide, find its length.
4 8

Answers and Hints


7
I. A. 1. Reciprocal of a number greater than B. 1. 2. multiply, reciprocal
4 3
1 is less than 1,
3. whole 4. 5. ×
1 5 3 5
e.g., reciprocal of 2 = < 1 and is < 1.
2 3 5 C. 1. True
2. Product of a fraction and its reciprocal 6 6 1 1
2. False [Hint: ÷6= × = ]
is 1. 8 8 6 8
3. Reciprocal of a proper fraction is an 1 7 1 9
3. True [Hint: ÷ = × ]
improper fraction. 7 9 7 7
4 2 1 2 4 4. True
4. [Hint: ÷ = × 2 = . ]
5 5 2 5 5
3 18 18 1 18
9 2 1 2 4 Hint: 3 ÷ 5 = ÷ 5 = × =
5. Hint: ' = ×2 = 5 5 5 5 25
4 9 2 9 9 22
4 9 II. A. 1. 2
Reciprocal of = 45
9 4 4 1 16 15
8 Hint: 4 ' 2 = '
6. 3 7 3 7
7 16 7 112 22
= × = =2
3 15 45 45

Fractions and Decimals  37


4 2 240 × 3
2. 4 x = 240 ÷ =
⇒ = 120 × 3 = 360 km
5 3 2
2 5 3 24 4
Hint: 8 ÷ 1 = 8 ÷ = 8 × = =4 Distance to be covered = (360 – 240) km
3 3 5 5 5 = 120 km
1 2 5 5 1 1 7
3. [Hint: 1 ' 5 = ' 5 = × = ] 4. `8
3 3 3 3 5 3 10
23 1 2 16 5 1 3
4. 1 Hint: 3 ' 1 = ' [Hint: Cost of 2 kg potatoes = ` 21
25 5 3 5 3 2 4
16 3 48 23 87
= × = = 1 = `
5 5 25 25 4 87 5
Cost of 1 kg potatoes = ` '
2 4 2
5. 2 87 2 87 7
3 1 2 64 8 = × = =`8
Hint: 7 ' 2 = ' 4 5 10 10
9 3 9 3 7
64 3 8 2 IV. 1. 4
= × = =2 16
9 8 3 3 2 1 2
Hint: d1 ' n + d1 ' 3 n – d1 ' 2 n
B. 1. True,  total weight of cake = 6 kg 9 5 3
2 16 8
Weight of each piece = kg
3
= d1 ' n + d1 ' n – d1 ' n
5 9 5 3
Number of each pieces
9 5 3
3 5 = d1 × n + d1 × n – d1 × n
= 6 ÷ = 6 × = 2 × 5 = 10 2 16 8
5 3 9 5 3 72 + 5 – 6 77 – 6
2. False as reciprocal of 0 does not exist. = + – = =
a b 2 16 8 16 16
3. True as × = 1 71 7
b a = = 4
16 16
III. 1. 40 boxes 2
2.
Hint: Total weight of milk cake = 5 kg 125
1 1 1 5 1
Weight of 1 box = kg 2 + 1 +
8 Hint: 2 5 – = 2 5 –1
1 8 1 1 5 5 1 5
Number of boxes = 5 ÷ = 5 × = 40 2 ' '
8 1 2 5 2 5
2. 2 25 + 2
1
= 10 – 1 = 27 × 2 – 1
Hint: Length of one bookmark = 20 cm 5 5 10 25 5
2
41 ×5
= cm 2
2 27 1 27 – 25 2
Total length of ribbon = 41 cm = – = =
125 5 125 125
41 3
Number of bookmarks = 41 ' 3. Cost of 3 kg oranges = `124
2 2 4 3
= 41× = 2 Cost of 1 kg oranges = 124 ' 3
41 4
3. 120 km 15 4 496
Hint: Let total distance be x km = 124 ' = 124 × = `
4 15 15
2 1 9
Distance covered = x = 240 km Cost of 4 kg i.e; kg oranges
3 2 2

38 Mathematics–7
496 9 5. 18 m
= # = AB 3
15 2 Hint: Area of the room = 87 sq m.
1 351 4
4. 5 = sqm
2 4
3 275 7 39
Hint: Product of two numbers = 68 = Width of the room = 4 m = m
4 4 8 8
1 25 \ Length of the room = Area ÷ Width
One of the numbers is 12 = 351 39 351 8
2 2 = ' = #
\ Other number = Product ÷ Given number 4 8 4 39
275 25 275 2 11 1 = 9 × 2 = 18 m
= ' = × = = 5
4 2 4 25 2 2
2.5  Decimals
• A fraction whose denominator is 10 or 100 or 1000 or 10,000,... etc., is called a decimal
3 2 4125
fraction. For example, , , , etc., are decimal fractions.
10 100 1000
• If decimal fractions are written using a decimal point, then they are called decimal numbers
or simply decimal. For example, the above decimal fractions can be written as 0.3, 0.02,
4.125, etc.
0.02 4.125
Whole part Decimal part Whole part Decimal part
Let us write some decimals using place value table.

Decimal numbers Hundreds Tens Ones • Tenths Hundredths Thousandths


D
4 E 1 2 5
4.125 C
I
6 3 M 0 9 2
63.092 A
L
P
O
483.714 4 8 3 I 7 1 4
N
T
• A decimal number can be written in the expanded form. For example, the expanded form of
7 1 4
483.714 = 4 × 100 + 8 × 10 + 3 × 1 + + +
10 100 1000
• Like decimals are decimal numbers in which number of digits in decimal part is same.
e.g. 2.5, 3.8, 0.7, are like decimals whereas 3.08, 9.5, 3.436 are unlike decimals.
• To compare decimal numbers, we start with the whole number part. If the whole number parts
are equal, we compare tenths digits and so on.
For example, 3.7 > 2.9, 0.77 > 0.25, 8.7 > 8.45
Example 1: Arrange in ascending order: 6.6, 0.93, 2.87, 2.9

Fractions and Decimals  39


Solution. Changing to like decimals, we get 6.60, 0.93, 2.87, 2.90
Since 0.93 < 2.87 < 2.90 < 6.60. Hence, the numbers in the ascending order are
0.93, 2.87, 2.90, 6.60
• Addition and subtraction of decimal numbers and addition and substraction of whole numbers
are the same. But, if the given decimal numbers are not like decimals, convert them into like
decimal. Do not forget to place the decimal in the result.
Example 2: Add 2.9, 0.8 and 3.56
2 . 90 First convert unlike decimals into like decimals.
+ 0 . 80 Write the decimal points directly below to each
+ 3 . 56 other and rest of the digits in the correct place value
7 . 26 columns. Then add like whole numbers addition.

Example 3: Subtract 7.16 from 22.5.


Solution. Changing to like decimals,
we get 7.16 and 22.50. First convert unlike decimals into like decimals.
22.50 Write the decimal point directly below to each
Now, –7.16 other and rest of the digits in the correct place
value columns. Then subtract like whole numbers
15.34
subtraction.

Exercise 2.5
I. Very Short Answer Type Questions (1 Mark)
A. Answer the following.
9 11
1. Is a decimal fraction? 2. Write as a decimal.
80 8
579
3. Write as a decimal fraction.
125
B. Fill in the blanks.
1. 0.2 + 0.05 = .................. .
2. 0.9 – 0.23 = .................. .
3. Expanded form of 350.103 is .................. .
II.  Short Answer Type Questions–I (2 Marks)
1. Arrange in the ascending order: 2.01, 3.1, 2.14, 0.689
2. Arrange in descending order: 7.8, 8.7, 17.5, 4.93, 3.12
3. Add: 13.21, 12 and 15.869
4. Subtract 189.62 from 200.1

III.  Short Answer Type Questions–II (3 Marks)


1. Kusum paid an electricity bill of `475.90 out of a 2000 rupee note. How much change
should she have received back?

40 Mathematics–7
2. The atomic weight of Helium is 4.0030 m, of Hydrogen is 1.0080 m, and of Oxygen is
16.000 m. Find the difference between the atomic weights  of
(i) Oxygen and Hydrogen (ii) Oxygen and Helium
(iii) Helium and Hydrogen
3. The normal body temperature is 98.6°F. When Savitri was ill, her temperature rose to
102.4°F. How many degrees above normal was that?
4. Sapna weights 53.8 kg. What will be her weight after losing 8.95 kg?
IV.  Long Answer Type Questions (4 Marks)
1. The table below shows average global temperature measure of several years.
Year Difference from the base
1958 0.10°C
1964 – 0.17°C
1965 – 0.10°C

 1
1978   C
50
2002 0.54°C
(i) Order the five years from coldest to warmest.
(ii) In a particular year, the average temperature varied by –0.13°C from the base measure.
Between which two years should this particular year fall, when the years are ordered
from coldest to warmest?
2. Kamini bought a radio for `325.80, a fan for `809.06 and a chair for `201.75. How much
did she spend on all these items?
3. Rajat has 30.5 litres petrol in his car. He drove his car and used 18.95 litres petrol. How
much petrol will he need to fill in the tank if it can hold 45 litres?

Answers and Hints


11 = `2000.00 – 475.90
I. A. 1. No 2. = 1.375 = `1524.10
8
579 2. (i) 14.9920 m
3. = 4.632
125 Hint: 16.000 m – 1.0080 m = 14.9920 m
B. 1. 0.25 2. 0.67
1 3 (ii) 11.9970 m
3. 3 × 100 + 5 × 10 + + Hint: 16.000 m – 4.0030 m = 11.9970 m
10 1000
(iii) 2.9950 m
II. 1. 0.689, 2.01, 2.14, 3.1
Hint: 4.0030 m – 1.0080 m = 2.9950 m
2. 17.5, 8.7, 7.8, 4.93, 3.12
3. 3.8°F
3. 41.079 4. 10.48
III. 1. `1524.10 Hint: Number of degrees above normal
Hint: `2,000 – `475.90 = 102.4°F – 98.6°F = 3.8°F

Fractions and Decimals  41


4. 44.85 kg Cost of fan = + `809.06
Hint: Sapna’s weight = 53.8 kg Cost of chair = + `201.75
Weight loss = 8.95 kg Total amount spent = `1336.61
Sapna’s weight after losing = 53.80
3. 33.45L
–  8.95
Hint: Quantity of petrol Rajat had = 30.50L
44.85 kg
Quantity of petrol Rajat used = – 18.95L
IV. 1. (i) 1964, 1965, 1978, 1958, 2002
1 Quantity of petrol left in car = 11.55L
Hint: –0.17°C < –0.10°C < d = 0.02 n °C
50 To have 45L of petrol in tank he needs
< –0.10°C < 0.54°C 45.00L
(ii) Between 1964 and 1978
–11.55 L
2. 1336.61
Hint: Cost of radio = `325.80 33.45 L

2.6  Multiplication of Decimals


Multiplication of a Decimal by a Whole Number
• To multiply a decimal by a whole number, we multiply like whole numbers ignoring the
decimal point. We put decimal point in the product leaving from the right, as many decimal
places as are in the given decimal number.
604 604 ×12 7248
For example, 6.04 × 12 = × 12 = = = 72.48
100 100 100
Or multiply: 604 × 12 = 7248
Now, place decimal point leaving two digits i.e. 72.48
Multiplication of Decimals by 10, 100 and 1000
• To multiply a decimal by 10, shift the decimal point 1 place to the right.
For example, 5.16 × 10 = 51.6
• To multiply a decimal by 100, shift decimal point by two places to the right.
For example, 83.702 × 100 = 8370.2
• To multiply a decimal by 1000, shift the decimal point by three places to the right.
For example, 501.06 × 1000 = 501060.0 = 501060
Multiplication of a Decimal by Decimal
• To find the product of two decimals, multiply the numbers in the same way as for whole
numbers. Then put the decimal point in the product leaving from the right as many decimal
places as the total number of decimal places in the multiplicand and the multiplier.
3468 × 12 41616
For example, 34.68 × 1.2 = = = 41.616 Or 34.68  ×  1.2 = 41.616
100 × 10 1000 2 places   1 place   3 places

Exercise 2.6
I. Very Short Answer Type Questions (1 Mark)
A. Fill in the blanks.
1. 0.8 × 9 = ............... . 2. 6.95 × 5 = ............... .

42 Mathematics–7
3. 2.675 × 10 = ............... . 4. 18.673 × 100 = ............... .
5. 3.5679 × 1000 = ............... .
6. Number of decimal places in the product of 124.9 × 0.002 is ............... .
B. Find the products.
1. 0.2 × 0.3 = 2. 200.48 × 3.8 = 3. 12 × 4.08 =
4. 4.095 × 10 = 5. 6.924 × 100 = 6. 0.0842 × 1000 =
II.  Short Answer Type Questions-I (2 Marks)
1. Give pictorial representation of 0.2 × 0.5.
2. Navneet purchased 4.5 kg of tomatoes for soup at the rate of `40.75 per kg. How much
money should he pay in nearest rupees?
3. Find the continued product of 2.5 × 8.9 × 0.3
III.  Short Answer Type Questions–II (3 Marks)
1. Raghav bought 2.5 m cloth at `155.5 per metre. What did he pay for it?
2. Jaina earns `108.50 per hour. If she works for 11.5 hours, how much does she earn?
3. Find the mass of 2.8 litres of oil if mass of each litre is 0.96 kg.
IV.  Long Answer Type Questions (4 Marks)
1. Vijaya bought 2.5 m lace which costs `72.50 per metre. If she handed over a `500 note
to the shopkeeper, how much change would she receive?
2. A dealer sold 15 bags of rice weighing 25.8 kg each and 10 bags of wheat weighing
30.2 kg each. Find the total weight of the grains.
3. Chhavi bought 1750 g potatoes at `18.60 per kg, 3250 g cauliflower at `29.80 per kg.
How much total vegetables (in kg) did she buy and what did she pay to the shopkeeper?

Answers and Hints


I. A. 1. 7.2 2. 34.75 3. 26.75 3. 6.675
4. 1867.3 5. 3567.9 6. four Hint: We find the product of 25 × 89 × 3
B. 1. 0.06 and put the decimal point in the product.
2. 761.824 [Hint: 200.48 × 3.8 = 761.824] We have, 2 5 7 5
3. 48.96 × 3 × 8 9
4. 40.95 [Hint: 4.095 × 10 = 40.95] 7 5 6 7 5
5. 692.4 [Hint: 6.924 × 100 = 692.4] 6000
6. 84.2 [Hint: 0.0842 × 1000 = 84.2] 6675
Total number of decimal places in the
II. 1. = 0.2 × 0.5 = 0.1 given the decimals = 1 + 1 + 1 = 3
\ 2.5 × 8.9 × 0.3 = 6.675
2. `183.375 III. 1. 388.75
Hint: Price of 1 kg of tomatoes = `40.75 Hint: Cost of 1 m of cloth = ` 155.5
Price of 4.5 kg of tomatoes = 4.5 × `40.75 Cost of 2.5 m of cloth = ` 155.5 × 2.5
= `183.375 = ` 388.75

Fractions and Decimals  43


2. `1247.75 kg Total weight of the grains = 387
Hint: Jaina earns in 1 hour = `108.50 + 302
Jaina earns in 11.5 hours = `108.50 × 11.5 689 kg
= `1247.75 3. 5 kg, `129.40
3. 2.668 kg 1750
Hint: Mass of 1 litre of oil = 0.96 kg Hint: Weight of potatoes in kg =
1000
Mass of 2.8 litre of oil = 0.96 =1.75 kg
× 2.8 3250
= 2.688 kg Weight of cauliflower in kg = kg
1000
IV. 1. `318.75 = 3.25 kg
Hint: Cost of 1 m of lace = `72.50 Total weight = 1. 7 5
Cost of 2.5 m of lace = ` 72.5 × 2.5 + 3. 2 5
= `181.25 5. 0 0 kg
She gave `500 to the shopkeeper.
\ Change she would receive Cost of potatoes per kg = `18.60
= `500 – `181.25 = `318.75 Cost of 1.75 kg = 1.75 × 18.60
2. 689 kg
= `32.55
Hint: Weight of 1 bag of rice = 25.8 kg Cost of 1 kg of cauliflower = `29.80
Weight of 15 bags of rice Cost of 3.25 kg of cauliflower
= (25.8 × 15) kg = 387 kg = `29.8 × 3.25 = `96.85
Weight of 1 bag of wheat = 30.2 kg Total amount paid = `32.55
Weight of 10 bags of wheat + `96.85
= (30.2 × 10) kg = 302 kg `129.40

2.7 Division of Decimals
Division of Decimals by a Non-zero Whole Number
• To divide a decimal by a non-zero whole number, we divide like whole numbers ignoring
the decimal point. Then we put the decimal point in the quotient leaving from the right as
many decimal places as are in the given decimal.
Example 1: Divide 9.6 by 3
9.6 96 1 32
Solution: 9.6 ÷ 3 = = × = = 3.2
3 10 3 10
Example 2: Divide 13.5 by 5
1 135 1 27
Solution: 13.5 ÷ 5 = 13.5 × = × = = 2.7
5 10 5 10
Division of Decimals by 10, 100 and 1000
• While dividing a number by 10, 100 or 1000, the digits of the number and the quotient are same
but the decimal point in the quotient shifts to the left by as many places as there are zeroes
over 1.

44 Mathematics–7
Example 3: Find the following. (i) 41.5 ÷ 10 (ii) 46.8 ÷ 100
1 415 1 415
Solution: (i) 41.5 ÷ 10 = 41.5 × = × = = 4.15
10 10 10 100
1 468 1 468
(ii) 46.8 ÷ 100 = 46.8 × = × = = 0.468
100 10 100 1000

Division of a Decimal Number by Another Decimal Number


• To divide a decimal number, shift the decimal point in the dividend and the divisor to the
right by as many places as there are in the divisor so as to make divisor a whole number.
Then divide.
Example 4: Divide 0.64 by 0.4
0.64 0.64 10 6.4 64 1 16
Solution. 0.64 ÷ 0.4 = = × = = × = = 1.6
0.4 0.4 10 4 10 4 10
Exercise 2.7
I. Very Short Answer Type Questions (1 Mark)
A. Fill in the blanks.
1. 6.05 ÷ 10 = ............... . 2. 0.8 ÷ 20 = ............... .
3. 7.2 ÷ 10 = ............... . 4. 0.96 ÷ 100 = ............... .
5. 0.048 ÷ 1000 = ............... .
B. Say True or False.
1. 58.37 ÷ 1000 = 0.5837 2. 98.4 ÷ 200 = 49.20
3. To divide a decimal number by 100, we shift the decimal point two places to the right.
4. To divide a decimal number by 100, we shift the decimal point two places to the left.
C. Find the following.
1. 7 ÷ 3.5 2. 57.44 ÷ 8
3. 3.25 ÷ 0.5 4. 30.94 ÷ 0.7
II.  Short Answer Type Questions–I (2 Marks)
A. Solve the following.
19
1. Kavita had a piece of rope of length m. It is to be cut into small pieces of length
2
of 1.9  m each. How many pieces of the required length will she got out of this rope?
2. Three girls carry a total of `345.60 in the purse. What was the average amount per girl?
B. Say True or False. Justify your answer.
7 7
1. The number which when multiplied by gives the product 10 is .
9 90
2. Reciprocal of –1 is –1. 1
3. Multiplicative inverse of 8 is .
8
III.  Short Answer Type Questions–II (3 Marks)
1
1. A public electricity line is being installed along 160 m of road. The labourer will be
4
able to complete 7.5 m in one day. How long will the project take to complete?

Fractions and Decimals  45


2. A school uniform requires 3.25 m cloth. How many uniforms will be prepared from
318.5 m cloth.
3. The product of two numbers is 59.328. If one of the numbers is 12.36, find the other
number.
IV.  Long Answer Type Questions (4 Marks)
1. Rachna paid `549.60 for 12 notebooks. If she has to buy 2 more notebooks, how much
more money will she pay to the bookseller?
2. Each side of a regular polygon is 3.4 cm long in length. The perimeter of the polygon is
20.4 cm. How many sides does the polygon have? Also, name the polygon.
3. A vehicle covers 43.2 km in 2.4 L of petrol. How many litres of petrol is required to
cover a distance of 108 km.

Answers and Hints


I. A. 1. 0.605 2. 0.04 7 9 90
Then, x × = 10 ⇒ x = 10 × =
3. 0.72 [Hint: 7.2 ÷ 10 = 0.72] 9 7 7
4 . 0.0096 [Hint: 0.96 ÷ 100 = 0.0096] 1
5. 0.000048 2. True, as reciprocal of −1 = = −1
( −1) 1
B. 1. False 2. False 3. False 3. True, as multiplicative inverse of 8 =
11 8
4. True III. 1. 21 days
7 7 × 10 30
C. 1. 2 [Hint: 7 ÷ 3.5 = = = 2]
3.5 35 Hint: Total length of the road
2. 7.18 Hint: 57.44 ÷ 8 1 641
= 160 m = m
5744 1 718 4 4
= × = = 7.18 Length of the road covered in 1 day
100 8 100 75 15
3.25 10
3. 6.5 [Hint: 3.25 ÷ 0.5 = × = 7.5 m = m= m
0.5 10 10 2
32.5 Number of days required
= = 6.5
5 641 15 641 2 641 11
= ÷ = × = = 21
4. 44.2 4 2 4 15 30 30
2. 98
II. A. 1. 5 pieces
Hint: Total cloth = 318.5 m
Hint: Number of pieces Cloth required for 1 uniform = 3.25 m
19 318.5
length of rope 19 Number of uniforms = = 98
= = 2 = ' 1.9 3.25
length of each piece 1.9 2 3. 4.8
19 10 10
= × = = 5 pieces Hint: Product of two numbers = 59.328
2 19 2
One number = 12.36
2. `115.2 59.328
Hint: Average amount per girl Other number = = 4.8
12.36
` 345.60 IV. 1. `91.60
= 3
= ` 115.20
B. 1. False as suppose the required number Hint: Cost of 12 books = `549.60
be x. Cost of 1 book = 549.60 ÷ 12 = `45.80

46 Mathematics–7
Cost of 2 books = `45.80 × 2 = `91.60 Hint: Distance covered 43.2 km
2. 6, Hexagon Petrol consumed = 2.4L
43.2
Hint: We know that all sides of a regular Distance covered in 1L =
432 2.4
polygon are equal.
Perimeter 20.4 = = 18 km
24 1
No. of sides = = 1 km is covered in L
Length of each side 3.4 18
204 For covering a distance of 108 km petrol
= = 6.
34 1
3. 6L required = # 108 = 6L
18
Subject Enrichment Activities
I. Word-maze
here are six hidden words related to the chapter
T
‘Fractions and Decimals’ in the given word-maze.
Let us find them. One is done for you.

II. Match the columns.


Column I Column II
(i) 0.07 + 0.09 (a) 0.85
(ii) 0.5 + 0.8 (b) 0.14
(iii) 0.9 – 0.05 (c) 890 cm
(iv) 0.05 + 0.09 (d) 0.16
(v) 8.9 m (e) 1.3

III. Multiple Choice Questions


Choose the correct answers.
1
1. What is the reciprocal of 3 ?
7
7 22 7 1
(a) 3 (b) (c) (d)
1 7 22 21
5
2. The product of 2 × 6 is
12
5 5 5 1
(a) 12 (b) 2 (c) 3 (d) 14
12 12 12 2
1 4
3. ×  3 is equal to
2 9
4 2 4 2
(a) (b) (c) (d)
3 3 12 27

Fractions and Decimals  47


5
4. A parking lot can hold 360 cars at a time. At a particular time, of parking lot is occupied.
12
How many more cars can be parked in the parking lot?
(a) 30 cars (b) 75 cars (c) 210 cars (d) 250 cars
3 1
5. of adults in a population are men, of these men are nonvegetarians. What fraction of the
5 3
total population are non-vegetarians?
1 1 3 14
(a) (b) (c) (d)
5 3 5 15
1 2
6. The perimeter of a rectangle, 20 m long and 12 m wide, is
3 5
1 11 7 2
(a) 100 m (b) 32 m (c) 65 m (d) 120 m
5 15 15 15
7. The perimeter of an equilateral triangle of side 9.6 cm is
(a) 19.2 cm (b) 27.6 cm (c) 28.8 cm (d) 22.52 cm

8. The number of sides of a regular polygon whose perimeter and length of each side are 50.4 cm
and 8.4 cm, respectively, is
(a) 5 (b) 6 (c) 7 (d) 8
1 1
9. The area of a rectangle, 24 cm long and 20 cm wide, is
2 5
1 9 2 13
(a) 246 cm 2 (b) 494 cm 2 (c) 90 cm 2 (d) 2 cm 2
4 10 5 102
3
10. Multiplicative inverse of 2 is
8
19 8 2 8
(a) (b) (c) (d)
8 19 8 3

Answers

I. 7 1
III. 1. (c) 2. (d) 14
22 2
2
3. (d) 4. (c) 210 cars
27
1 7
5. (a) 6. (c) 65 m
5 15
7. (c) 28.8 cm 8. (b) 6
9
9. (b) 494 cm 2
10
8
1 0. (b)
19
II. (i)—(d) (ii)—(e) (iii)—(a)
(iv)—(b) (v)—(c)
qqq

48 Mathematics–7
3 Data Handling
Topics Covered
3.1 Collecting and Organising Data for 3.2 Other Measures of Central Tendency –
Arithmetic Mean Median and Mode
3.3 Bar Graph 3.4 Probability

Let’s Remember
• The information collected in the form of numbers is called data.
Example: Sangeeta has 102°F fever is a data but simply saying Sangeeta has fever is not a
data.
• Data is of two types—Primary and Secondary data
• Primary data—Data collected by a person himself is called primary data. For example, to
know the performance of the class in mathematics, class teacher collected the math marks
of students of her class. Such data is called primary data.
• Secondary data—The data collected by one person used by some other person is called
secondary data.
For example, principal uses data collected by the class teachers of different sections of
class VIII to know the average performance of the students. Such data is called secondary data.

3.1 Collecting and Organising Data for Arithmetic Mean


Raw data (ungrouped data)
• The data collected and put without any specific arrangement is called raw data. Each entry
of raw data is called observation. For example, the shoe sizes of 15 students of your class.
5, 7, 5, 6, 6, 5, 8, 6, 5, 7, 5, 5, 7, 6, 6
• Arranging the data in ascending or descending order is called an array.
• We arrange the above shoe sizes in ascending order as below.
5, 5, 5, 5, 5, 5, 6, 6, 6, 6, 6, 7, 7, 7, 8.
Frequency distribution
• Look at the array data above. Shoe sizes are called variables and the number of times a
variable occurs is called its frequency. For example, here shoe size 6 is called a variable
and its frequency is 5. What is the frequency of 8?
• When the number of observation is large, it is difficult to arrange the data in array form.
We prepare a frequency table using tally marks. For this we take each observation from the
data, one at a time and put a tally mark (|) opposite to that observation. Look at the table
given below.

49
Shoe size Tally Marks Frequency
5 6

6 5

7 3

8 1
Total 15
• The difference between the highest and the lowest observation in a given data is called its
range. Here in above example, range is 8 – 5 = 3
• Data have a tendency to cluster at a centre or have a central tendency. Measures of central
tendency are mean, median and mode.
• The average or arithmetic mean or mean of a given data is defined as
Sum of all observations
Mean 
Total number of observations
Example: The attendance of students of a school for a week is as follows: 1425, 1430, 1400,
1408, 1410, and 1423. Find the mean attendance.
Solution. Sum of observations = 1425 + 1430 + 1400 + 1408 + 1410 + 1423 = 8496
Number of observation = 6
Sum of observations 8496
Mean = = = 1416
No. of observations 6
Exercise 3.1
I. Very Short Answer Type Questions (1 Mark)
A. Answer the following.
1. What is the range of the data?
2. How do you find mean of the data?
3. The marks in a subject for 12 students are as follows:
31, 37, 35, 38, 42, 23, 17, 18, 35, 25, 35, 29. For the given data, find the range.
4. Find the mean of 6, 2, 5, 4, 3, 4, 1.
5. Find the range of the following data.
45, 86, 62, 47, 80, 45, 31, 51, 48, 41, 78, 80, 99
6. Mean of 10 numbers is 45. Find the sum of the numbers.
B. Fill in the blanks.
1. The range of the data 14, 6, 12, 17, 21, 10, 4, 3 is ............... .
2. The mean of first five prime numbers is ............... .
3. On tossing a coin the outcomes are ............ .
C. Say True or False.
1. The range of the data 31, 16, 27, 28, 22, 18, 14, 23 is 17.
xyz
2. Let x, y, z be three observations. The mean of these observations is .
3

50 Mathematics–7
II. Short Answer Type Questions–I (2 Marks)
1. Would the range of the data 5, 9, 3, 0, 4, 8, –1, –3 change if 6 was added to each value
in the data?
2. Find the mean of first six multiples of 4.
3. Find the mean of 6, 9, 6, 4, 7, 2, 8.
III. Short Answer Type Questions–II (3 Marks)
1. Marks scored in a test of 20 marks by a group of students are: 11, 13, 15, 9 and 12. What
is the mean score? It was later discovered that a 3 mark question asked in the test was
incorrect, so the teacher decided to give 3 marks extra to each student. What will be the
new mean score?
2. For the given data, 6, 2, 5, 4, 3, 4, 4, 2, 3
(i) Calculate the mean.
(ii) How many observations are above the mean?
(iii) Find the range of the data. 
3. If the arithmetic mean of 8, 4, x, 6, 2, 7 is 5, then find the value of x.  [HOTS]
4. Find the range of height of any ten students of your class.  [Life Skills]
IV.  Long Answer Type Questions (4 Marks)
1. Organise the following marks in a class assessment in a tabular form  [Life Skills]
4, 6, 7, 5, 3, 5, 4, 5, 2, 6, 2, 5, 1, 9, 6, 5, 8, 4, 6, 7
(i) Write the highest marks. (ii) What are the lowest marks?
(iii) What is the range of the data? (iv) Find the arithmetic mean.
2. A cricketer scored 280 runs in 12 innings out of which he was not-out 2 times. What is
his average score per inning?  [Multidisciplinary Question]
3. The heights of 10 girls (in cm) are as follows:
151, 147, 127, 136, 150, 138, 145, 147, 155, 150
(i) What is the height of the tallest girl?
(ii) What is the height of the shortest girl?
(iii) What is the range of the data?
(iv) Find the mean height.
4. Following are the goals scored by two hockey teams.
APS 4 0 2 3 3 0 4 3 —
KPS 2 3 4 1 2 0 0 3 6
Which team’s performance is better?
Answers and Hints
I. A. 1. The difference between the highest 3. 25 [Hint: Range = 42 – 17 = 25]
and the lowest observation in a given 4. 3.57
data is called its range. 6 + 2 + 5 + 4 + 3 + 4 +1
2. Mean of the data Hint: Mean =
7
Sum of all observations 25
= = = 3.57
Number of observations 7

Data Handling  51


5. 68 New mean = 15, i.e., 3 more than the old
Hint: The smallest observation = 31 mean.
Largest observation = 99 2. (i) 3.67
Range = 99 – 31 = 68 Hint : Mean = 6 + 2 + 5 + 4 + 3 + 4 + 4 + 2 + 3
6. 450 33 9
= = 3.666 = 3.67
Hint: Sum of the numbers = mean × number 9
of numbers = 45 × 10 = 450 (ii) Number of observations above the mean
B. 1. 18 =5
[Hint: Range = 21 – 3 = 18] (iii) 4 [Hint: Range = 6 – 2 = 4]
2. 5.6
3. x = 3
2 + 3 + 5 + 7 + 11 28
[Hint: = = 5.6 ]
5 5 8+ 4+ x+ 6+ 2+ 7
Hint: A.M. = = 5
3. Head, Tail 6
C. 1. True 2. False ⇒ 27 + x = 5 ⇒ x = 30 − 27 = 3
6
II. 1. No.
4. Range will vary
Hint: Range of 5, 9, 3, 0, 4, 8, –1, –3 is
9 – (–3) = 9 + 3 = 12 IV. 1. Hint
when 6 is added to each value, we get Marks Tally Frequency
11, 15, 9, 6, 10, 14, 5, 3 Marks
Range = 15 – 3 = 12 1 | 1
No, it will remain the same. 2 || 2
2. 14 3 | 1
Hint: First 6 multiples of 4 are 4, 8, 12, 4 ||| 3
16, 20, 24 5 |||| 5
4 + 8 + 12 + 16 + 20 + 24
Mean = 6 |||| 4
6
84 7 || 2
= 14
6 8 | 1
3. 6
9 | 1
Hint: Sum of the observations = 6 + 9
20
+ 6 + 4 + 7 + 2 + 8 = 42
Number of observations = 7 (i) Highest marks = 9
Sum 42 (ii) Lowest marks = 1
Mean = = =6
Number 7 (iii) Range = 9 – 1 = 8
100
III. 1. Hint: Mean Score (iv) Arithmetic mean = =5
20
11 + 13 + 15 + 9 + 12 60 2. 23.33
= = = 12
5 5
New marks = 11 + 3 = 14, 13 + 3 Hint: Average score per inning
= 16, 15 + 3 = 18, 9 + 3 = 12 280 70
= = = 23.33.
and 12 + 3 = 15 12 3
14 + 16 + 18 + 12 + 15 75
New mean = = = 15
5 5

52 Mathematics–7
3. (i) Height of the tallest girl = 155 cm 4. Performance of APS is better:
(ii) Height of the shortest girl = 127 cm Hint: Mean goals scored by team APS
(iii) Range = 155 – 127 = 28 4+0+2+3+3+0+4+3
=
(iv) 144.6 8
19
Hint: Mean height = = 2.37
8
151 + 147 + 127 + 136 + 150 Mean goals scored by team KPS
+ 138 + 145 + 147 + 155 + 150 2+3+4+1+2+0+0+3+6
= =
10 9
1446 21
= = 144.6 cm = = 2.33
10 9

3.2 Other Measures of Central Tendency – Median and Mode


• We have discussed mean as one of the measures of central tendency but it is not the only
form of representative value. There are two others called mode and median. These are used
according to the requirement from the data.
• Mode is the observation that occurs most frequently in the data.
• When the data is arranged in ascending (or descending) order, then the middlemost observation
is the median of the data.
Let the number of observations be n.
th
 n  1
If ‘n’ is odd, Median =   observation
2 
  n  th  n  th 
and if ‘n’ is even, Median =         observation.
2 2 2 
Exercise 3.2
I. Very Short Answer Type Questions (1 Mark)
A. Answer the following.
1. Find the median of first nine even natural numbers.
2. What is the mode of the data 12, 13, 11, 16, 12, 12, 20, 22, 12 and 18?
3. Find the mode of 1, 1, 2, 2, 3, 3, 3, 3, 4, 5, 0.
4. What is mode of the data? 5. What is median of the data?
6. Find the mode from the following table.
Marks 40 45 52 53 55 58 90
No. of the students 6 3 2 7 3 2 1
B. Fill in the blanks.
1. The mode of the data 14, 6, 12, 14, 21, 10, 14, 6 is ............... .
2. The median of the data 40, 50, 99, 68, 98, 60, 94 is ............... .
3. The measure of central tendency which best represents the data of the most popular
politician after a debate is ............... .

Data Handling  53


4. A data consists of 14 observations written in increasing order. The median is the ...............
of (...............)th and ...............)th observations.
5. The median of 4, 2, 5, 3 2, 5, 4, 6, 8, 7, 7, 9 is ............... .
C. Say True or False.
1. The mode of the observations 13, 16, 5, 2, 18, 28, 9, 13, 16, 13 is 18.
2. Median of 5, 6, 10, 3, 7, 9, 8 is 3.
3. The mode is always one of the observations in the data.
II.  Short Answer Type Questions–I (2 Marks)
1. Find the median of first nine odd natural numbers.
2. Find the median of 12, 11, 13, 16, 15, 13, 18, 16, 24, 18, 22 and 20.
3. If the extreme observations on both the ends of a data arranged in ascending order are
removed, then the median gets affected. Justify your answer.
4. Find the median of 1, 2, 3, 5, 6, 7, 8, 9
III.  Short Answer Type Questions–II (3 Marks)
1. Ages (in years) of employees in a company are as follows: 25, 27, 27, 21, 28, 36, 28,
34, 20 and 31, find the median age.
2. Calculate the mean, median and mode of the following data:
5, 10, 10, 12, 13. Are these three equal?
3. Find the mode of the data: 14, 15, 13, 18, 14, 14, 22, 24, 14 and 20.
Also, find the new mode if 12 is added to each observation. [HOTS]
4. Scores of players of a team in a cricket match were as follows: 17, 21, 28, 21, 27, 24,
21, 16, 34, 18 and 20. What is the modal score? If each scored thrice as many runs, what
will be the new modal score?  [HOTS]
IV.  Long Answer Type Questions (4 Marks)
1. Observe the data and answer the questions that follow:
18, 17, 18, 18, 10, 17, 19
(i) Which data value can be put in the data so that the mode remains the same?
(ii) At least how many and which value (s) must be put in to change the mode to 17?
(iii) What is the least number of data values that must be put in to change the mode to
19?
2. Age (in years) of 6 children of two groups are recorded as below:
Age (in years)
Group A Group B
10 10
10 12
12 14
11 15
13 15
14 15
(i) Find the mode and range for each group.
(ii) Find the mode and the range if the two groups are combined together.

54 Mathematics–7
3. Data of height of students arranged in ascending order is as follows:
2, 3, 7, 9, 9, a, b, 17, 17, 17, 18, 18  [HOTS]
If median is 11, then find all pairs of possible values of unknown a and b.
4. Find median of first fifty natural numbers.

Answers and Hints


I. A. 1. First 9 even natural numbers are 16 + 16 32
Median = = = 16
2, 4, 6, 8, 10, 12, 14, 16, 18 2 2
 9 + 1 th 10 th 3. False. The median remains the same.
Median =   term = term 8 8
4. Median = Mean of th term + d + 1 n
 2  2
= 5 term = 10 th term 2 2
2. Mode = 12 = Mean of 4th term + 5th term
Hint: 11, 12, 12, 12, 12, 13, 16, 18, 20, 22 5 + 6 11
= = = 5.5
2 2
Mode = 12
III. 1. 20, 21, 25, 27, 27, 28, 28, 31, 34, 36
3. 3, as it occurs maximum no. of times.
Number of terms = 10
4. The observation that occurs most
frequently in a data is called its mode. 1  10 th  10  th 
Median =  +  + 1 term
5. When the data is arranged in ascending 2 2  2  

or descending order the middle most 1
observation is called the median of the = (5th + 6th term)
2
data.
1 1 55
6. Here maximum frequency is 7 for 53 = ( 27 + 28) = × 55 = = 27.5
2 2 2
Mode = 53 marks.
5 + 10 + 10 + 12 + 13 50
B. 1. 14 2. Median = 68 2. Mean = = = 10
5 5
5+1
3. Mode 4. Mean, 7th, 8th 5. 5 Median = d n th term = 3rd term = 10
2
C. 1. False 2. False
Mode = 10
3. True
Yes all three are equal.
II. 1. First nine odd natural numbers 1, 3, 5,
3. Mode = 14.
7, 9, 11, 13, 15, 17
New mode = 14 + 12 = 26
Median = 9
4. Modal score = 21
2. Arranging in ascending order = 11, 12, New score = 51, 63, 84, 63, 81, 72, 63,
13, 13, 15, 16, 16, 18, 18, 20, 22, 24 48, 102, 54 and 60
Here, n = 12 New modal score = 63
Median is average of c m term and
n th
IV. 1. (i) 18
2
c + 1 m term
n th (ii) At least 2 and value 17

2
(iii) Least number = 3
Median = Mean of 6th and 7th term

Data Handling  55


2. (i) Group A mode = 10; 1 1
= (6th + 7th) = (a + b)
Range = 14 – 10 = 4 2 2
1
Group B mode = 15; ⇒
11 = (a + b)
2
Range = 15 – 10 = 5 ⇒ 22 = a + b
(ii) Mode of the combined group = 10 All possible a and b can take a = 9,
and 15 b = 13; a = 10, b = 12 ; a = 11, b = 11
Range of the combined group 4. No. of observation = 50
= 15 – 10 = 5 n 50
Here, = = 25
3. Number of observation = 12 2 2
1
1  12  12   Median = (25th + 26th observation)
Median =  th +  + 1 th  2
2 2  2   1 51
= (25 + 26) = = 25.5
2 2

3.3 Bar Graph
• Bar graph is a representation of a numerical data by a number of bars of uniform width
drawn horizontally or vertically with equal spacing between them.
• The uniform width of the bars and the uniform space between them is suitably chosen keeping
in view the given data and the space available for the diagram.
• To draw a bar graph we have to first choose a convenient scale.
• At times we need to represent/compare two collections of data at the same time, we draw a
double bar graph.
Example: The following data gives the maximum and minimum temperature (in °C) of the
cities on a particular day. Plot a double graph from this data and answer the follwing questions.

City Delhi Banglore Srinagar Mumbai Chennai


Maximum Temperature (in °C ) 23 20 10 30 32
Minimum Temperature (in °C ) 20 19 4 28 27
(a) Which city has the maximum difference in day’s temperature?
(b) Which city has the minimum difference in day’s temperature?
(c) Which are the hottest and coldest cities?
Solution. (a) Srinagar, (b) Banglore (c) Hottest is Chennai, Coldest is Srinagar
Example: Draw the bar graph for the following data:

Day Monday Tuesday Wednesday Thursday Friday Saturday Sunday


Temperature 35°C 30°C 34°C 33°C 33.5°C 34.5°C 30°C

56 Mathematics–7
Exercise 3.3
I. Very Short Answer Type Questions (1 Mark)
1. When do we need to draw a double bar graph?
2. (i) Frequency of 3 in the given data is ....................... .
Observation 1 2 3 4 5
No. of observations 10 8 7 3 4
(ii) Observation ....................... has 8 as frequency.
II. Short Answer Type Questions–I (2 Marks)
1. Find the mode from the following bar graph.


Data Handling 57
2. Find the median from the following graph.

III. Short Answer Type Questions–II (3 Marks)


1. Draw a frequency table using the following data about the number of animals at a
farmhouse.
13, 10, 11, 10, 15, 11, 11, 15, 15, 15, 10, 10, 10, 10, 12, 13, 14, 12, 10, 15, 14, 12, 13,
11, 11, 12, 12, 15, 14, 13, 13, 14, 15, 11, 12, 14.
2. A die was thrown 20 times and the following outcomes were recorded.
6, 1, 2, 4, 2, 3, 2, 5, 4, 1, 4, 5, 4, 6, 2, 3, 5, 4, 6, 4
prepare a frequency table using the above data and answer the following questions.
(i) How many times does 4 appear?
(ii) What is the frequency of 3?
(iii) Draw a bar graph to represent the data.
3. The following data gives the value (in crores of rupees) of the Indian exports of cotton
textiles for different years.
Year Value of exports of cotton textiles
(in crores of rupees)
1994-1995 300
1995-1996 325
1996-1997 475
1997-1998 450
1998-1999 550
Draw a bar graph to represent the data.
IV.  Long Answer Type Questions (4 Marks)
1. The table below gives the number of tourists visiting 5 hill stations over two consecutive
years. Study the table and answer the questions that follow.

58 Mathematics–7
Hill Station 2008 2009
Nainital 5000 5800
Shimla 6400 5500
Manali 4700 5200
Mussoorie 6800 7200
Kullu 4500 5600
(i) Draw a double bar graph to depict the above information using appropriate scale.
(ii) In which hill stations was there increase in number of tourists in the year of 2009?
(iii) Which hill station was visited by the maximum number of tourists in 2009?
(iv) Which hill station was visited by the least number of tourists in 2008?
 [Multidisciplinary Questions]
2. The data giving the minimum and maximum temperatures in the month of March 2015
from various cities are given in the table below:
Cities Max. Temp Min. Temp.
Delhi 29°C 15°C
Mussoorie 16.6°C 5.1°C
Shimla 14°C 7°C
Chennai 33°C 24°C
Plot a double bar graph and answer the questions below:
(i) Which city has the largest difference in the maximum and minimum temperature on the
given data?
(ii) Which is the hottest city and which is the coldest city?
(iii) Name two cities where maximum temperature was less than the minimum temperature of
the other.
(iv) Name the city which has the least difference between its maximum and minimum
temperature.  [Multidisciplinary Questions]
3. The table below gives the flavours of ice cream liked by children (boys and girls) of a
society.
Flavours Boys Girls
Vanilla 6 10
Chocolate 11 13
Strawberry 5 9
Mango 10 11
Butterscotch 15 12
Study the table and answer the following questions:
(i) Draw a double bar graph using appropriate scale to represent the above information.
(ii) Which flavour is liked by the most of the boys?
(iii) How many girls are there in all?
(iv) Find the ratio of children who like strawberry to vanilla flavour ice cream.
(v) Is eating too many ice creams a day good for health?  [Value Based Questions]

Data Handling  59


Answers and Hints
I. 1. Double bar graph is used to compare 11 |||| | 6
two collections of data.
12 |||| | 6
2. (i) 7 (ii) 2
13 |||| 5
II. 1. Mode = 9
14 |||| 5
Hint: As maximum number of students
is 8 and 8 students scored 9 marks, so 15 |||| || 7
9 is the mode. 2. Hint:
2. Median = 6 Recorded Tally Frequency
Hint: Total number of students = 3 + 5 Outcomes Marks
+ 2 + 5 = 15 = number of observation 1 || 2
15 + 1
and 15 is odd so median is = 2 |||| 4
2
3 || 2
8th term, and 8th term is number 6.
So, median is 6. 4 |||| | 6
III. 1. Hint: 5 ||| 3
6 ||| 3
Number Tally Frequency
of Marks Total 20
animals
(i) Six times (ii) 2
10 |||||| || 7

(iii)

60 Mathematics–7
3.

IV. 1. Hint:
(i)

(ii) There was increase in tourists in 2009 in Nainital, Manali, Mussoorie and Kullu.
(iii) Mussoorie was visited by maximum number of tourists in the year 2009.
(iv) Kullu was visited by the least number of tourists in the year 2008

Data Handling 61
2.

(i) Delhi has the largest difference in the maximum and minimum temperature.
(ii) Hottest city is Chennai and coldest city is Mussoorie.
(iii) Shimla was less than Delhi.
Mussoorie was less than Chennai. (iv) Shimla
3. (i)


(ii) Butterscotch is liked the most by the boys. (iii) 55 girls in all.
Strawberry 14 7
(iv) Ratio of children who like strawberry to who like vanilla = = = = 7 :8
Vanilla 16 8
(v) No

62 Mathematics–7
3.4  Probability
• The situation that may or may not happen, has a chance of happening.
• Tossing of a coin is an experiment. The possible results of an experiment are called events.
When a coin is tossed, a head or a tail may appear. Occurrence of Head / Tail are events.
• The uncertainty associated with happening of an event is called its ‘chance’.
• The measure of chance of happening of an event is called its probability.
number of favourable outcomes
• Probability of an event = .
total number of outcomes
• The outcomes which have equal chances of occurrence are called equally likely outcomes.
On tossing a coin occurrence of head and occurrence of tail are equally likely. However, on
rolling a dice occurrence of 1 and an ‘odd number’ are not equally likely.
• Probability of an event lies between 0 and 1, both included.
• An event which cannot happen is called an impossible event. The probability of an impossible
event is 0.
• An event which is certain to happen is called a sure event. The probability of a sure event
is 1.
Example: A die is thrown 10 times. The number 5 appears 3 times. What is the probability of
getting number 5?
Solution. Total number of times die is thrown = 10
The number of times 5 appears = 3
3
Probability of getting number 5 =
10
Exercise 3.4
I. Very Short Answer Type Questions (1 Mark)
A. Answer the following.
1. What is an event? 2. What is the probability of a certain event?
3. What is the probability of an impossible event?
4. The letters written on paper slips of the word ‘MEDIAN’ are put in a bag. If one slip is
drawn randomly, what is the probability that it bears the letter D?
5. Write all possible outcomes of tossing a fair coin.
6. Write all possible outcomes of throwing a fair dice.
7. When the spinner shown here is rotated, which colour has more chance to show up with
arrow than the others?

8. What is the probability of the Sun rising tomorrow?

Data Handling  63


9. What is the probability that a student chosen at random out of 3 girls and 4 boys is a
girl?
B. Fill in the blanks.
1. The probability of getting a number greater than 3 on throwing a dice is ............... .
2. On tossing a coin the outcomes are ............... .
3. The possible results of an experiment are called ............... .
4. Probability of an event lies between ............... and ............... .
5. A coin is flipped to get a head or tail, the probability of getting a head is ............... .
6. A container has 7 red balls and 9 green balls. A ball is drawn at random. The probability
of drawing a green ball is ............... .
C. Say True or False.
1. Probability of an impossible event is 0.
2. In a school, only 2 out of 5 students can participate in a debate. The chance that a student
picked at random makes it to the competition is 40%.
II. Short Answer Type Questions–I (2 Marks)
1. Following cards are put facing down
A E I O U
       
What is the chance of drawing out
(i) a vowel? (ii) A or I? (iii) a card marked U? (iv) a consonant?
2. A car seller collects the following data of cars sold in his shop.
Colour of Car Number of cars sold
Red 15
Black 20
White 17
Silver 12
Others 9
(i) Which colour of the car is most liked?
(ii) Which measure of central tendency was used in (i)?
3. Mr. Bansal collected the data regarding weights of students in his class and prepared the
following table:
Weight (in kg) 42-45 46-49 50-53 54-57
No. of students 5 7 29 9
A student is to be selected randomly from his class for some competition. In which interval
of the weight is the probability of selection of the student lowest?
4. Classify the following events as certain to happen, impossible to happen, may or may not
happen.
(i) Getting a number greater than 6 on throwing a die.
(ii) Getting a tail when a coin is tossed.
(iii) Republic Day will be on 26 January.
(iv) A team winning a match.

64 Mathematics–7
5. If a die is thrown, the probability of getting a number greater than 6 is 1. Justify your
answer.

6. When a coin is tossed, there are two possible outcomes. Justify your answer.
7. Justify that the probability of getting an ace out of a deck of cards is greater than 1.
1
8. The probability of the spinning arrow hitting the shaded region is . Justify.
3
III. Short Answer Type Questions–II (3 Marks)
1. A fair coin is tossed. Then find the following probabilities:
(i) P(T) (ii) P(H or T) (iii) P(H and T)
2. A fair dice is rolled. Then find the following probabilities:
(i) P (an even number) =
(ii) P (an odd number) =
(iii) P (number more than 3) =
(iv) P (number less than 4) =
(v) P (a multiple of 3) =
(vi) P (obtaining 6) =
IV. Long Answer Type Questions (4 Marks)
1. There are first 10 prime numbers in a box. What is the possibility of drawing an even
number? [HOTS]
2. What is the probability of choosing prime number from the first 10 natural number?
3. What is the probability of picking a month from the year having 31 days?
4. What is the probability of getting a vowel from the word ‘NUMBERS’?

Answers and Hints


I. A. 1. A possible result of an experiment 7. Orange 8. 1
is called an event. 3
9. Hint:
2. Probability of a certain event is 1. 7
Total number of students = 3 + 4
3. Probability of an impossible event is 0.
=7
1 3
4. Probability of choosing a girl =
6 1 7
B. 1. 2. Head and Tail
Hint: Total number of letters = 6 2
3. event 4. 0 and 1
Favourable letter = 1 1 1 9
Probability of drawing D = 5. 6.
6 2 16
5. {H, T} 6. {1, 2, 3, 4, 5, 6}
C. 1. True 2. True

Data Handling  65


5 2 1
II. 1. (i) =1 (ii) (iii) 3 1
5 5 5 = =
6 2
(iv) 0
(ii) P (an odd number) = P(1, 3, 5)
2. (i) Black (ii) Mode 3 1
5 1 = =
3. Lowest = 6 2
50 10 (iii) P(number more than 3)
Weight group is 42 – 45 3 1
= P(4, 5, 6) = =
6 2
4. (i) Impossible
(iv) P(number less than 4)
(ii) May or may not happen
(iii) Certain to happen 3 1
= P(1, 2, 3) = =
(iv) May or may not happen 6 2
(vi) P(number is multiple of 3)
5. False. It is not possible to get a number 2 1
greater than 6 when a die is thrown. = P(3, 6) = =
6 3
1
6. True. {H, T} (vi) P(obtaining 6) = P(6) =
6
7. False. Probability of getting an ace 1
IV. 1.
4 1 10
= = < 1 otherwise also probability 2. Natural numbers 1 to 10 are 1, 2, 3, 4,
52 13
of an event cannot be greater 5, 6, 7, 8, 9, 10
than 1. Prime numbers 2, 3, 5 and 7 = 4
3
8. False. It is . Probability of choosing a prime number
4 4 2
III. 1. Total number of outcomes when a coin = =
10 5
is tossed = {H, T} = 2.
3. Total number of months in year of 12
(i) Number of favourable outcomes = months having 31 days = Jan, Mar, May,
{T} = 1 July, Aug, October, December = 7
1
Probability P(T) = Probability of picking a month with 31
2
2 7
(ii) P(H or T) = = 1 days =
2 12
0 4. Total number of letters = 6
(iii) P(H and T) = = 0
2 Number of vowels = 2
2 1
2. Total outcomes are 1, 2, 3, 4, 5, 6 Probability of picking a vowel = =
6 3
(i) P(an even number) = P(2, 4, 6)
Subject Enrichment Activities
I. Crossword
Complete the following crossword using the given clues.
Across Down
1. Mean median and mode are collectively 4. Difference of the highest and the lowest
known as the measures of .............. . observation of data is .............. .
2. .............. is the sum of observation divided 5. Throwing a dice given six possible
by the number of observations. ..............  .

66 Mathematics–7
3. The most common obseravation of a 6. Value of the middle-most observation
group of data is the .............. . when the data is arranged in ascending
or descending order is called ...............
of the data.

II. Activity
Collect the data of the most favourite sport activity out of the given activities of your class.
Badminton, table tennis, football, swimming, cricket, chess.
1. Draw the bar graph.
2. Which is the least popular game?
3. Find ratio of indoor games and outdoor games.
III. Match the columns:
Column I Column II
(a) Mean of the first seven odd natural numbers is (i) 0
(b) Probability of getting 10 when a number from (ii) 5.5
1 to 9 is selected
(c) Median of first ten natural numbers is (iii) 11
(d) Range of the integers from –5 to 6 is ( iv) 7
(e) Mode of the data 2, 3, 3, 4, 4, 4, 5, 5, 5, 5, 6, 6, (v) 6
6, 6, 6 is

IV. Multiple Choice Questions


1. The number of trees in different parks of a city are 23, 28, 38, 23, 24, 24, 23 and 14. The mode
of this data is
(a) 14 (b) 24 (c) 23 (d) 38

2. The range of the data 41, 26, 37, 38, 32, 28, 24, 33 is
(a) 37 (b) 17 (c) 28 (d) 35

Data Handling  67


3. The median of the data: 3, 4, 5, 6, 7, 3, 4 is
(a) 5 (b) 3 (c) 4 (d) 6

4. The difference between the highest and the lowest observation in a data is its
(a) frequency (b) width (c) range (d) mode

5. Range of the integers written on the board is


(a) 52 (b) 57 (c) 3 (d) 13

6. Moyna earned scores of 97, 73 and 88, respectively in her first three examinations.
If she scored 80 in the fourth examination, then her average score will be
(a) increased by 1 (b) increased by 1.5
(c) decreased by 1 (d) decreased by 1.5
7. Which of the following cannot be the probability of an event?
(a) 7 (b) 1 (c) 1 (d) 0
5 2 4
8. Which of the following events is certain to happen?
(a) Getting a number less than 7 when a dice is thrown.
(b) Tomorrow will be a rainy day.
(c) Getting a head when a coin is tossed.
(d) India winning the next cricket World Cup.
9. Which of the following outcomes are not equally likely?
(a) Getting a tail or a head when a coin is tossed.
(b) Getting a number from 1 to 6 when a dice is thrown.
(c) Drawing a green or a black pen from a bag containing 6 green and 8 black pens.
(d) Getting an even number or an odd number when a dice is thrown.
10. On tossing a coin, the outcome is
(a) only head (b) only tail (c) neither head nor tail (d) either head or tail

Answers
4. (c) range 5. (b) 57
6. (d) decreased by 1.5
7
I. 7. (a)
5
8. (a) Getting a number less than 7 when
dice is thrown
9. (c) Drawing a green or a black pen
from a bag containing 6 green
III. (a)–(iv), (b)–(i), (c)–(ii), ( d)–(iii),
and 8 black pens.
(e)–(v)
1 0. (d) either head or tail.
IV. 1. (c) 23 2. (b) 17 3. (c)
qqq

68 Mathematics–7
4 Simple Equations
Topics Covered
4.1 Equation 4.2  Solving an Equation
4.3 More Equations 4.4  Application of Simple Equations to Solve Practical Problems

Let’s Remember
• In algebra, letters are used to represent the unknown and are called variables, e.g., x, y, a, b, etc.
• Variables as word suggest vary and its value is not fixed.
• Algebraic expressions can be formed using numbers, variables and four mathematical operations
of +, –, ×, ÷.

4.1 Equation
• A statement of equality of two algebraic expression is called an equation.
• Linear equation in one variable is an equation which has one variable and its power is 1.
• An equation remains unchanged if the LHS and RHS are interchanged.
• The value(s) of the variables in an equation which satisfy/satisfies the equation, i.e., makes left
side and right side equal is/are called the solution(s) or root(s) of the equation.

Exercise 4.1
I. Very Short Answer Type Questions (1 Mark)
A. Answer the following.
1. What is meant by the word variable?
2. Does the equation remain same when the LHS and the RHS are interchanged?
3. What is meant by solution of an equation?
4. Is x = –17 a solution of the equation –24 = x – 7?
5. Write 8p – 4 = 44 in statement form.
B. Fill in the blanks.
1. x – 1 = 0 is a ................ equation.
5
2. 2x – y = is a linear equation in ................ variables.
2
3. p + 4 = 12 is a linear equation in ................ variable(s).
C. Say True or False.
1. y2 = 36 is a linear equation 2. 3y = 21 is a linear equation.
3. p + q = 12 is a linear equation in one variable.

69
II.  Short Answer Type Questions–I (2 Marks)
Rewrite the following in the statement form:
1. 3p – 2 = 7 2. 3x + 5 = 20
2
3. z + 8 = 23 4. x –1=1
3
III. Short Answer Type Questions–II (3 Marks)
Rewrite the following in the form of equations:
1. Difference of z and 9 is 15.
2. The sum of twice of a and (–5) is 16.
3. The number x multiplied by itself is 8 more than 15.
4. Three fourth of q when added to 7 becomes 15.

Answers and Hints


I. A. 1. Variable means it varies and its value 5. 4 subtracted from 8 times a number is
is not fixed. 44.
2. Yes B. 1. Linear 2. Two 3. One
3. The value(s) of the variables in an C. 1. False 2. True 3. False
equation which satisfy/satisfies the II. 1. The difference of 3p and 2 is 7.
equation making the left hand side and 2. 5 added to 3x gives 20.
the right hand side equal is/are called 3. Sum of z and 8 is 23.
solution(s) or root(s) of the equation. 2
4. One less than x is 1.
4. Yes. 3
Hint: R.H.S. = x – 7 = – 17 – 7 III. 1. z – 9 = 15 2. 2a + (–5) = 16
3
= – 24 = L.H.S. 3. x2 – 15 = 8 4. q + 7 = 15
4
4.2  Solving an Equation
• If we add the same number to both the sides of the equation, the equation remains unchanged.
• If we subtract the same number from both the sides of the equation, the equation remains
unchanged.
• If we multiply or divide both sides of the equation by the same number (other than zero),
the equation remains the same.
Example 1: Write the steps you will use to separate the variable x + 8 = 19.
Solution. Subtract 8 from both sides
i.e., x + 8 – 8 = 19 – 8
⇒ x = 19 – 8 = 11
Example 2: Solve the equation 2m + 6 = 12
Solution. Subtract 6 from both the sides
2m + 6 – 6 = 12 – 6
2m 6
⇒ 2m = 6 ⇒ = (Divide both sides by 2)
2 2
⇒ m = 3

70 Mathematics–7
Exercise 4.2
I. Very Short Answer Type Questions (1 Mark)
A. Answer the following.
1. x + 9 = – 9 2. x – 5 = –3 3. 4 – x = –3
x
4. 2x = 18 5. –6x = 9 6. =5
7
–4x 5x 3 x
7. = 11 8. = –5 9. = –9
9 7 5
B. Say True or False.
1. When 2x – 7 = 5, then x = 6.
2. If the same quantity is added to both sides of an equation, then the equality does not
change.
3. The value of the variable that we get on solving an equation is degree of the equation.
x
4. If (–3x) is taken from LHS to RHS, then it becomes - .
3
5. If both sides of an equation are divided by the same (non-zero) quantity, the equality
changes.
II.  Short Answer Type Questions–I (2 Marks)
Solve for x:
1. 7x + 2 = 23 2. 3x – 4 = 16 3. 8x – 4 = 2x + 30
4. 2x – 3 = 5 5. 2x + 4 = 3x – 4 6. 7x – 6 = 8
III.  Short Answer Type Questions–II (3 Marks)
2y
1. Solve the equation = 18 and verify the result.
3
2. Solve the following equation and verify the answer.
5x + 7 = 22
3. Solve and verify: 8x – 1 = 23

Answers and Hints


I. A. 1. x + 9 = –9 ⇒ x = –9 – 9 = –18 5x
8. = −5 ⇒ 5x = –5 × (–7) = 35
2. x – 5 = –3 ⇒ x = –3 + 5 =2 −7
35
3. 4 – x = –3 ⇒ 4 = –3 + x ⇒ x = = 7
5
⇒ x = 4 + 3 = 7 −3 x
9. = −9 ⇒ –3x = –9 × (–5) = 45
4. 2x = 18 ⇒ x = 18 ÷ 2 ⇒ x=9 −5
5. –6x = 9 ⇒ x = 9 = −3 ⇒ x =
+45
= −15
x −6 2 −3
6. = 5 ⇒ x = 5 × (–7) = –35 B. 1. True 2. True 3. False
−7
−4 x 4. False 5. False
7. = 11 ⇒ –4x = 11 × 9

9
99 II. 1. 7x + 2 = 23 ⇒ 7x = 23 – 2 = 21
⇒ x = −
4 ⇒ 7x = 21 ⇒ ⇒ x = 3

Simple Equations 71
2. 3x – 4 = 16 ⇒ 3x = 16 + 4 2
LHS = × 27 = 2 × 9 = 18 = RHS
⇒ 3x = 20 ⇒ 3
\ For y = 27 LHS = RHS
3. 8x – 4 = 2x + 30
⇒ 8x – 2x = 30 + 4 2. 5x + 7 = 22
⇒ 6x = 34 ⇒ ⇒ 5x + 7 – 7 = 22 – 7
⇒ ⇒ 5x = 15
5x 1
4. 2x – 3 = 5 ⇒ 2x = 5 + 3 ⇒ = 15 #
s 5 1 5
⇒ 2x = 8 ⇒ ⇒ x = 4 ⇒ 5 # x = 3
5
5. 2x + 4 = 3x – 4 ⇒ x = 3
⇒ 4 + 4 = 3x – 2x ⇒ x=8 Verification. Putting x = 3 in LHS
6. 7x – 6 = 8 ⇒ 7x = 8 + 6 LHS = 5 × 3 + 7
⇒ 7x = 14 ⇒ = 15 + 7 = 22 = RHS
⇒ x = 2 Hence, for x = 3 LHS = RHS
2y 3. 8x – 1 = 23
III. 1. = 18
3 ⇒ 8x – 1 + 1 = 23 + 1
Multiplying both sides of the equation ⇒ 8x = 24
3 2y 3 3 1 1
by , we get # = 18 × ⇒ 8x × = 24 ×
2 3 2 2 8 8
2y 3 3 1
⇒ # = 18 9 # ⇒ d8 # n x = 3
3 2 2 8
⇒ y = 9 × 3 = 27 ⇒ x = 4
\ y = 27 is the solution of the given Verification. Putting x = 3 in LHS
equation. LHS = 8x – 1 = 8 × 3 – 1
Verification. Putting y = 27 in the given = 24 – 1 = 23 = RHS
equation. Hence, for x = 3 LHS = RHS
4.3  More Equation
• In this section, we will practice solving some more equations. While solving these equations,
we shall learn about transposing a number, i.e., moving it from one side to the other. We
can transpose a number instead of adding or subtracting it from both sides of the equation.
• Transposing a term means moving the term to the other side of the equation.
• When a term is transposed, its sign gets reversed (+ changes to –, – to + and × to ÷ and ÷
changes to ×.)
Example: Solve 12p – 11 = 25
Solution. Adding 11 to both sides
12p – 11 + 11 = 25 + 11 or 12p = 36
Dividing both sides by 12
12p 36
= or p = 3
12 12
Check. Putting p = 3 in the LHS of equation

72 Mathematics–7
LHS = 12 × 3 – 11 = 36 – 11 = 25 = RHS
Note: Adding 11 to both sides is the same as changing side of (–11).
36
12p = 25 + 11 = 36 ⇒ p =
12p – 11 = 25, ⇒ =3
12
Changing side is called transposing while transposing a number, we change its sign.
Example: Solve 4 (n + 3) = 16
Solution. Divide both sides by 4
16
n+3= or n + 3 = 4
4
Transposing 3 to RHS
n = 4 – 3 or n = 1
Check: 4 (1 + 3) = 4 × 4 = 16 = RHS
From Solution to Equation
We have solved equations we know the successful steps that we take to solve an equation. Now
lets follow the reverse path.
Normal path Equation → Solution
Reverse path Solution → Equation
Let’s start with m = 7
Multiply both sides by 9
m × 9 = 7 × 9 ⇒ 9m = 63
Add 3 to both sides
9m + 3 = 63 + 3
OR lets start with m = 7
Multiply both sides by 6
6m = 42
Subtract 11 from both sides
6m – 11 = 42 – 11 = 31
With m = 7 we have made two different equations.

Exercise 4.3
I. Very Short Answer Type Questions (1 Mark)
A. Answer the following.
1 3
1. Solve x – 1 = 1 2. Solve x = 6
2 4
B. Fill in the blanks.
4y
1. Solution of = –12 is ..................... .
3
2. Solution of 2x – 3 = 5 is ..................... .
1
3. The solution of (2 – 1) = 3 is ..................... .
3

Simple Equations  73


3m – 5 2m – 5
4. The solution of = is ..................... .
2 3
C. Say True or False.
2x
1. x = 24 is a solution of the equation = 16 .
3
II.  Short Answer Type Questions–I (2 Marks)
1. Solve 5 |x| = 15.
F  32
2. C = where F stands for temperature in Fahrenheit degrees and C for Celsius
1.8
degrees.  [HOTS]
Is there any temperature at which the number of Celsius degrees is the same as the number
of Fahrenheit degrees? If so, find it. If not, explain why not.  [HOTS]
IIII.  Short Answer Type Questions–II (3 Marks)
Solve:
 2x 1   5 x
1. 6   
 10 2. 4    8
 3 2   2 3
IV.  Long Answer Type Questions (4 Marks)
16
1. Solve for x: 10 x - = 8 and check your result.
3
y 2y 1 5 1
2. Solve: + 2= – 3. Solve: (x – 4) = (2x + 8)
4 3 2 2 4
1 1
4. (3x + 5) + (4x – 5) = 9
5 3
Answers and Hints
1 F − 32
I. A. 1. x – 1 = 2 or x = 4 2. C =
2 1.8
F − 32
24 When C = F ⇒ F =
2. 3x = 24 or x = =8 1.8
3 ⇒ 1.8 F = F – 32 ⇒ 1.8F – F = –32
B. 1. y = –9 2. x = 4 −32
⇒ 0.8 F = –32 ⇒ F = 8 × 10 = −40
3. 5 4. 1
Yes at –40°F.
C. 1. True, Putting x = 24 in
 2x 1 
24 8 III. 1. −6  − = −10
LHS = 2 × = 16  3 2 
3 2 x 1 −10 −5
⇒ − = =
3 2 6 3
 x for x ≥ 0
II. 1. 5|x| = 15 as x =  2 x −5 1 −10 + 3
 − x for x < 0 ⇒ = + =
3 3 2 6
When x ≥ 0 2 x −7
5x = 15 ⇒ x = 3 ⇒ =
3 6
When x < 0 −7 3 7
⇒ x = × =−
–5x = 15 ⇒ x = –3 6 2 4

74 Mathematics–7
5 x 5 x 3y – 8y –1 – 4
2. −4  −  = −8 ⇒ − = 2 or =
 2 3 2 3 12 2
5 x 5−4 x –5y –5
⇒ − 2 = ⇒ = or = or  y = 6.
2 3 2 3 12 2
1 x 3 5 1
⇒ = ⇒ x = 3. (x – 4) = (2x + 8)
2 3 2 2 4
16 1
IV. 1. 10 x − = 8 5(x – 4) = (2x + 8)
3 2
16 24 + 16 40
⇒ 10 x = 8 + = = or 10 (x – 4) = 2x + 8
3 3 3
40 1 4 or 10x – 40 = 2x + 8
⇒ x = × ⇒ x = or 10x – 2x = 40 + 8
3 10 3
16 or 8x = 48 or x = 6
Check LHS. = 10 x −
3 3 (3x + 5) + 5 (4x – 5)
4 4. =9
Putting x = we get 15
3
4 16 40 16 ⇒ 9x + 15 + 20x – 25 = 135
= 10 × − = − ⇒ 29x – 10 = 135
3 3 3 3
40 – 16 24 ⇒ 29x = 145
= = = 8 = RHS 145
3 3 ⇒ x =
y 2y –1 29
2. – = –2
4 3 2 ⇒ x = 5
4.4 Application of Simple Equations to Practical Problems
We have already learnt to convert statements into simple equations and have also learnt to solve
simple equations. The method is first to form equations corresponding to each situation and
then solve the equations.
Example: Set up an equation and solve it to find the unknown number. Adding 7 to eight times
a number, you get 63.
Solution. Let us take the number be x
8 times of x = 8x
Adding 7 to eight times = 8x + 7
We get, 8x + 7 = 63
⇒ 8x = 63 – 7
⇒ 8x = 56
56
⇒ x = =7
8
x = 7

Exercise 4.4
I. Very Short Answer Type Questions (1 Mark)
A. Answer the following.
1. One-fourth a number is 8. Form an equation.
2. If 5 is added to a number, the result is 15. Form an equation.

Simple Equations  75


3. The equation whose solution is not x = 2 (i) 3x = 5 (ii) x – 6 = – 4
4. If 2 is subtracted from thrice a number, the result is 7. Form an equation.
B. Fill in the blanks.
1. 5 added to thrice a number is 11, the number is ..................... .
2. One-fourth a number is 8, the number is ..................... .
C. Say True or False.
1. The solution of the equation 2x – 3 = 5 is 8.
2. The difference of x and 3 is 9, the number is 12.
II.  Short Answer Type Questions–I (2 Marks)
Solve:
1. 8 added to 9x is 80.
2. 7 times a number added to 15 is 64.
3. 6 added to 11 times a number is 72.
4. Sum of two consecutive natural numbers is 55. Find the numbers.
III.  Short Answer Type Questions–II (3 Marks)
1. Find the number, such that half of the number is 5 less than 8.
2. Find a number, such that, sum of the three times the number and 9 is 21.
3. A man is 7 times as old as his son. If he is 56 years, find the age of his son.
4. Find the numbers such that one is 8 times the other and their sum is 81.
IV.  Long Answer Type Questions (4 Marks)
1. If two adjacent sides of a square are represented as 4p – 7 and 3p + 5, find the value of
p.
2. The total cost of 6 tables and 4 chairs is `1490. If one table costs `40 more than one
chair, find the price of each piece of furniture.
3. The numerator of a fraction is 6 less than the denominator. If 3 is added to the numerator,
2
the fraction is equal to . What is the original fraction?
3
4. A purse contains 25-paise coins and 50-paise coins. The number of 25-paise coins is
three times the number of 50-paise coins. If the total money in the purse is `100, find
the number of coins of each type.
5. In a family the consumption of wheat to that of rice is in the ratio of 3 : 1. If the total
consumption of the cereal is 50 kg, find the consumption of wheat and rice.
6. Suppose you spend `60 to rent bowling shoes at the Bowling Alley. Each game costs
`70. How many games did you bowl if you have spent `340? 
[Multidisciplinary Questions]
7. The temperature is 20°C. It is expected to rise at a rate of 2°C each hour for the next
several hours. In how many hours will the temperature be 32°C? 
[Multidisciplinary Questions]
8. People of Lajpat Nagar RWA started a plantation drive in their area. Some of the trees
were fruit trees. The number of non-fruit trees were three more than three times the number

76 Mathematics–7
of fruit trees. What was the number of fruit trees planted if the number of non-fruit trees
planted is 144? Write three benefits of trees.  [VBQs]
9. Rama Public School organised an inter-class cricket match. Sachin of class 7A, scored twice
as many runs as Rahul of 7B. Togeter their runs fell two short of a double century. How
many runs did each one score? What are the benefits of such inter-class matches? [VBQs]
10. A shopkeeper sells apples in two types of boxes, one small and one large. The large box
has 125 apples. Also the large box contains as many apples as 12 small boxes plus 5
loose apples. Set up an equation which gives the number of apples in each small box.
Also find number of apples in each small box.  [Life Skills]
11. Construct 2 equations starting with x = 2.

Answers and Hints


1  Sum = 81
I. A. 1. x = 8 2. x + 5 = 15
4 8x + x = 81 or 9x = 81
3. (i) 3x = 5 4. 3x – 2 = 7 81
or x = = 9 or x = 9
B. 1. 2 2. 32 9
C. 1. False 2. True \ One number = x = 9 and the
II. 1. 9x + 8 = 80 ⇒ 9x = 80 – 8 Other number = 8x = 8 × 9 = 72.
IV. 1. We know that the adjacent sides of a
⇒ 9x = 72 square are equal
72 ⇒ 4p – 7 = 3p + 5
⇒ x =
= 8 ⇒ x = 8
9
2. 7x + 15 = 64 ⇒ 7x = 64 – 15 ⇒ 4p – 3p = 5 + 7 ⇒ p = 12
49 2. Let the cost of 1 chair be = `x
⇒ 7x = 49 ⇒ x = =7
So, the cost of 1 table = `(x + 40)
7
⇒ x = 7 Cost of 6 tables = `6(x + 40)
3. 11x + 6 = 72 ⇒ 11x = 72 – 6 Cost of 4 chairs = `4x
66
⇒ x = =6 ⇒ x = 6 Total cost = `4x + `6(x + 40)
11
4. x + (x + 1) = 55 ⇒ 2x = 54 = `1490
⇒ x = 27 ⇒ 4x + 6(x + 40) = 1490
1 1
III. 1. x = 8 − 5 ⇒ x = 3 ⇒ x = 6 ⇒ 4x + 6x + 240 = 1490
2 2
⇒ 10x = 1490 – 240
2. 3x + 9 = 21 ⇒ 3x = 21 – 9 1250
12 ⇒ 10x = 1250 ⇒ x = = 125
⇒ 3x = 12 ⇒ x = ⇒ x = 4 10
3
3. Let son’s age be x years and \ Cost of 1 chair = `x = `125
man’s age = 7x years. \ Cost of 1 table = `(125 + 40) = `165
7x = 56 given 3. Let the numerator of the fraction be x
56 So, the denominator of the fraction = x + 6
⇒ x = = 8 years Fraction =
x
7
\ Son’s age = 8 years. x+6
4. Let one number be x and other number According to the question
is 8x. x+3 2
= ⇒ 3(x + 3) = 2(x + 6)
x+6 3

Simple Equations  77


⇒ 3x + 9 = 2x + 12 7. Let after x hours the temperature is
⇒ 3x – 2x = 12 – 9 32°C.
⇒ x=3 ⇒ 20°C + 2°Cx = 32°C
So, the original fraction ⇒ 2°Cx = 32°C – 20°C
x 3 3 1 12° C
= = = = ⇒ 2°Cx = 12°C ⇒ x = =6
x + 6 3+ 6 9 3 2° C
⇒ So in 6 hours the temperature will be
4. Let the number of 50p coins be x
32°C.
So, the number of 25p coins = 3x
8. Let th e number of fruit trees be x
M oney in the form of 50p coins
50 1 So, let the number of non-fruit trees be
= 50 xp = ` x =` x 3x + 3
100 2
3x + 3 = 144 ⇒ 3x = 144 – 3
M oney in the form of 25p coins 141
= 25 × 3xp = 75 xp ⇒ 3x = 141 ⇒ x = = 47
3
75 3
=`x= ` x \ Number of fruit trees = 47.
100 4
Benefits of trees: (i) Give cleaner air
Total money in the purse = `100 by reducing pollution; (ii) Give fruits
1 3 2 x + 3x and vegetables; (iii) Shade in summer;

⇒ x + x = 100 ⇒ = 100
2 4 4 (iv) Stop soil erosion.
5 100 9. Let Rahul scored x runs

⇒ x = 100 ⇒ x = × 4 = 80
4 5 Sachin scored 2x runs
So, number of 50p coins = 80 According to question
Number of 25p coins = 3x = 3 × 80 = 240 x + 2x = 200 – 2 ⇒ 3x = 198
5. Let the ratio constant be x 198
⇒ x = = 66 ⇒ x = 66
Consumption of wheat = 3x kg 3
Consumption of rice = x kg \ Rahul scored 66 runs and Sachin scored
132 runs.
Total consumption of the cereal
Benefits of interclass matches: (i) Develop
= x + 3x = 4x = 50
sportsmanship; (ii) Builds up attitude for
50 25
⇒ x = = kg team work.
4 2 25
Consumption of rice x = kg = 12.5 kg 10. Let number of apples in a small box
2 be x
Consumption of wheat = 3x = 3 × 12.5 So, the number of apples in 12 small box
= 37.5 kg be 12x
6. Rent of bowling shoes = `60 N umber of apples in 1 large box
= 125
Cost of 1 game = `70
According to question,
Let number of games be = x
⇒ Total amount spent = `70x + 60 = `340 12x + 5 = 125 ⇒ 12x = 125 – 5
⇒ 70x = 340 – 60 ⇒ 12x = 120 ⇒ x = 10
\ N u m b e r o f a p p l e s i n a s m a l l
⇒ 70x = 280
box = 10.
280
⇒ x = ⇒ x =4
11. x = 2, 3x = 6, 3x – 7 = –1; 5x = 10;
70

\ I could bowl 4 games. 5x – 2 = 10 – 2 = 8 or 5x – 2 = 8

78 Mathematics–7
Subject Enrichment Activities
I. Crossword
Complete the following crossword puzzle using the given clues.

Across Down
1. .................is a statement of equality of 4. A branch of mathematics which uses
two algebraic expressions. letters to represent unknown is called
................ .
2. Moving a terms to other side is called 5. Equations in one variable is known as
................ . ................ .
3. Letter which are used to represent 6. The value of the variable which satisfies
unknown in algebra. the equation is called ................ of the
equation.

II. Multiple Choice Questions


Choose the correct option
1. 3x + 20 = 5
(a) –5 (b) 5 (c) 8 (d) –8

2. Twice a number added to 8 gives 10


(a) 9 (b) –9 (c) 1 (d) –1

3. Three-fourth of a number minus 4 gives –10.


(a) 8 (b) –8 (c) 6 (d) 9

4. If 2x + 9 = 3459, then n = ?
(a) –1725 (b) 1752 (c) –1752 (d) 1725

Simple Equations  79


3
5.
x + 8 = 68
5
(a) 100 (b) –100 (c) 36 (d) –36

6. My mother's age is 8 years more than thrice of my age. If she is 41 years old, find my age.
(a) 13 years (b) 11 years (c) 14 years (d) 17 years

7. One-third of a number when subtracted from three times the number gives 616, then the number is
(a) 331 (b) –331 (c) 231 (d) –231

8. If 5a + 20 = –60, then value of a is


(a) –16 (b) –18 (c) 16 (d) 18
3a
9. What value of a satisfies + 6 = 51?
5
(a) 65 (b) 55 (c) 75 (d) 45
3a
10. If + 8 = 17, then a is ............... .
19
(a) 47 (b) 57 (c) 67 (d) 77

Answers
I. II. 1. (a) –5 2. (c) 1 3. (b) –8
4 . (d) 1725 5. (a) 100
6. (b) 11 years 7. (c) 231 8. (b) 16
9. (c) 75 10. (b) 57

qqq

80 Mathematics–7
5 Lines and Angles
Topics Covered
5.1 Related Angles 5.2  Intersecting Lines

Let’s Remember
• A point represents a dot with no length, breadth and thickness.
• Line is a concept of straight path which can be extended in both the directions.
• A ray has a fixed point and extends indefinitely in only one direction.
• Plane is a flat surface which extends indefinitely in all the directions.
• Two lines in a plane either intersect or are parallel.
• Two intersecting lines have only one point in common.
• Two lines in a plane are said to be parallel if they do not intersect or they are at equal
distance apart  everywhere.
• Two rays with a common initial point are said to form an angle. The common initial point
is called the vertex and the rays form arms of the angle.

5.1 Related Angles
• Two angles formed by two intersecting lines having no common arm are said to be vertically
opposite angles. In the following figure 1 and 2 are vertically opposite angles and
3  and  4 are also vertically opposite angles.

Vertically opposite angles are equal, i.e., 1 = 2, 3 = 4.


• Two angles whose sum is 90° are called complementary angles.


1 + 2 = 30° + 60° = 90°   \ 1 and 2 are complementary angles.
• Two angles whose sum is 180° are called supplementary angles.

81
1 + 2 = 60° + 120° = 180°   \ 1 and 2 are supplementary.
• Two angles with a common vertex, a common arm and the non-common arms lying on the
opposite sides of the common arm form a pair of adjacent angles.
In the following figure, a and b are adjacent angles.

• A pair of adjacent angles with non-common arms as opposite rays are said to form a linear
pair.

a + b = 180°    \ a and b are forming a linear pair.


• Two angles forming a linear pair are adjacent and supplementary.

Exercise 5.1
I. Very Short Answer Type Questions (1 Mark)
A. Answer the following.
1. Find the complement of 63°.
2. Which of the following pairs of angles are complementary: (i) 60°, 30° (ii) 60°, 120°?
3. Which angle is equal to its complement?
4. Which angle is equal to its supplement?
5. If one angle of a linear pair is a right angle, what kind of angle is the other?
6. Find the complement of an angle of measure 90° – x.
7. In the figure below, is RPB adjacent to 50° angle?

8. Are the angles x – 20° and 170° – x supplementary? Why?


9. What is the measure of an angle that makes a linear pair with an angle of 71°?
10. What is the measure of an angle which is four times its supplement?

82 Mathematics–7
11. In the following figure, is 1 adjacent to 2? Give the reason.

12. Are a = 67° and b = 23° supplementary?


13. What are complementary angles?
14. If one angle in a pair of supplementary angles is decreased, what will happen to the other
angle?
15. If one angle in a pair of complementary angles is increased, what will happen to the other
angle?
16. Write the necessary conditions for a pair of angles to be adjacent.
17. When is a pair of angles said to form a linear pair?
18. What are vertically opposite angles?
B. Fill in the blanks.
1. Measure of the complement of an angle of 73° is ............... .
2. Measure of the supplement of an angle of 30° is ............... .
3. Measure of an angle which is equal to its complement is ............... .
4. If one angle of a linear pair is right angle, then the other is ............... .
5. In a pair of supplementary angles if one angle is obtuse then the other angle is ............... .
6. In the figure, COE is a right angle.

(i) COB and DOC are forming a ........ .


(ii) 3 = ........... .
(iii) Name a pair of complementary angles ........... .
(iv) Name a pair of vertically opposite angles ........... .
(v) Name a pair of supplementary angles ........... .
7. Angles of a linear pair are ........... as well as ........... .
8. Adjacent angles have a common vertex, a common ........... and non-common arms on
........... of common arm.
C. Say True or False.
1. Sum of two complementary angles is 180°.
2. Sum of two supplementary angles is 180°.
3. Two acute angles can be complements of each other.
4. A pair of right angles are supplementary.

Lines and Angles  83


5. Vertically opposite angles are equal.
6. In a pair of adjacent angles uncommon arms are on opposite sides of the common arm.
7. If two parallel lines are cut by a transversal then each pair of alternate interior angles is
complementary.
8. Angles of a linear pair are adjacent as well as supplementary.
9. In a pair of complementary angles if one angle is more than 45° then the other will be
less than 45°.
II. Short Answer Type Questions–I (2 Marks)
1. Question 1 to 5: Find x in each of the following figures. Give reasons also.

1. 2.

3. 1 = 48° 4.

5.

Justify whether True or False.


6. Adjacent angles are always complementary.
7. Two straight angles are supplementary.
8. In a linear pair both angles can be obtuse.
9. In a linear pair, a pair of angles has to be adjacent.
10. Vertically opposite angles have the same measure.
11. Adjacent angles are equal in measure.
III.  Short Answer Type Questions–II (3 Marks)
1. In the figure, POR is a straight line. Find p.

84 Mathematics–7
2. In the figure, XOY is a straight line. Find y.

3. The difference of two complementary angles is 30°. Find the angles.


4. Find the value of a, in the figure below.

5. In the figure, POR = 90°. If y : z = 2 : 3, then find x.

6. In the figure, OC  OA.

(i) Name all the pairs of adjacent angles.


(ii) Name the pair of complementary angles.

IV. Long Answer Type Questions (4 Marks)


1. Measures of two supplementary angles are two consecutive odd integers. Find the angles.
 [HOTS]
2. The difference of two interior angles on the same side of the transversal intersecting two
parallel lines is 30°. Find the angles.  [HOTS]
3. In the figure, A, B and C are collinear points.

Lines and Angles  85


Name
(i) A pair of complementary angles.
(ii) Two pairs of supplementary angles.
(iii) Three pairs of adjacent angles.

Answers and Hints


I. A. 1. Complement of 63° = 90° – 63° 16. They should have a common vertex, a
= 27° common arm and uncommon arms on
2. (i) 60° + 30° = 90° the opposite side of the common arm.
⇒ 60°, 30° are complementary 17. A pair of adjacent angles is said to form
(ii) 60° + 120° = 180° a linear pair if the two uncommon arms
⇒ 60°, 120° are not complementary are two opposite rays.
3. 45° 4. 90° 5. right 18. When two lines intersect, four angles
are formed. Two angles with a common
6. Complement of 90° – x
vertex and no common arm are called
= 90° – (90° – x) = 90° – 90° + x° = x°. vertically opposite angles.
7. No, ∠RPB is not adjacent to 50° angle. B. 1. Complement of 73° = 90° – 73° = 17°
8. No, as x – 20° + 170° – x = 150° ≠ 2. Supplement of 30° = 180° – 30° = 150°
180° 3. 45° 4. right angle
9. Measure of the angle that makes a linear 5. acute
pair with 71° = 180° – 71° 6. (i) linear pair
= 109° (ii) 90° (iii) ∠1 and ∠2
10. Let the angle be x (iv) ∠4 and ∠1
Its supplement = 180° – x (v) ∠4 and ∠5
x = 4(180° – x) = 720° – 4x 7. adjacent, supplementary
⇒ x + 4x = 720° 8. arm, opposite side
720°
⇒ 5x = 720° ⇒ x = = 144° C. 1. False 2. True 3. True
5
4. True 5. True 6. True
11. No, as they don’t have a common vertex.
7. False 8. True 9. True
12. No; ∠a + ∠b = 67° + 23° = 90°
≠ 180°. Hence, a and b are complementary II. 1. x + 110° = 180° (L.P.A)
and not supplementary. ⇒ x = 180° – 110° = 70° ⇒ x = 70°
13. Two angles whose sum is 90° are called 2. x + 3x + x = 180°
complementary angles. (Angles on a straight line)
14. Other angle will increase. ⇒ 5x = 180 ⇒ x = 36°
15. Other angle will decrease.
3. ∠1 = 48° ⇒ x = 48°

86 Mathematics–7
4. x + 120° + 40° + 75° = 360° 60°
(Angles at a point) ⇒
x = = 30°
2
⇒ x + 235° = 360° Angles are 30°, 30° + 30° = 60°
⇒ x = 360° – 235° = 125° 4. ∠1 = 90° vertically opposite angles
5. x + 35° = 90° (Complementary angles) 40° + 90° + 5a = 180°
⇒ x = 90° – 35° = 55° ⇒ 130° + 5a = 180°
6. False. a and b are adjacent but not ⇒ 5a = 180° – 130°
complementary. 50°
⇒ 5a = 50° ⇒ a = = 10°
5
5. ∠POR + ∠QOR = 180° (Linear pair)
90° + z + y = 180°
⇒ z + y = 90°
7. False. y : z = 2 : 3 ⇒ y = 2a, z = 3a
⇒ y + z = 2a + 3a = 90°
∠AOB and ∠COD are not supplementary. ⇒ 5a = 90° ⇒ a = 18°
8. False, sum of linear pair must not y = 2 × 18° = 36°
exceed 180°. Sum of two obtuse angles ⇒ z = 3a = 3 × 18° = 54°
is greater than 180°. x + y = 180° (linear pair)
9. True, as a linear pair is a pair of adjacent ⇒ x + 36° = 180°
angles and whose non-common arms ⇒ x = 180° – 36° = 144°
are in a straight line. 6. (i) Pairs of adjacent angles c, b are
10. True, as vertically opposite angles are adjacent.
equal. Pairs of adjacent angles b, a are adjacent.
(ii) b, a are complementary.
IV. 1. Let the angles be x and x + 2
x + (x + 2) = 180°
⇒ 2x = 180° – 2 ⇒ 2x = 178°
11. False,
⇒ x = 89°
e.g., a and b are adjacent but their
So angles are 89° and 89° + 2° = 91°
measure is not equal.
2. Let the angles be x and x + 30°
III. 1. 2p + 5 + 3p – 25° = 180° (Linear pair)
⇒ 5p – 20° = 180° ⇒ x + (x + 30°) = 180° (interior angles
on the same side of transversal are
⇒ 5p = 200° ⇒ p = 40°
supplementry)
2. 50° + 2y° + 3y° = 180°
⇒ 2x = 180° – 30°
(angles at a point on a straight line)
⇒ 50° + 5y° = 180° ⇒ 2x = 150° ⇒ x = 75°
⇒ 5y° = 180° – 50° = 130° Angles are 75° and 105°. (i.e., 75° + 30°)
130° 3. (i) ∠CBD and ∠DBE are complementary.
⇒ y = = 26°
5 (ii) ∠CBE and ∠ABE are supplementary.
3. Let one angle be x and its complement or ∠CBD and ∠ABD are supplementary.
be x + 30°
(iii) ∠CBD and ∠DBE, ∠CBE and ∠ABE,
Now, x + x + 30° = 90°
–ABD and –CBD are three pairs of adjacent
⇒ 2x = 90° – 30° ⇒ 2x = 60° angles.

Lines and Angles  87


5.2 Intersecting Lines
• Two lines l and m in a plane intersect if they have a common point and are
called intersecting lines. The common point O is their point of intersection.
Here, in the given figure, lines l and m are intersecting lines and common
point O, is the point of intersection.
• A line which intersects two or more given lines at distinct points is called a transversal for
the given lines. In the figure, n is a transversal.

• Two lines in a plane are said to be parallel if they do not intersect or are at equal distance
apart everywhere.

    l || m

• If two parallel lines are intersected by a transversal then eight angles


are formed and
(a) (1, 5); (4, 8); (2, 6), (3, 7) are called pairs of
corresponding angles.
(b) (3, 5), (4, 6), (2, 8) and (1, 7) are called pairs of
alternate angles.
(c) (1, 8), (2, 7) are called co-exterior angles.
(d) (4, 5), (3, 6) are called co-interior angles.
• If two parallel lines are intersected by a transversal then,
(a) each pair of corresponding angles are equal, i.e., 1 = 5; 2 = 6; 4 = 8 and 3 = 7
(b) each pair of alternate angles are equal, i.e., 3 = 5, 4 = 6, 2 = 8, 1 = 7
(c) interior angles on the same side of the transversal, i.e., co-interior and co-exterior angles are
supplementary, i.e., 4 + 5 = 180°; 3 + 6 = 180°; 1 + 8 = 180°; 2 + 7 = 180°.
• If any one of the above three statements (a, b, c) is true then two lines are necessarily parallel.

Exercise 5.2
I. Very Short Answer Type Questions (1 Mark)
A. Answer the following.
1. When are two lines said to be parallel?
2. What is a transversal?
3. If two lines are cut by a transversal then what is the number of angles formed?
4. If two parallel lines are cut by the transversal then what can be said about the pair of
alternate angles?

88 Mathematics–7
5. If two lines are cut by a transversal and a pair of alternate angles are equal then what
can be said about the two lines?
B. Fill in the blanks.
1. For the given figure, AB || CD, line EF is transversal.
If –APQ = 130° then –PQD = .......................... .
F

P B
A

C Q D

2. In the figure given in Q1, if –BPQ = 35°, then –DQE = .......................... .


3. In figure ..........................the line l is not a transversal.

l l

(a) (b)
4. In the figure given below, if –1 = 70°, then –2 = .......................... .
A
D

B 1 C
E 2 F

5. (i) Corresponding angle of the shaded angle is ........... .


(ii) Alternate angle of the shaded angle is ........... .
(iii) Vertically opposite angle of the shaded angle is ........... .
6. Two lines in a plane which never meet at any point are called ........... .
7. Sum of interior angles on the same side of a ........... with two parallel lines is 180°.

Lines and Angles  89


II.  Short Answer Type Questions–I (2 Marks)
Questions 1 to 5: Find x, if l || m.

1. 2.

y
3. 4.

5.

Questions 6 to 12. Check if l || m. Give reasons also.

6. 7.

8. 9.

10 11.
x

12 1

90 Mathematics–7
III.  Short Answer Type Questions–II (3 Marks)
1. In the figure, FE || DC || BA and BC || ED. Find the values of x and y.
F E B A
y
130°

z
x
D C
2. In the figure, QP || SR. Find angle a.

3. In the given figure, the arms of the two angles are parallel. If
ABC  =  70°, then find
(i) DGC
(ii) DEF
4. In the figure, AB || EF. Find the value of x  +  y.

5. In the figure, l || m. Find the values of a°, b° and c°.

IV. Long Answer Type Questions (4 Marks)


1. In the figure, four line segments PQ, QR, RS and ST
are making the letter W, PQ || RS and QR || ST. If angle
between PQ and QR is 39°, find the values of x and y.

Lines and Angles  91


2. In the figure, BA || EF; ED || BC and ÐAQE = 49°.
Find BPF.

2 1

3. Out of a pair of complementary angles, one is one-third of the other. Find the measures
of the angles.
4. Furniture: The legs of a stool make an angle of 45° with the floor as shown. Find a
and b.  [Multidisciplinary Questions]

5. Rangoli: Anisha has made a rangoli as shown in the figure below. She has used parallel
lines for drawing this.

p || s, q || t and r || l.
If v = 130°. Find x, y and z. [Multidisciplinary Questions]

Answers and Hints


I. A. 1. Two lines in a plane are said to 2. A line intersecting two or more lines at
be parallel if they do not intersect different points is called a transversal
or they are at equal distance apart for those lines.
everywhere. 3 . Eight (8)

92 Mathematics–7
4. Each pair of alternate angles are equal. Q P
60°
5. Lines are parallel.
B. 1. –PQD = 130° a 1
2
T

2. –DQE = 35° 3. (b) 4. 70° 30°


S R
5. (i) ∠4 (ii) ∠2 (iii) ∠6
6. parallel 7. Transversal ∠ 1 = 60° alternate
II. 1. x = 140° (Corresponding angles) ∠ 2 = 30° alternate
2. x = 60° (alternate angles) a = ∠1 + ∠2 = 60° + 30° = 90°
3. 110° + x = 180° (cointerior angles) 3. ∠ABC = 70° as AB || DE
⇒ x = 180° – 110° = 70° ⇒ ∠ABC = ∠DGC (corresponding angles)
4. y = x (alternate angles) (i) ∠DGC = 70° (corresponding angles)
⇒ y + 71° = 180° (linear pair) (ii) ∠DEF = ∠DGC = 70° (corresponding
⇒ y = 180° – 71° = 109° ⇒ x = 109° angles)
5. 2x + 3 = 55° (alternate angles) 4. 110° + x° = 180° (linear pair)
⇒ 2x = 55 – 3 x = 180° – 110° ⇒ x = 70°
52 AC || EF ⇒ y = 95°
⇒ 2x = 52 \ x = =26°
2 (alternate angles)
6. Yes, l || m as alternate angles are equal.
⇒ y = 95°
7. Yes, l || m as corresponding angles are
x + y = 70° + 95° = 165°
equal.
5. l || m ⇒ b = 40° (alternate angles)
8. Yes, l || m as interior angles on the same
side of the transversal are supplementary. c + 40° = 180° (interior angles on the same side
of the transversal)
9. Yes, l || m as alternate exterior angles
are equal. ⇒ c = 140°
10. l || m as corresponding angles are equal. 100° = a° + 40° (alternate angles)
⇒ a° = 100° – 40° = 60°
11. x + 68° = 180° ⇒ x = 180° – 68° = 112°
b = 40°
Yes, l || m as corresponding angles are
c = 140°
equal.
12. ∠1 = 75° vertically opposite angles IV. 1. PQ || RS ⇒ 39° = x (alternate angles)
⇒ ∠1 + 75° = 75° + 75° = 150° ≠ 180° RQ || ST ⇒ x° = y° = 39°
Hence, l || m. 2. ∠1 + 49° = 180° (linear pair)
III. 1. FE || DC ⇒ y = x (alternate angles) ⇒ ∠1 = 180° – 49° = 131°
DC || BA ⇒ z = 130° (alternate angles) BP || QE
BC || ED \ ∠1 + ∠2 = 180° (interior angles on same
⇒ x + z = 180° (interior angles on the side of the transversal)
same side of the transversal) ⇒ 131° + ∠2 = 180°
⇒ x = 180° – z = 180° – 130° ⇒ ∠2 = 180° – 131° = 49°
⇒ x = 50° ⇒ y = x = 50° ⇒ ∠BPF = ∠2 = 49° (alternate angles)
⇒ x = 50°, y = 50°, z = 130° 3. Let one angle be x other is 3x
2. Draw l || QP || SR ⇒ x + 3x = 90°
⇒ x = 22.5°
4x = 90° ⇒

Lines and Angles  93


Angles are x = 22.5°, 3x = 3 × 22.5° same side of the transversal)
= 67.5° ⇒ 130° + y = 180°
4. Furniture ⇒ y = 180° – 130° = 50°
∠ a = 45° (alternate angles) ⇒ u + x = 180° (interior angles on the
a + b = 180° (linear pair) same side of the transversal)
⇒ b = 180° – 45° = 135° ⇒ 130° + x = 180° ⇒ x = 50°
5. Rangoli x + w = 180° (interior angles on the same
v = 130° ⇒ u = v (vertically side of the transversal)
opposite angles) w = 180° – 50° = 130°
⇒ u = 130° z = w ⇒ z = 130° (vertically opposite
⇒ u + y = 180° (interior angles on the angles)

Subject Enrichment Activities


I. Activity
In the figure, a 4 × 4 square grid is given. Copy the grid and draw all possible
shaped parallelograms on it. Note that each vertex of a parallelogram must
be on a dot. Rectangles and squares are not allowed. One parallelogram
has been drawn. A parallelogram is a quadrilateral whose opposite sides are
parallel and equal.
II. Match the columns
In the given figure, l || m and n is a transversal. Observe the figure and match the columns.
Column I Column II
(i) Vertically opposite angle to g (a) c
(ii) Linear pair of c (b) a
(iii) Alternate interior angle of e (c) e
(iv) Alternate exterior angle of g (d) b
(v) Corresponding angle of h (e) d
(vi) Supplement of f (f ) g
III. Multiple Choice Questions
1. Parallel lines are always
(a) equal (b) coincident (c) equidistant (d) concurrent
2. The angles between North and West and South and East are

94 Mathematics–7
(a) complementary (b) supplementary (c) both are acute (d) both are obtuse
3. If the complement of an angle is 89°, then the angle will be
(a) 1° (b) 11° (c) 89° (d) 91°
4. In the figure, the value of x is

(a) 100° (b) 56° (c) 54° (d) 140°

5. If two lines are intersected by a transversal then:


(a) alternate angles so formed add up to 180°
(b) sum of the interior angles on the same side of the transversal is 180°
(c) corresponding angles so formed add up to 180°
(d) none of these
6. When two parallel lines are intersected by a transversal, how many pairs of alternate interior
angles are formed?
(a) 1 (b) 2 (c) 3 (d) 4
7. The number of transversals that can be drawn for two given lines is
(a) 1 (b) 2 (c) 3 (d) infinite

8. ZOX is a straight line. Find the value of YOZ.

(a) 22° (b) 142° (c) 125° (d) 38°

9. Two angles forming a linear pair are in the ratio 1 : 5, then the angles are
(a) 60°, 120° (b) 50°, 130° (c) 30°, 150° (d) 70°, 110°

10. Which of the following is false?

(a) c = d (b) a + b = 180°


(c) b = d (d) c + d = 180°

Lines and Angles  95


Answers
I. Do it yourself. ⇒ 10x – 40 = 180° ⇒ 10x = 220°
II. (i)–(c), (ii)–(d)(e), (iii)–(a), 220
⇒ x = = 22°
(iv)–(b) (v)–(e), (vi)–(c)(f) 10
III. 1. (c) equidistant 2. (b) supplementary So, ∠ YOZ = 6x + 10
3. (c) 90° – 89°  =  1° = 6 × 22° + 10 = 132° + 10 = 142°
4. (d) 110° + 56° + 54° + x° = 360° 9. (c) Let the ratio of constant be x, then
the angles are x, 5x
⇒ 110° + 110° + x° = 220° + x° = 360°
⇒ x + 5x = 180° ⇒ 6x = 180°
⇒ x = 360° – 220° = 140°
180°
5. (b) sum of the interior angles on the ⇒ x = = 30° ⇒ x = 30°
6
same side of the transversal is 180°. ⇒ Angles are x = 30°
6. (b) 2 7. (d) infinite ⇒ 5x = 5 × 30 = 150°
8. (b) 6x + 10 + 4x – 50 = 180° 1 0. (a) ∠c = ∠d as ∠c ≠ ∠d
qqq

96 Mathematics–7
6 The Triangle and Its Properties
Topics Covered
6.1 Triangles 6.2  Exterior Angle and Its Property
6.3 Sum of the Angles of a Triangle 6.4  Various kinds of Triangles
6.5  Pythagoras Theorem

Let’s Remember
• The three-sided closed figure obtained by joining three non-collinear points in a plane is
called a triangle.
• The three line segments forming a triangle are called the sides of the triangle.
• The three sides and three angles of a triangle together form the six elements of the triangle.

6.1  Triangles
• A triangle with each angle less than 90° is an acute angled triangle.

• A triangle with one of its angle equal to 90° is called a right angled triangle.

• A triangle with one of its angle greater than 90° is an obtuse angled triangle.

• Line segment joining a vertex of the triangle to the mid-point of its


opposite side is called a median of the triangle. In figure, AD
is the median.
• Medians of a triangle always lie in its interior.
(a) In a scalene triangle, all medians are of different length.

97
(b) In an isosceles triangle, the medians to the equal sides are equal in length.
(c) In an equilateral triangle, all three medians are equal in length.
• A perpendicular line segment drawn from a vertex of a triangle to
its opposite side is called an altitude of the triangle. In figure, AD
is the altitude.
(a) In any triangle, three altitudes can be drawn.

(b) In an acute triangle altitudes lie in the interior of the triangle.

(c) In a right triangle, the two arms of the right angle are
also its two altitudes. The other altitude lies in the interior
of the triangle.
(d) In an obtuse triangle, two altitudes lie outside the triangle.

(e) • In a scalene triangle, the altitudes are different in length.


• In an isosceles triangle, the altitudes to the equal sides are equal in length.
• In an equilateral triangle, all three altitudes are equal in length.

Exercise 6.1
I. Very Short Answer Type Questions (1 Mark)
A. Answer the following questions.
1. What is meant by median of the triangle?
2. Write the number of medians a triangle has?
3. Where does the median of a triangle lie, in its interior or exterior?
4. Are medians of a scalene triangle equal?
5. What do you mean by the altitude of a triangle?

98 Mathematics–7
6. In which triangle all medians are equal?
7. How many altitudes can a triangle have?
8. Write the number of altitudes of a right triangle that can lie on its sides.
9. Are altitudes of an equilateral triangle equal in length?
10. What can you say about the medians of an acute triangle?
11. What can be said about the altitudes of an acute triangle?
12. State the type of a triangle in which all medians are also the altitudes.
13. State the type of the triangle in which two of its sides are also its altitudes.
14. Can a triangle have any of its medians in its exterior?
15. State the type of a triangle in which exactly one median is also the altitude of the triangle.
B. Fill in the blanks.
1. The line segment joining a vertex of a triangle to the ............. of its opposite side is called
its median.
2. Two altitudes of an ............. triangle are always outside itself.
C. Say True or False.
1. A median of an obtuse triangle may lie in its exterior.
2. Median of a triangle is the line segment joining mid-point of a side of the triangle to its
opposite vertex.
3. A triangle can have six medians.
4. We can draw infinite number of altitudes of a triangle.
5. In a scalene triangle all medians are unequal in length.
6. All altitudes of an equilateral triangle are equal.
7. In an obtuse triangle all its altitudes lie outside the triangle.
II. Short Answer Type Questions–I (2 Marks)
1. In DABC, AD is the bisector of A meeting BC in D, CF  AB and E is the mid-point
of AC. Write the median of the triangle.
2. Draw a triangle which will have two of its altitudes outside the triangle.
3. In DPQR, if median PS = median QT = median RU. Is DPQR an equilateral triangle?
4. In DABC, altitude AD = altitude BE  altitude CF. Is DABC an isosceles triangle?
5. In an equilateral triangle all three altitudes are unequal in length. Justify your answer.
III. Short Answer Type Questions–II (3 Marks)
1. AM is a median of a triangle ABC.

Is AB + BC + CA > 2AM?

The Triangle and Its Properties  99


IV. Long Answer Type Questions (4 Marks)
1. ABCD is a quadrilateal.
Is AB + BC + CD + DA > AC + BD?
2. ABCD is quadrilateral.
Is AB + BC + CD + DA < 2(AC + BD)?
Answers and Hints
I. A. 1. Line segment joining a vertex of Adding (1) and (2), we get
the triangle to the mid-point of its AB + (BM + CM) + CA > AM + AM
opposite side is called a median of
⇒ AB + BC + AC > 2AM.
the triangle.
2. 3 3. Interior 4. No IV. 1. The sum of the lengths
5. A perpendicular line segment drawn from of any two sides of a
a vertex of a triangle to its opposite side triangle is greater than
is called an altitude of the triangle. the third side.
6. Equilateral 7. Three 8. Three \ In DABC, we have
9. Yes 10. Lie inside (AB + BC) > AC …(1)
11. Lie inside 12. Equilateral triangle Similarly, in DACD, we have
13. Right angled Triangle (CD + DA) > AC …(2)
14. No Adding (1) and (2), we have
15. Iscosceles triangle
[(AB + BC) + (CD + DA] > 2AC …(3)
B. 1. mid-point 2. obtuse
Again,
C. 1. False 2. True 3. False
In DABD, we have
4. False 5. True 6. True
7. False AB + DA > BD  …(4)
II. 1. BE In DBCD, we have
2. BC + CD > BD …(5)
Adding (4) and (5), we have
[(AB + DA) + (BC + CD)] > 2BD …(6)
Now adding (3) and (6), we have
2[AB + BC) + (CD + DA)] > 2 (AC + BD)
or (AB + BC + CD + DA) > (AC + BD)
2. Since the sum
AD ⊥ DC, triangle with altitude AD is of lengths of
outside of the DABC. any two sides
3. Yes. 4. Yes of a triangle
is greater than
5. False. In an equilateral triangle all three
the length of
altitudes are of equal length.
the third side.
III. 1. Yes.
\ In DOAB, we have
Hint: In DABM, AB + BM > AM …(1)
(OA + OB) > AB …(1)
Similarly, in DACM, AC + CM > AM
Similarly,
…(2)
In DOBC, we have

100 Mathematics–7
(OB + OC) > BC …(2) > (AB + BC + CD + DA)
In DOCD, we have ⇒ (AB + BC + CD + DA)
(OC + OD) > CD …(3) < 2[OA + OB + OC + OD]
In DOAD, we have ⇒ (AB + BC + CD + DA)
(OA + OD) > AD  …(4) < 2[(OA + OC) + (OB + OD)]
Adding (1), (2), (3) and (4), we have (AB + BC + CD + DA) < 2[AC + BD]
2[OA + OB + OC + OD]
6.2  Exterior angle and Its Property
• An exterior angle of a triangle is formed when a side of a triangle is produced at any vertex.
In the adjacent figure, ACX and BCY are exterior angles.

• Exterior angle of a triangle is always equal to the sum of its interior


opposite angles. In the figure, exterior angle ACD = A + B.

Exercise 6.2
I. Very Short Answer Type Questions (1 Mark)
A. Answer the following.
1. In the given figure, find the measure of –Q.
2. Write the number of ways in which an exterior angle can
be drawn at each vertex of a triangle.
3. What is the sum of the exterior angles of a triangle?
4. What do you understand by an exterior angle of a triangle?
5. State the exterior angle property.
6. What is the sum of the exterior angle and an adjacent interior angle of a triangle?
B. Fill in the blanks.
1. In Fig. –XZP = –YXZ + .............
2. In a triangle the exterior angle and its adjacent angle of a triangle
always form a .............  .
3. The exterior angle at the vertex C of a triangle ABC is 130°
and A = 70°, then B = .............
4. In Fig., 1 + 2 + 3 = .............

The Triangle and Its Properties  101


C. Say True or False.
1. In the given figure, OP + OQ > PQ.
2. It is possible to have a triangle with sides 2  cm, 3  cm, 5 cm.
II.  Short Answer Type Questions–I (2 Marks)
1. If the exterior angle of a triangle is 130° and its interior opposite angles are equal, find
the measure of each interior opposite angle.
2. In DXYZ, if X = 50° and Y = 40°, then find the measure of the exterior angle formed
by producing the side YZ.
3. In the figure, if AB || CD, then 2 = ? (Why?)

4. In the figure, find the value of x and y.

III.  Short Answer Type Questions–II (3 Marks)


1. One of the exterior angles of a triangle is 120° and the interior opposite angles are in the
ratio 1: 5. Find the angles of the triangle.
2. In the figure, find the following:
(i) –PRS (ii) –PTS

3. In the figure, find the value of x and y.

102 Mathematics–7
IV. Long Answer Type Questions (4 Marks)
1. In the figure, find (i) –ACD (ii) –AED (iii) –CED (iv) –ACB

2. Show that sum of the exterior angles of DABC shown below is 360°.

OR
The sides of a triangle are produced in such an order that form exterior angles, show that
the sum of the exterior angles so formed is 360°.
3. In the following figure, find the values of x and y.

Answers and Hints


I. A. 1. 70° 2. Two 3. 360°
4. An exterior angle of a triangle is
formed when a side of a triangle
is produced at any vertex. In figure
∠ACX and ∠BCY are exterior
5. E xterior angle of a triangle is
angles.
always equal to the sum of its
interior opposite angles. In the figure

The Triangle and Its Properties  103


given below, the exterior angle, III. 1. Exterior angle –ACD = 120°
ACD = A + B.

6. 180°
B. 1. ∠XYZ 2. linear pair But –ACD = –A + –B
3. 60° 4. 360° and –A : –B = 1 : 5 (given)
C. 1. True 2. False Let x be the ratio constant.
II. 1. Let each interior opposite angle be x, ⇒ –A = x and B = 5x
then So, –ACD = –A + –B
= x + 5x = 6x
⇒ 120° = 6x ⇒ x = 20°
Thus, –A = x = 20°,
–B = 5x = 5 × 20° = 100°;
–C = 180° – 120° = 60°.
2. –PRS is an ext. angle of the DPQR
⇒ x + x = 130° ⇒ 2x = 130° –PRS = –P + –Q
= 100° + 25° = 125°
130°
\ x = = 65° (Exterior angle theorem)
2
2. –PTS is an ext. angle of DTRS
So, –PTS = –TSR + –SRP
= 20° + 125° = 145°
3. Given y is an ext. angle of the DDEF.
So, y = –D + –F = 50° + 18°
= 68° (Ext. angle theorem)
–x is an ext. angle of DHEG
\ x° = y° + 35°
∠XZU = ∠X + ∠Y
= 68° + 35° (ext. angle theorem)
[Exterior angle is equal to
= 103°.
sum of interior angles]
IV. 1. (i) In DABC, –ACD is an exterior
= 50° + 40° = 90° angle.
3. ∠2 = ∠4 (corresponding angle) \ –ACD = –A + –B
4. ∠ACD = ∠A + ∠B (Ext. angles) = 50° + 35° =85°
⇒ 135° = x + 65° ( ) –AED is an exterior angle of DCDE.
⇒ x = 70° \ –AED = –ACD + –EDC
∠ACD + y = 180° = 85° + 45° = 130°
135° + y = 180° (iii) –CED = 180° – (–DCE + –EDC)
⇒ y = 180° – 135° = 45° (Angle sum property)

104 Mathematics–7
= 180° – (85° + 45°) = 180° – 130° = 50° ⇒ –1 + –2 + –3 = 540° – 180° = 360°
(iv) –ACB = –CED + –D (ext. angle) Hence proved.
= 50° + 45° = 95° 3. –TRS is an ext. angle of DRPS
2. –2 + –4 = 180° (L.P.A) …(1) y° + 65° = 40° + 105°
–3 + –5 = 180° (L.P.A)  …(2) ⇒ y° + 65° = 145°
–1 + –6 = 180° (L.P.A)  …(3) ⇒ y° = 145° – 65° = 80°
Adding (1), (2) and (3), we get Q x + 65° + y° = 180°
(Angles at a point on a straight line)
–1 + –2 + –3 + – 4 + –5 + –6
⇒ x + y = 180° – 65°
= 180° + 180° + 180°
⇒ x + y = 115°
⇒ –1 + –2 + –3 + (–4 + –5 + –6)
= 540° ⇒ x + 80° = 115°
⇒ –1 + –2 + –3 + 180° = 540° ⇒ x = 115° – 80° = 35°.

6.3  Sum of the angles of a Triangle

• Sum of the angles of a triangle is equal to 180°, i.e.,


in DABC, A + B + C = 180°.

Exercise 6.3
I. Very Short Answer Type Questions (1 Mark)
A. Answer the following.
1. Is it possible to have a triangle in which
(i) two of the angles are right angles? (ii) two of the angles are acute?
(iii) two of the angles are obtuse? (iv) each angle is less than 60°?
(v) each angle is greater than 50°? (vi) each angle is 60°?
(vii) each angle is greater than 60°?
2. State the angle sum property of a triangle.
3. What is the relation between the sum of the exterior and interior angles at a vertex of a
triangle?
4. DABC is a right triangle with B = 90°. What kind of pair of angles are A and C?
5. In an isosceles triangle measure of the vertical angle is 80°. What is the measure of the
other two angles?
6. If an exterior angle of a triangle is 130° and one of its interior opposite angles is 60°,
find the other interior opposite angles.
B. Fill in the blanks.
1. If three angles of a triangle are equal, then each of them is equal to ............. .
2. Two angles of a triangle are 70° and 30°, then its third angle is ............. .
3. Every triangle has at least ............. acute angle(s).

The Triangle and Its Properties  105


4. In DABC shown below, B = .............and C = ............. .

C. Say True or False.


1. A triangle can have two right angles.
2. It is possible to have a right-angled equilateral triangle.
3. If two angles of a triangle are equal, the third angle is also equal to each of the other
two angles.
II.  Short Answer Type Questions–I (2 Marks)
1. In the given figure, find the value of P + Q + R + S + T + U

2. In DPQR, P = 100°, PS bisects P and PS   QR. Find the measure of Q.
3. In the figure given below, in DABC, A = 50°, B = 70° and bisector of C meets AB
at D. Find the measure of ADC.

4. In the figure given below, find the value of x,

5. Find the value of ‘a’ for each of the following:

(i) (ii)

106 Mathematics–7
(iii) (iv)

6. The sum of the measures of three angles of a triangle is greater than 180°. Justify your
answer.
7. Is it possible to have a triangle with two obtuse angles? Justify your answer.
III. Short Answer Type Questions–II (3 Marks)
1. Check if it is possible to draw a triangle with angles 117°, 25° and 38°. Give reasons.
2. If two angles of a triangle are 90° and 45°, then show that it is an isosceles triangle.
3. In the figure given below, QP || RT. Find the values of x, y and  z.

4. The angles of a triangle are in the ratio 2 : 3 : 5. Find the angles.
5. In a DXYZ, if X = Z and exterior angle XYU = 140°, then find the angles of the
triangle.
6. Each of the two equal angles of an isosceles triangle is four times the third angle. Find
the angles of the triangle.
7. Find the value of x in the figure given below.

8. In the given figure, find the value of x and y.

9. Find the values of ‘a’ and ‘b’ for each of the following:

(i) (ii) (iii)

The Triangle and Its Properties  107


IV.  Long Answer Type Questions (4 Marks)
1. In DPQR, if P = 3Q and R = 2Q, find all the angles of DPQR.
2. In the figure given below, DPQR is right angled at R and RS  PQ. Also P = 55°.

Find:
(i) PRS (ii) QRS (iii) RQS
3. The angles of a triangle are in the ratio 2 : 3 : 4. Find the angles.
4. Find the value of angle x in the figure.

5. In the figure given below, S and T are the points on sides


PQ and RQ of DPQR, such that ST || PR. If Q = 30°
and P = 40°, find the values of x, y and z.

6. In the figure given below, ED || AB find the values of x and y.

7. In DABC shown below, line DE || BC. Find the value of x, y and z.

8. Find the measure of x and y in the triangle given below:

108 Mathematics–7
9. Find the values of a, b, c and d in the given figure.

10. Find the value of the missing angle(s).  [HOTS]

11. Find x in the given figure.  [HOTS]

Answers and Hints


I. A. 1. (i) No (ii) Yes (iii) No are equal, therefore two sides are equal.
(iv) No (v) Yes (vi) Yes Hence, this is an isosceles triangle.
(vii) No 3. x = 70°, y = 80°, z = 30°
4. 36°, 54°, 90°
2. Sum of angles of a triangle is 180°.
5. ∠X = ∠Z = 70°, ∠Y = 40°
3. 180° 4. Acute angles 6. 20°, 80°, 80° 7. 40°
5. 50° each 8. y = 60°, x = 120°
6. Other angle is = 130° – 60° = 70° 9. (i) a = 50°, b = 70°
B. 1. 60° 2. 80° 3. two (ii) a = 60°, b = 60° (iii) a = 55°, b = 70°
4. ∠B = 75° and ∠C = 75° IV. 1. ∠P = 90°, ∠Q = 30°, ∠R = 60°
C. 1. False 2. False 3. False Hint: Q ∠P + ∠Q + ∠R = 180°
II. 1. 360° 2. 40° 3. 100° ⇒ ∠3Q + ∠Q + 2Q = 180°
4. x = 35° 5. (i) a = 43.5°
(ii) a = 20° (iii) a = 57°
(iv) a = 50°
6. False. Sum of the angles of a triangle
is 180°.
7. No. Triangle can have maximum one
obtuse angle otherwise sum of angles
will exceed 180°.
⇒ 6∠Q = 180°
III. 1. Yes, because sum of angles is 180°.
\ ∠Q = 30°
2. Third angle is 45°. Since two angles
∠P = 90°, ∠R = 60°

The Triangle and Its Properties  109


2. (i) ∠PRS = 35° Now, in DCED,
Hint: In DPRS, ∠c + y + x = 180°
∠ P + ∠PSR + ∠PRS = 180° 80° + 70° + x = 180°
55° + 90° + ∠PRS = 180°
⇒ 150° + x = 180°
145° + ∠PRS = 180°
\ ∠PRS = 180° – 145° = 35° \ x = 180 –150° = 30°
(ii) ∠QRS = 55° 7. x = 60°, y = 50°, z = 70°
Hint: In DQRS, Hint: Q DE || BC, ⇒ y = 50°
∠Q + ∠QRS + ∠RSQ = 180° (corresponding angles)
35° + ∠QRS + 90° = 180°
Also, z = 70° (corresponding angles)
∠QRS = 180° – 125° = 55°
(iii) ∠RQS = 35°. Now, in DADE,
Hint: In DPQR, ∠A + ∠ADE + ∠AED = 180°
∠P + ∠Q + ∠R = 180° (Angle sum property)
⇒ 55° + ∠Q + 90° = 180° ⇒ x° + y° + 70° = 180°
\ ∠Q = 180° – 145° = 35° ⇒ x° + 50° + 70° = 180°
3. 40°, 60°, 80°
Hint: Let x be the ratio constant. ⇒ x + 120° = 180°
So that, 2x + 3x + 4x =180° \ x = 180° – 120° = 60°.
180 8. y = 105°, x = 40°
⇒ 9x = 180° \ x = = 20°
9 ∠BDC = ∠A + ∠ACD
Thus, the angles are 40°, 60° and 80°.
4. 60° \ y = 65° + 40° = 105°
Hint: Q AB | | CD, and BC is a transversal. In DABC,
\ ∠DCE = ∠ABC = 80° ∠A + ∠B + ∠C = 180°
Now, in DCDE ⇒ 65° + 35° + x + 40° = 180°
x + ∠D + ∠C = 180° ⇒ 140° + x = 180°
⇒ x + 40° + 80° = 180° \ x = 180° – 140° = 40°
⇒ x + 120° = 180° 9. a = 55°, b = 65°, d = 30°, c = 60°
⇒ x = 180° –120° = 60° 10. a = 35°, b = d = 105°, f = 100°,
5. x = 40°, y = 110°, z = 110° h = 105°, c = e = 75°, g = 40°
Hint: In DPQR, 11. x = 95°
∠P + ∠Q + ∠R = 180° Hint: In DABC
⇒ 40° + 30° + z° = 180° ∠A + ∠B + ∠ACB = 180°
⇒ 70° + z° = 180° (Angle sum property)
\ z° = 180° – 70° = 110° ⇒ 35° + 110° + ∠ACB = 180°
Also, x° = 40° and y° = z° = 110° ⇒ 145° + ∠ACB = 180°
(Since ST | | PR, corresponding angles) \ ∠ACB = 180° – 145° = 35°
6. x = 30°, y = 70° Again in DDEF,
Hint: ∠AED + ∠CED = 180° (Linear pairs) ∠D + ∠E + ∠DFE = 180°
⇒ 110° + y = 180° (Angle sum property)
\ y = 180° – 110° = 70° ⇒ 100° + 30° + ∠DFE = 180°

110 Mathematics–7
⇒ 130° + ∠DFF = ∠180° ⇒ ∠ACB + ∠DFE + ∠AGE = 180°
⇒ ∠DFE = 180° – 130° = 50° (Q ∠FGC = ∠AGE, ver. opp. ∠S)
Now in DFCG, ⇒ 35° + 50° +∠AGE = 180°
∠GCF + ∠CFG + ∠FGC = 180° ⇒ 85° + x = 180°
(Angle sum property) ⇒ x = 180° – 85 = 95°.

6.4 Various Kinds of Triangles

• A triangle with all three sides unequal is called a


scalene triangle.

• A triangle with two equal sides is called an isosceles


triangle.

• A triangle with all three sides equal is called an


equilateral triangle.

• A triangle with each angle less than 90° is an acute


angled triangle.

• A triangle with one of its angle equal to 90° is called


a right angled triangle.

• A triangle with one of its angle greater than 90° is


an obtuse angled triangle.

The Triangle and Its Properties  111


• In a right triangle, side opposite to right angle is the
hypotenuse and it is the longest side. (AB > BC,
AB  > AC)

• The sum of any two sides of a triangle is greater than


the third side.
i.e., b + c > a, c + a > b and a + b > c.

Exercise 6.4
I. Very Short Answer Type Questions (1 Mark)
A. Answer the following.
1. Can 3 cm, 4 cm and 5 cm be the sides of a triangle?
2. The sides of a triangle have lengths (in cm) 10, 6.5 and a, where a is a whole number.
What is the minimum value that a can take?
3. In DPQR, PQ – QR is greater or less than PR?
4. Does the following dimensions represent the sides of a triangle?
2 cm, 3 cm, 4 cm.
5. State the relation between the sum of any two sides of a triangle to its third side.
6. State the relation between the difference of any two sides of a triangle to its third side.
B. Fill in the blanks.
1. In a triangle, the sum of the lengths of any two sides is always ............. than the length
of the third side.
2. In an ............. triangle two angles are always equal.
3. In an equilateral triangle measure of each angle is ............. .
4. In an ............. triangle only the base angles have the same measure.
5. The unequal side of an isosceles triangle is called its ............. .
C. Say True or False.
1. The difference between the lengths of any two sides of a triangle is smaller than the
length of the third side.
2. Sum of any two angles of a triangle is always greater than the third angle.
II.  Short Answer Type Questions–I (2 Marks)
1. In the given figure, PR = QR and P = 50°. Then find PRQ.

112 Mathematics–7
2. Sum of any two sides of a triangle is greater than or equal to the length of the third side.
Justify.
3. In DABC, M is a point on the side BC.

(i) AB + BM ___________ AM (ii) AC + CM ___________ AM


4. A is point in the interior of DABC. Justify if the following are True or False.

(i) AG + GB > AB (ii) AG + AC < GC


III.  Short Answer Type Questions–I (3 Marks)
1. Is it possible to draw a triangle with sides 9  cm, 10 cm and 11 cm? Give reasons for
your answer.
2. If two angles of a triangle are 60° each, then show that the triangle is an equilateral
triangle.
3. The lengths of two sides of a triangle are 8  cm and 9.5 cm. Between what two measures
the length of the third side lies?
4. ABCD is a quadrilateral.

Is AB + BC + CD + DA > AC + BD?
IV.  Long Answer Type Questions (4 Marks)
1. D is a point on the side BC of a DABC. Show that perimeter of DABC > 2AD.
2. O is a point in the interior of a triangle ABC. Show that: 2(OA + OB + OC) > AB
+ BC + AC.
3. Prove that in an equilateral triangle, all the exterior angles are equal. [HOTS]
4. In the figure given below, x : y = 3 : 5 and –DFG = 160°, find the values of x, y
and z.

The Triangle and Its Properties  113


Answers and Hints
I. A. 1. Yes 2. 4 3. Less III. 1. Yes, because sum of any two sides of
4. 3 cm + 2 cm = 5 cm, 5 > 4 a triangle is greater than third side.
2 cm + 4 cm = 6 cm, 6 > 3 2. Suppose the third angle = x, then
3 cm + 4 cm = 7 cm, 7 > 2 60° + 60° + x = 18 (Angle sum property)
Hence 2 cm, 3 cm and 4 cm can be ⇒ 120° + x = 180°
the lengths of the sides of a triangle.
\ x = 180° – 120° = 60°
5. In triangle sum of any two sides is
always greater than the third side. Since all the three angles of the triangle is
60°. Thus the triangle is an equilateral triangle.
6. In a triangle difference of lengths of
any two sides of a triangle is less than 3. Measure of third side lies between 1.5
the third side. and 17.5
B. 1. greater 2. isosceles 3. 60° 4. Since sum of two sides of a triangle is
4. isosceles 5. base greater than the third side. So, in DABC,
C. 1. True 2. False. AB + BC > AC …(1)
II. 1. 80° And, in DABD,
Q PR = QR AB + AD > BD …(2)
⇒ ∠P = ∠Q = 50° (∠S opp. to equal
In DACD,
sides are also equal.)
Now, in DPQR, AD + CD > AC …(3)
∠P + ∠Q + ∠PRQ = 180° Also, in DBCD,
(Angle sum property) BC + CD > BD …(4)
⇒ 50° + 50° + ∠PRQ = 180° Adding (1), (2), (3) and (4), we get
⇒ 100° + ∠PRQ = 180° 2 (AB + BC + CD + AD) > 2 (AC + BD)
\ ∠PRQ = 180° – 100° = 80° ⇒ AB + BC + CD + DA > AC + BD
2. False. Sum of any two sides of a triangle IV. 1. Perimeter of DABC
is greater than the length of the third = AB + BC + CA
side.
3. (i) AB + BM > AM
(ii) AC + CM > AM
4. (i) In DAGB, AG + GB > AB (True)
(Sum of any two sides of a triangle is
greater than the third side.)
(ii) In DAGC
AG + AC < GC (False)
(Sum of any two sides of a triangle is Since sum of two sides of a triangle is greater
greater than the third side. than the third side, so

114 Mathematics–7
AB + BD > AD …(1) opposite angle. Thus,
And AC + DC > AD  …(2)
Adding (1) and (2), we get
⇒ AB + BD + DC + AC > AD + AD
⇒ AB + (BD + DC) + AC > 2 AD
⇒ AB + BC + AC > 2AD
Thus, perimeter of DABC > 2AD
2. Since, sum of two sides of a triangle is
greater than the third sides. So

∠BAC + ∠ABC = ∠ACD


⇒ a + a = x
\ 2a = x …(1)
Similarly, 2a = y  …(2)
and, 2a = z …(3)
From (1), (2) and (3), we get
x = y = z
In DAOB, Hence, in an equilateral triangle, all the
OA + OB > AB  …(1) exterior angles are equal.
In DBOC, 4. Let x = 3K and y = 5K
OB + OC > BC …(2) –DFG = 160° ⇒ x + y = 3K + 5K
160°
And, in DAOC ⇒ 8K = 160° ⇒ K = = 20°
8
OC + OA > AC  …(3) ⇒ x = 3 × 20° = 60°
Adding (1), (2) and (3), we get y = 5 × 20° = 100°
2 (OA + OB + OC) > AB + BC + AC. z° + 160° = 180° (Linear pair of angle)
3. ABC is an equilateral triangle such that ⇒ z° = 180° –160°
∠ABC = ∠BCA = ∠CBA = a (suppose) ⇒ z° = 20°
Since exterior angle is equal to the interior \ x = 60°, y = 100°, z = 20°

6.5 Pythagoras theorem
• Pythagoras theorem states that in a right triangle, the
square of the hypotenuse equals the sum of the squares of
the remaining two sides, i.e., b2 = a2 + c2.

• Converse of Pythagoras theorem: If in a triangle, the square of the longest side is equal
to the sum of the squares of the remaining two sides, then the angle opposite to the longest
side is a right angle.

The Triangle and Its Properties  115


• Of all the segments that can be drawn to a given line from
a point outside it, the perpendicular line segment is the
shortest i.e., AB is the shortest in the figure.

Exercise 6.5
I. Very Short Answer Type Questions (1 Mark)
A. Answer the following.
1. A boy cycles 10 km South-East, then 8 km West. How far is he from his initial position?
2. Examine whether the following are Pythagorean triplets.
(i) (5, 12, 13) (ii) (9, 10, 11)
3. In a right triangle PQR, PQ + RQ2 = PR2. Write the hypotenuse of the triangle.
2

4. In a right triangle ABC, AB2 + BC2 = AC2. Which angle is equal to 90°?
B. Fill in the blanks.
1. In a right angled triangle, the square of its ............. is equal to the sum of the squares of
its ............. .
C. Say True or False.
1. In a right triangle, sum of squares of two sides is equal to the square of the longest  side.
II. Short Answer Type Questions–I (2 Marks)
1. Find the perimeter of the rectangle whose length is 240 cm and a diagonal is 244 cm.
2. In a right-angled triangle PQR, if –Q = 90°, QR = 3 cm and PR = 5 cm, then find the
length of the side PQ.
3. Determine whether the triangle whose lengths of sides are 3 cm, 4 cm, 5 cm is a right-
angled triangle.
4. The sides of a right-angled triangle are 15 cm, 9 cm, and 12 cm. What is the length of
its hypotenuse.
III. Short Answer Type Questions–II (3 Marks)
1. Determine whether a triangle with the given lengths is a right triangle or not.
1.5 cm, 2 cm and 2.5 cm.
2. A cyclist goes 3 km east. Now it turns right and goes another 4 km. How far is the cyclist
from its starting point? [Multidisciplinary Questions]
3. An aeroplane leaves an airport and flies due North at a speed of 500 km per hour. At
the same time, another aeroplane leaves the same airport and flies due East at a speed of
1200 km per hour. How far apart will be the two planes after 2  hours? 
[Multidisciplinary Questions]

116 Mathematics–7
4. Manasi takes the shortest route to her home by walking diagonally across a rectangular park.
The park measures 60 m × 80 m. How much shorter is the route across the park than the
route around its edges? Is it better to walk or using car for short distances? Why?  [VBQs]
5. Two poles of height 10 m and 15 m stand upright on a plane ground. Kuber knows that
the distance between the tops is 13 m. How did he find the distance between their feet?
What values are shown by Kuber? [VBQs]
IV.  Long Answer Type Questions (4 Marks)
1. The diagonals of a rhombus measure 12 cm and 16 cm. Find its perimeter.
2. PQR is an isosceles triangle right angled at R. Prove that PQ2 = 2PR2.
3. A tree is broken at a height of 5  m from the ground and its top touches the ground at a
distance of 12  m from the base of the tree. Find the original height of the tree.

Answers and Hints


I. A. 1. x2 = 102 – 82 = 100 – 64 ⇒ 57600 + x2 = 59536
x = 36 = 6 km ⇒ x2 = 59536 – 57600
⇒ x2 = 1936 ⇒ x2 = (44)2
\ x = 44 cm
Now, perimeter of rectangle
= 2 (AB + AD) = 2 (240 + 44)
= 2 × 284 = 568 cm
2. 4 cm
2
2. (i) 132 = 169, 12 = 144, 52 = 25 Hint:
 169 = 144 + 125
 132 = 122 + 52
\ (5, 12, 13) is a Pythagorean triplet
(ii) 112 = 121, 102 = 100, 92 = 81
92 + 102 = 81 + 100 = 181 π 121 = 112
\ (9, 10, 11) is not a Pythagorean triplet
3. PR 4. –B.
B. 1. hypotenuse, legs Q PQ2 + QR2 = PR2
C. 1. True ⇒ PQ2 + 32 = 52 ⇒ PQ2 + 9 = 25
II. 1. 568 cm ⇒ PQ2 = 25 – 9 ⇒ PQ2 = 16
Hint: ⇒ PQ2 = 42 \ PQ = 4 cm
3. Yes, because 3 + 42 = 52
2

4. Hypotenuse is 15 cm. because


92 + 122 = 152.
III. 1. 2.52 = 6.25, 1.52 + 22 = 2.25 + 4 = 6.25
Yes, it is a right-angled triangle.
Q AB2 + AD2 = BD2 2. 5 km
(240)2 + x2 = (244)2

The Triangle and Its Properties  117


Hint: Value: To keep the body fit, walking is must.
To use car is not good as it can cause air
pollution and sound pollution and wastage
of money.
5. 12 m
Hint:

OB2 = OA2 + AB2


= 32 + 42 = 9 + 16 = 25
⇒ OB2 = 52
\ OB = 5 km.
Thus, the cyclist is 5 km from its starting
point in South-East direction.
3. 2600 km
Hint:
AB = 15 m, CD = 10 m
AE = AB – BE = AB – CD
= 15 –10 = 5m
DE2 = AD2 – AE2
= 132 –52 = 169 – 25 = 144
⇒ DE2 = 122
\ DE = 12 m
AB2 = OA2 + OB2
Values shown by Kuber is scientific
= (1000)2 + (2400)2 = 1000000 + 5760000 temperament.
= 6760000 IV. 1. 40 cm
\ AB2 = (2600)2 ⇒ AB = 2600 Km. Hint:
4. 40 m
Hint: AC2 = AB2 + BC2
= 802 + 602 = 6400 + 3600 = 10000
AC2 = (100)2

\ AC = 100 m

AC = 16 cm
1 1
\ OA = × AC = × 16 = 8 cm
2 2
BD = 12 cm
1 1
AB + BC = 80 m + 60 m = 140 m \ OB = × BD = × 12 = 6 cm
2 2
The route is shorter by 40 m (i.e., 140 m In right angled triangle AOB,
– 100 m) across the park.

118 Mathematics–7
AB2 = OA2 + OB2 = 82 + 62 = PR2 + PR2  [Q PR = QR]
= 64 + 36 = 100 2
PQ = 2PR . 2
2 2
⇒ AB = 10 3. Height of tree is 18 m

\ AB = 10 cm Hint:

Perimeter of rhombus = 4 × side = 4 × 10


= 40 cm
2. Here, PQR is an isosceles right triangle
with PR = QR.

BC2 = AB2 + AC2 = 52 + 122


= 24 + 144 = 169
⇒ BC2 = 132
\ BC = 13 m
Thus, height of the tree = AB + BC
\ PQ2 = PR2 + QR2
= 5m + 13m = 18 m

Subject Enrichment Activities


I. Crossword
Across
1. An angle formed by producing a side of a triangle.
2. Longest side of a right triangle.
3. A line segment joining mid-point of a side to its opposite
vertex.
Down
4. A perpendicular drawn from a vertex of the triangle to its
opposite side.
5. A triangle with two of its angles equal.
II. Activity
bjective: To verify experimentally the angle sum property of a triangle.
O
Previous knowledge: Concept of a triangle, concept of angles, skill of drawing a triangle, angle
sum property of a triangle.
Materials required: Drawing sheet, a pair of scissors, glue, scale, pencil, colours, erasers, etc.
Steps:
1. On a drawing sheet, draw a triangle and cut it out.
C

A B

The Triangle and Its Properties  119


2. Tear it into three parts as shown in Fig. (a).
3. Colour them with different colours and mark angles as shown in Fig. (b).
C

A B

4. Paste them as shown in the figure below. Also write A, O and B as shown in Fig. (c).

5. Put a scale along AB.

Observation: AOB is a straight line or AOB = 180°.


Conclusion: Angle sum property of a triangle is verified.

III. Mutliple Choice Questions


1. In which of the following cases, the angles are not possibly the angles of a triangle?
(a) 90°, 65°, 25° (b) 80°, 40°, 60° (c) 37°, 49°, 84° (d) 39°, 41°, 100°

2. In DPQR
(a) PQ + QR > PR (b) PQ + QR < PR (c) PQ + PR < QR (d) PR + QR < PQ

3. If two angles of a triangle are 60° each, then the triangle is


(a) Isosceles but not equilateral (b) Scalene
(c) Equilateral (d) Right-angled

4. In DXYZ, if XY = YZ and Y = 100°, then Z is equal to


(a) 40° (b) 80° (c) 120° (d) 50°
5. Two trees, 7 m and 4 m high, stand upright on the ground. If their bases are 12 m apart, then the
distance between their tops is
(a) 3 m (b) 5 m (c) 4 m (d) 11 m

120 Mathematics–7
6. In the given figure, a + b equals

(a) 235° (b) 215° (c) 195° (d) 225°


7. Angles of a triangle are in the ratio of 1 : 2 : 3. The smallest angle is:
(a) 15° (b) 90° (c) 60° (d) 30°
8. A 26 m long ladder is placed against the wall in such a way that the foot of the ladder is 10 m
away from the wall. How up on the wall is the upper end of the ladder?
(a) 20 m (b) 24 m (c) 18 m (d) 25 m
9. The top of a broken tree touches the ground at a distance of 15 m from its base. If the tree is
broken at a height of 8 m from the ground then the actual height of the tree is
(a) 25 m (b) 13 m (c) 18 m (d) 17 m
10. In the given right triangle, the value of x is

(a) 55° (b) 110° (c) 70° (d) 27.5°

Answers

I. III. 1. (c) 37°, 49°, 84°


2. (a) PQ + QR > PR
3. (c) equilateral
4. (a) 40°
5. (b) 5 m 6. (b) 215°
7. (d) 90° 8. (b) 24 m
9. (a) 25 m
10. (d) 27.5°
II. Do it yourself.

qqq

The Triangle and Its Properties  121


7 Congruence of Triangles
Topics Covered
7.1 Congruence of Plane Figures, Line Segments 7.2  Criteria of Congruence of Triangles
and Angles

Let’s Remember
• Any flat surface is called a plane.
• Any figure drawn on a plane is called a plane figure (2-dimensional).

  7.1 Congruence of Plane Figures, Line Segments and Angles


• The plane figures P1 and P2 are said to be congruent, if P1, when superimposed over P2, fits
it exactly. The symbol ‘’ is used to indicate ‘congruent to’.

• Two line segments say, AB and CD, are congruent if they have equal lengths. We write this
as AB  CD . A B
C D
• Two angles, say XYZ and PQR, are congruent if their measures are equal. We write
XYZ  PQR.

Congruence of Triangles
• Two triangles are congruent if all the six elements of one triangle (three sides and three
angles) are equal to the corresponding six elements of the other triangle.


Thus, DABC  DDEF, if A = D, B = E, C = F, AB = DE, AC = DF and
BC = EF.

122
• The bisector of the vertical angle of an isosceles triangle bisects the
base at right angles. Thus, in an isosceles DABC, AD is the bisector
of BAC  BAD = CAD, AD  BC, and BD = DC.

• Two congruent figures are always equal in area. But two figures, equal in area need not be congruent,
e.g., the two rectangles shown in the figure have equal areas, but they are not congruent.

Exercise 7.1
I. Very Short Answer Type Questions (1 Mark)
A. Answer the following.
1. Measure each of the following line segments and state which two of them are congruent.
A E

C D

L M F
B
2. Measure each of the angles given below and write which two of them are congruent?

3. Which of the following pair of figures are congruent?

(a) (b)

(c) (d)

4. Give any two real-life examples for congruent shapes.  [NCERT]


5. How can we show that two line segments are congruent?
6. If DABC  DFED under the correspondence ABC ´ FED, write all the corresponding
congruent parts of the triangles.  [NCERT]
B. Fill in the blanks.
1. When we say R = S, we mean that ............... .
2. Two squares are congruent if they have ...............  of equal lengths.

Congruence of Triangles  123


3. If DPQR  DYXZ, then PQ = ..............., QR  = ............... and PR = ............... .
4. ............... is the symbol of congruence.
5. Two objects are said to be congruent if they have same ............... and same ............... .
6. A circle of diameter 8 cm is congruent to a circle of radius ............... .
C. Say True or False.
1. If two triangles have equal perimeter, they are congruent.
2. Two line segments of same length are congruent.
3. If two rectangles have equal areas, then they are congruent.
4. A five-rupee coin and a ten-rupee coin are congruent.
5. Two congruent squares have the same area.
6. Two squares of same area are congruent.
7. If DABC  DFED, then CA  DF.
8. If DXYZ  DSTU and T = 90°, then Y  =  90°.
II.  Short Answer Type Questions–I (2 Marks)
Justify whether True or False.

1. Area of a square PQRS is 16 sq. cm and perimeter of square ABCD is 16 cm. Then
square ABCD is congruent to the square PQRS.
2. Two quadrilaterals having same measures of their corresponding angles are also congruent.
3. In the figure given below, the following line segments and angles are congruent.

(i) PQ  PS (ii) PS  QR (iii) PQ  SR (iv) P  S

III. Short Answer Type Questions–II (3 Marks)


1. If –PQR ≅ –LMN and –LMN = 65°, then find the magnitude
of –PQR.
2. In the figure, m–POQ = m–ROS.
Is m–POR = m–QOS?
Give reasons.
IV. Long Answer Type Questions (4 Marks)
1. In figure, show that AC @ BD if AB @ CD.

Answers and Hints


I. A. 1. Line segments AB and FE are 3. Set of figures shown in (a) are congruent
congruent 4. (i) Two pages of a book;
2. Figures (a) and (b) are congruent. (ii) The shoe laces of your school shoes.

124 Mathematics–7
5. By measuring their lengths. (ii) True because PS = QR
6. AB = FE, ∠A = ∠F; BC = ED, (iii) True because PQ = SR
∠B = ∠E; AC = FD, ∠C = ∠D (iv) True as ∠P = ∠S = 90° each.
B. 1. ∠R and ∠S are congruent
III. 1. –PQR ≅ –LMN (given)
2. sides 3. YX, XZ, YZ
4. ≅ 5. shape, size ⇒ –PQR = –LMN
6. 4 cm ⇒ –PQR = 65° (as –LMN = 65°)
C. 1. False 2. True 3. False 2. Yes; m–POQ = m–ROS (given)
4. False 5. True 6. True ⇒ m–POQ + –QOR
7. True 8. True = m–ROS + m–QOR
II. 1. Tr u e b e c a u s e a r e a o f s q u a r e ⇒ m–POR = m–QOS
PQRS = 16 sq.cm
IV. 1. Given AB @ CD.
⇒ S2 = 16 ⇒ S2 = 42
We know that two line segments are
⇒ S = 4 cm congruent if their lengths are equal.
Perimeter of ABCD = 16 \ Length of AB = length of CD
⇒ 4S = 16 ⇒ S = 4 cm Adding length BC on both sides,
Both squares have same side of length Length of (AB + BC)
4 cm.
= Length of (BC + CD)
2. False because they may have different
i.e., Length of AC = Length of BD
side lengths, i.e., different size as angles
define shape. or AC @ BD.
3. (i) False because PQ ≠ RS
7.2 Criteria of Congruence of Triangles
Conditions for congruence of triangles: Two triangles are congruent if
(i) Three sides of one triangle are respectively equal to the three corresponding sides of the
other triangle. (SSS congruence)

(ii) Two sides and included angle of one triangle is respectively equal to the two corresponding
sides and the included angle of the other triangle. (SAS congruence)

(iii) Two angles and the included side of one triangle are respectively equal to the two
corresponding angles and included side of the other triangle. (ASA congruence)

Congruence of Triangles  125


(iv) Two angles and a side of one triangle are respectively equal to the two corresponding
angles and the corresponding side of the other triangle. (AAS congruence)
(v) Two right triangles are congruent if hypotenuse and one side of right angled triangle are
respectively equal to the hypotenuse and a corresponding side of the other right angled
triangle. (RHS congruence)
(vi) If two triangles are congruent then all their corresponding parts are congruent/equal. The
above fact is generally used to give the reason for drawing inference after showing two
triangles to be congruent. The reason is usually written as ‘CPCTE’ (corresponding parts
of congruent triangles are equal).
(vii) In an isosceles triangle, the angles opposite to the equal sides are equal.
Thus, in DABC, if AB = AC  B = C.
Similarly, in an isosceles triangle the sides, opposite to equal angles, are equal.

Exercise 7.2
I. Very Short Answer Type Questions (1 Mark)
A. Answer the following.
1. If DABC and DXYZ are equilateral triangles and AB = XY, state the congruence condition
under which DABC  DXYZ.
2. Which angle is included between the sides PQ and QR of DPQR?
3. Which side is included between the angles X and Y of DXYZ?
4. Name the two rules by which we can prove that DLOM  DNOP.

B. Fill in the blanks.


1. Among two congruent angles, one has a measure of 110°; the measure of the other angle
is ...............  .
2. If DABC  DDEF, D = 60°, E = 30°, then C = ...............  .

126 Mathematics–7
3. A triangle ABC has each angle of measure 60°. Another triangle PQR also has each angle
of measure 60°. Is it necessary that the two triangles are congruent? ...............
4. If all the three ............... of a triangle are respectively equal to that of the other triangle,
the triangles may not be congruent.
5. DABC  ...............
by ............... Rule

6. DCAB  ...............
by ............... Rule

7. DROS  ...............
by ............... Rule

8. DABC  ...............
by ............... Rule

9. DXYZ  ...............
by ............... Rule

Congruence of Triangles  127


C. Say True or False.
1. All isosceles triangles are congruent. 2. All equilateral triangles are congruent.
3. Two right angled triangles are congruent.

II.  Short Answer Type Questions–I (2 Marks)


1. Are the triangles congruent in each of the following figures?
If ‘yes’ mention the congruence criterion used. If ‘no’ give reasons.

(i) (ii)

(iii) (iv)

2. Look at the figures given below and state which two of the three triangles are congruent to each
other. Specify the condition by which they are congruent.

(i) (ii)

(iii)

III.  Short Answer Type Questions–II (3 Marks)


Q1. to Q6. in each of the following, show that the given pair of triangles is congruent. State
the criterion and give the three matching pairs for each.

128 Mathematics–7
1. 2.

3. 4.

(NCERT)

5. 6.

 [NCERT]
7. You want to show that DART @ DPEN,

(a) If you have to use SSS criterion, then you need to show [NCERT]
(i) AR = (ii) RT = (iii) AT =
(b) If it is given that T = N and you are to use SAS criterion, you need to have

(i) RT = (ii) PN =
(c) If it is given that AT = PN and you are to use ASA criterion, you need to have [NCERT]
(i) A = (ii) T =
8. ABCDE is a regular pentagon. Show that DABC    DAED

Congruence of Triangles  129


IV. Long Answer Type Questions (4 Marks)
1. In the figure alongside, AB || DC and AB = DC
(i) Is BAC = DCA? Why?
(ii) Is DABC  DCDA by SAS congruency condition?
(iii) State the three facts you have used to answer (ii).
2. In the figure given below, PR = RS and PRQ = SRQ. Prove that DPQR  DSQR.

3. In the figure given below, DABC  DDEF such that AB  = DE, AC = DF and BC = EF. Find
x and y on the basis of data provided in the figure.

4. In the figure given alongside, if AB and MN bisect each


other at O and AM and BN are perpendiculars on XY, prove
that Ds OAM and OBN are congruent and hence prove that
AM  = BN.

5. In the given figure, ABC is an isosceles triangle in which AB = AC. Also, D is a point
such that BD = CD. Prove that AD bisects A and D. [NCERT]

6. Lengths of the sides of the triangles are indicated. By applying the SSS congruence rule,
state whether given triangles are congruent. In case of congruent triangles, write the result
in symbolic form.

130 Mathematics–7
7. In the figure given below, BD and CE are altitudes of DABC such that BD  = CE.
 [NCERT]

(i) State the three pairs of equal parts in DCBD and DBCE.
(ii) Is DCBD  DBCE? Why or why not?
(iii) Is DCB = EBC? Why or why not?
8. You have to show that DAMP  DAMQ. In the following table, supply the missing reasons.

Steps Reasons
(i) PM = QM (i)
(ii) PMA = QMA (ii)
(iii) AM = AM (iii)
(iv) DAMP  DAMQ (iv)
9. DABC is an equilateral triangle. C is the mid-point of DE. –DAC and –EBC are equal
and supplementary. Prove that DDAC @ DEBC.

Answers and Hints


I. A. 1. SSS, SAS, ASA  2. ∠Q 9. DRQP by ASA Rule
3. XY 4. (i) SAS; (ii) RHS; (iii) ASA C. 1. False 2. False 3. False
B. 1. 110° 2. 90° 3. No II. 1. (i) DBAC ≅ DCDB (SSS)
4. angles 5. DADC by SAS Rule (ii) DADB ≅ DADC by ASA
6. DACD by ASA Rule (iii) DBAC ≅ DDCA (SAS)
7. DQOP by SSS Rule (iv) DABC ≅ DEDF by RHS
8. DZYX by RHS Rule 2. DDEF ≅ DRPQ by (SAS)

Congruence of Triangles  131


III. 1. DRPQ ≅ DXYZ by SAS ⇒ (x – 5)° = 45°
(i) PR = XY (ii) PQ = YZ ⇒ x = 45 + 5 = 50°
(iii) ∠P = ∠Y = 120° Also, ∠ACB = ∠DFE (By CPCT)
2. DABC ≅ DEFD by ASA
⇒ 25° = y + 5°
(i) ∠A = ∠E (ii) ∠B = ∠F
⇒ y = 25° – 5° = 20°
(iii) AB = EF
y = 20°, x = 50°
3. DABC ≅ DFDE by SSS
4. In DOAM and DOBM
(i) AB = FD (ii) BC = ED
(iii) AC = EF OA = OB 

4. DABC ≅ DEDF by RHS OM = ON 
(i) AC = EF (ii) BC = DF a AB and MN bisect each other
(iii) ∠B = ∠D ∠AMO = ∠BNO = 90° (each)
5. DRPQ ≅ DABC by RHS DOAM ≅ DOBN [by RHS congruence]
(i) RP = BA (ii) RQ = AC ⇒ AM = BN (C.P.C.T.)
(iii) ∠P = ∠B 5. In DABD and DACD,
6. DPQR ≅ DEDF by AAS OR RHS AB = AC (given) and BD = CD (given)
(i) RP = EF (ii) RQ = DE (iii) ∠P = ∠F AD = AD common
7. (a) (i) AR = PE (ii) RT = EN ∆ABD ≅ ∆ACD (by SAS congruence)
(iii) AT = PN ⇒ ∠BAD = ∠CAD and
(b) (i) RT = EN (ii) PN = AT (c) (i) ∠BDA = ∠CDA by C.P.C.T.
∠A = ∠P (ii) ∠T = ∠N
⇒ AD bisects ∠A and ∠D.
AB = AE 
8.  (sides of a regular pentagon) 6. Yes; ∆ABD ≅ ∆ACD by SSS
BC = DE  Q AB = AC (each 3.5 cm)

∠ABC = ∠AED BD = CD (each 2.5 cm)
Each angle of a regular polygon bb_ AD = AD common
bb
2 (n – 2) × 90° bb 7. (i) BD = CE (given)
= bb
n ∠BDC = ∠CEB (each 90° as BD and
b`b
For n = 5, Each angle bb CE are altitudes)
2 (5 – 2) × 90° bb BC = BC (common)
= = 108° bb
5 (ii) Yes, DCBD ≅ DBCE
a
∆ABC ≅ ∆AED by SAS  (by RHS congruence)
IV. 1. (i) Yes; alternate interior angles are
(iii) Yes, ∠DCB = ∠EBC by [C.P.C.T.]
equal; (ii) Yes
(iii) AB = DC; ∠BAC = ∠DCA; 8. (i) Given; (ii) Given; (iii) Common;
(iv) SAS
AC = AC
2. Q PR = SR; QR = QR; 9. ∆ABC is an equilateral triangle.
∠PRQ = ∠SRQ ∴ AC = BC
∆PQR @ ∆SQR ∠DAC = ∠CBE = 90°
By SAS congruency. {Q They are equal and are supplementary}
3. Q DABC @ DDEF DC = CE (Q C is mid-point of DE)
⇒ ∠ABC = ∠DEF (By CPCT) ∴ ∆DAC ≅ ∆EBC (by RHS)

132 Mathematics–7
Subject Enrichment Activities
I. Crossword
Across
1. AAA is not a criterion for ............... of two triangles.
2. If two circles are congruent then their ............... are equal.
3. If two squares are congruent then their ............... are
equal.
Down
4. SAS stands for side ............... side.
5. In RHS congruence condition, H stands for ............... .
6. A line is drawn to bisect the side and angle which is called ............... .
II. Activity
bjective: To verify the congruence criterion SSS (side-side-side)
O
Materials required: A drawing sheet, pencil, scale, compasses, eraser, tracing paper.
Steps:
1. Draw a triangle ABC with AB = 6.4 cm, BC = 8.6 cm and CA = 7 cm.
2. Draw another triangle PQR with PQ = 6.4 cm, QR = 7 cm and RP = 8.6 cm.
3. We have AB = PQ, BC = RP and CA = QR.
4. Make a trace copy of DPQR and place it over DABC.
5. We observe that the vertex P falls on B, Q on A and R on C.
6. Hence DPQR completely covers DABC under the correspondence ABC  QPR, each triangle can be
completely covered by the other.
7. Therefore, DABC  DQPR.
III. Match the columns
Match the corresponding parts given in Column I with congruence relation given in
Column II and congruence rule in Column III.
Column I Column II Column III
(i) AB = PR
BC = PQ (a) DADB  DADC (I) ASA
AC = QR
(ii) AB = AC
BAD = CAD (b) DABC  DPQR (II) SAS
AD = AD
(iii) AB = PR
A = R (c) DABC  DRPQ (III) SSS
B = P

Congruence of Triangles  133


IV. Multiple Choice Questions
1. If DPQR  DXYZ, which of the following is true?
(a) PR = XY (b) PQ = XY (c) QR = YZ (d) both (b) and (c)

2. In the given figure, if DABC is congruent to DDEF, then the value of x is

(a) 35° (b) 100° (c) 80° (d) 65°

3. In the given figure, if PQ = PR and QS = SR, then PSR =

(a) 30° (b) 60° (c) 120° (d) 90°

4. Which congruence condition, if any can be used to prove the given Ds congruent?

(a) RHS (b) ASA (c) SAS (d) SSS

5. In the congruent triangles ABC and XYZ, three equality relations between some corresponding
parts are given below.
AB = XY, B = Y and C = Z. Which congruence condition is used here?
(a) SSS (b) SAS (c) ASA (d) ASA

Answers
I. II. Do it yourself.
III. (i)—(b)—(III) (ii)—(a)—(II)
(iii)—(c)—(I)
IV. 1. (d) both (b) and (c)
2. (c) 80° 3. (d) 90°
4. (c) SAS 5. (c) ASA

qqq

134 Mathematics–7
8 Comparing Quantities
Topics Covered
8.1 Ratio 8.2 Percentage
8.3  Profit and loss

Let’s Remember
• Ratio is a way of comparing numbers by division.
• Ratio can be expressed as a fraction.
• In the ratio a : b the quantities being compared must be the same unit.
• Ratio has no unit.
• The ratio a : b  b : a.
• Four numbers a, b, c, d are said to be in proportion if a : b and c : d are equivalent ratios.
• The method to find the value of one unit first and then to find the value of the required
number of units, is known as unitary method.

8.1  Ratio
• If we multiply or divide both numerator and denominator by the same number, we get an
equivalent ratio.
• If two ratios are equivalent (equal), they are said to form a proportion.
• The first and fourth terms of a proportion are called extremes and the second and third terms
are called means.
• If a : b = c : d    ad = bc.
i.e., Product of extremes = Product of means.
• In the proportion a : b : : c : d, d is called the fourth proportional.
a b
• Three quantities a, b, c are said to be in continued proportion if a : b = b : c or  .
b c
• In a continued proportion a : b : : b : c, b is called the mean proportional between a and
c and c is called the third proportional.

Exercise 8.1
I. Very Short Answer Type Questions (1 Mark)
A. Answer the following.
1. Find the ratio of 500 g to 5 kg. 2. Which is greater 2 : 3 or 4 : 5?
3. Find the ratio between 70 cm and 2 m. 4. Ratio of 75 paise to `6.
B. Fill in the blanks.
1. If 2 : 3 : : x : 24, then x = ................ 2. If 7 : x : : 49 : 42, then x = ................

135
3. If 8 : 12 : : 40 : x, then x = ................ 4. If 25 : 11 : : x : 33, then x = ................
C. Say True or False.
1
(i) If 4 : 9 :: x : 18, then x = 8. (ii) The ratio between 60 cm and 2m is .
4
(iii) Every ratio has specific unit. (iv) Ratio can not be expressed as a fraction.
II.  Short Answer Type Questions–I (2 Marks)
1. In a computer lab there are 3 computers for every 6 students. How many computers will
be needed for 24 students?  [NCERT]
2. Are 30, 40, 45, 60 in proportion? 3. Find the third proportional of 9 and 12.
4. Find the mean proportional between 4 and 16.
5. If 3 kg of rice costs `241.20. What will 50 kg of such a quantity of rice cost?
6. If 10 boys consume 3 kg of rice in a day, how much will be consumed by 15 boys in a
day?
III.  Short Answer Type Questions–II (3 Marks)
1. Find the ratio of the price of a pencil to that of a ball pen, if pencils cost `16 per score
and ball pens cost `8.40 per dozen.
2. Population of Rajasthan = 570 lakhs and population of UP = 1660 lakhs. Area of Rajasthan
= 3 lakh km2 and area of UP = 2  lakh km2.
(i) How many people are there per km2 in both these States?
(ii) Which State is less populated?
3. 144 persons work in an office. 56 of them are men and the remaining are women. Find
the ratio of
(i) the number of men to that of women.
(ii) the number of men to the total number of persons.
(iii) the total number of persons to that of women.
IV. Long Answer Type Questions (4 Marks)
1. The ratio of the number of men to that of women in a party is 5 : 4. If the number of
men is 40, find the number of women in the party.
2. 39 packets of 12 pencils each cost `374.40. Find the cost of 52 packets of 10 pencils
each.
3. A train travels between two stations, A and B, a distance of 216 km in 1½ hrs. It passes
through a station C, 80 minutes after leaving A. What is the distance between B and C?
 [HOTS]
4. The sum of the squares of three numbers which are in the ratio 2 : 3 : 4 is 2900. What
are the numbers? [HOTS]

Answers and Hints


I. A. 1. Q 1kg = 1000 g 2 2 # 5 10
500 g 2. = =
\ Ratio of 500g to 5 kg = 3 3 # 5 15
5 # 1000 g 4 4 # 3 12
1
= = 1:10 = =
10 5 5 # 3 15

136 Mathematics–7
12 10 4 5.  3 kg of rice costs `241.20

 > & is greater
15 15 5
241.20
70 \ 1 kg of rice costs `
3.  1m = 100 cm, = 7:20 3
200
75 3 241.20
4. `1 = 100 paise, = = 1: 8
\ 50 kg of rice costs `
# 50
600 24 3
B. 1. 16 2. 6 3. 60 = `4020.
4. 75 6. Suppose 15 boys consumed x kg of rice.
C. 1. True, 2. False, 3. False So, 10 : 3 : : 15 : x
4. False.
10 15
= ⇒ 10x = 15 × 3
II. 1. Let 24 students need x computers. 3 x
So, 3 : 6 : : x : 24 15 × 3 9
3 x \x=
= kg = 4.5 kg of rice.
⇒ = ⇒ 6x = 3 × 24 10 2
6 24 III. 1. One score pencils means 20 pencils.
3 × 24
 20 pencils cost `16
\x=
= 12 computers
6
16 4
30 3 45 3
\ 1 pencil costs ` =`
2. = and = 20 5
40 4 60 4 Again,  12 ball pens cost `8.40
30 45 8.40 840 7

⇒ =
\ 1 ball pen costs ` = = `
40 60 12 1200 10
Thus, the ratio of the price of a pencil to
⇒ 30, 40, 45, 60 are in proportion. 4
Yes 4 10 2
that of a ball pen = 5 = ×
7 5 7
3. Let the third proportional between 9 and 810
12 be x, so that = = 8 : 7
7
9 : 12 :: 12 : x 2.  3 lakhs km2 area of Rajasthan has
9 12 570 lakhs of population.
⇒ = ⇒ 9x = 12 × 12
12 x 570
\ 1 lakh km2 area of Rajasthan has
12 4 # 12 4 3
\ x = = 16. lakh of population
93 = 190 lakhs of population.
4. Let the mean proportional between 4 Again
and 16 be x.
 2 lakhs km2 of UP has 1660 lakhs of
So,. 4 : x : : x : 16 population.
4 x 1660

⇒ = \ 1 lakh km2 area of UP has
x 16 2
= 830 lakhs of population
⇒ x2 = 4 × 16 = 22 × 42
Thus (i) Rajasthan has population 190 lakhs
⇒ x = 2 × 4 = 8. per square km.

Comparing Quantities  137


While UP has population 830 lakhs per 52 packets of 10 pencils = 52 × 10
square km. = 520 pencils
(ii) Rajasthan is less populated.
 468 pencils cost `374.40

3. Total number of persons = 144 374.40



\ 1 pencil cost = = `0.80
468
The number of men = 56

\ 520 pencils cost = 520 × 0.80 = `416.
The number of women = 144 – 56 = 88 1 3
3. 1
hours = × 60 minutes
(i) Ratio of the number of men to the 2 2
56 7 = 90 minutes
women = = = 7 : 11
88 11 Distance = 216 km
(ii) Ratio of the number of men to the total
Distance 216
56 7 Speed = =
Time 90
number of persons = = = 7 : 18
144 18 \ Distance covered in 80 minutes
(iii) Ratio of total number of persons to
144 18 216 × 80
that of women = = = 18 : 11 = = 192 km
88 11 90
IV. 1. Suppose the number of women in the Thus, the distance between B and C
partly = x = 216 – 192 = 24 km
4. Let the ratio constant be x.
And, it is given that the ratio of the number According to the question,
5
of men to that of women = (2x)2 + (3x)2 + (4x)2 = 2900
4
5 40 ⇒ 4x2 + 9x2 + 16x2 = 2900
Now, = ⇒ 5x = 40 × 4
4 x ⇒ 29x2 = 2900
40 # 4
8
2900
⇒x=
= 32 ⇒ x2 =
= 100 ⇒ x2 = 102
5 29
2.  39 packets of 12 pencils = 39 × 12
⇒ x = 10
= 468 pencils Thus, the numbers are 2x = 2 × 10 = 20,
And, 3x = 3 × 10 = 30,
and 4x = 4 × 10 = 40.
8.2  Percentage
27
• A fraction with denominator 100, such as is called a percent. The numerator tells the
100
percentage. It is 27%.
• Percent means per hundred or for every hundred. The symbol % is used to denote percent,
e.g., 63 marks out of 100 is 63%.
(i) To convert a fraction into a percent, multiply the fraction by 100,
1 1  100 25
e.g.,   25%. or we can write as 1 = 1 ´100% = 25% 
4 4  100 100 4 4 1
(Q % means )
100

138 Mathematics–7
(ii) a ratio into a percent, write it as a fraction and multiply the fraction by 100
19 19
19 : 25 = = × 100% = 76%.  (as % means )
25 25 100
(iii) a decimal into a percent, multiply by 100, i.e., shift the decimal point two places to the
right,
0.045 = 0.045 × 100% = 4.5%, i.e., 0.045 = 4.5%, Similarly, 0.5987 = 59.87%

(iv) a percent into a fraction, drop the percent symbol (%) and divide by 100., e.g.,
60 3
60% =  .
100 5
(v) a percent into a ratio, drop the percent symbol and form a ratio with the number obtained
after dropping the symbol as the first term and 100 as the second term,
48
48% = = 48 :100 = 12 : 25. (reduce to lowest term)
100
(vi) a percent into a decimal, drop the percent symbol and shift the decimal point two places
to the left,
79 15.85
79% = = 0.79, 15.85% = = 0.1585
100 100
Increase Decrease
 Increase % =
× 100%, Decrease % = × 100%
Original value Original value

Exercise 8.2
I. Very Short Answer Type Questions (1 Mark)
A. Answer the following.
1. 5% of a number is 42. Find the number.
2. What percent of 1900 is 114?
3. There are 25 radios, 16 of them are out of order. What percent of radios are out of order?
 [NCERT]
4. If 65% of students in a class have a bicycle, what percent of the students do not have
bicycles?
5. In a city, 30% are females, 40% are males and remaining are children. What percent are
children? [NCERT]
6. Complete the following table.
Fraction Decimal Percent
1
10
0.7
72

Comparing Quantities  139


7. How much is 20% of 400?
8. How much is 33% of 700?
9. What percent of 40 cm is 8 cm?
10. Find the number whose 15% is 270.
11. Ranjana scored 96 marks out of 160. What percentage marks did she get?
12. The price of an article is `500. If there is an increase of 10% in the price, what will be
the new price of the article?
13. A driver’s monthly salary is `10,000. If he spends `8,000, what percent of his income
does he save?
B. Fill in the blanks.
1. 4 : 5 = ............... % 2. 40% of `480 = ...............
3
3. 4 % = ............... : ...............
5
4. A fraction with ............... as 100 is called a percent.
5. Weight of Jayanti decreased from 80 kg to 72 kg. The decrease in weight is ............. %.
6. Out of a total deposit of `3000 in her bank account, Wasima withdrew 30% of the deposit.
New balance in her account is ............... .
C. Say True or False.
3
1. is same as 75%. 2. 0.7 is same as 70%.
4 9
3. 45% when expressed as a fraction is . 4. 55% when expressed as a decimal is 5.5.
25
3
5. 12% expressed as a fraction is .
25
II.  Short Answer Type Questions–I (2 Marks)
1. In an orchard 30% are apple trees out of a total of 360 trees. Find the number of apple
trees and also find the number of other trees.
2. Justify your answer to the following.
2 2
 66 % .
3 3
3. When an improper fraction is converted into percentage then the answer is always more
than 100.
4. 50% of `200 is `100.
5. Out of 300 students of a school 63 go for a picnic. The percentage of students that did
not go for the picnic is 75.
6. 125 cm is 2% of 1 km.
7. A man travelled 60 km by car and 240 km by train. Find what percent of total journey
did he travel by car and what percent by train?  [Life Skills Question]

III.  Short Answer Type Questions-II (3 Marks)


1. Sunita obtained 750 marks out of 800 and Priya obtained 540 marks out of 600. Whose
performance is better?

140 Mathematics–7
2. (i) Chalk contains calcium, carbon and oxygen in the ratio 10 : 3 : 12. Find the percentage
of carbon in chalk.
(ii) If in a stick of chalk, carbon is 3 g, what is the weight of the chalk stick?
3. If 15% of 80 is greater than 25% of a number by 4. Find the number.  [HOTS]
4. Science: The length of King Cobra can reach upto 5.58 m. This is only about 60%
of the largest reticulated python. Find the length of the largest reticulated python.
[Multidisciplinary Question]
5. Geography: Earth’s total land area is about 148428950 km2. The land area of Asia
is about 30% of this total. What is the approximate land area of Asia to the nearest
square km?  [Multidisciplinary Question]
6. The strength of a school is 4000. If 40% of the students are girls then how many boys
are there in the school? Government has started Ladli Yojna for the girl students. Do you
think we should encourage the girls to study?  [Value Based Question]

IV. Long Answer Type Questions (4 Marks)


1. A person donates 10% of his savings to a charitable trust. He distributed the rest among
his three children equally. If the total savings of the person is `75,000, find the amount
donated to the charitable trust. Also, find the amount received by each child.
2. From a basket full of oranges, 20% of the oranges were rotten and 25% of the oranges
were reserved for the guests. The remaining 22 oranges were consumed by the family
members. How many rot and how many were kept for the guests?
3. The population of a village is 8000. Out of these, 80% are literate and of these literate
people, 40% are women. Find the ratio of the number of literate women to the total
population.
4. In an entertainment programme, 250 tickets of `400 and 500 tickets of `100 were sold.
If the entertainment tax is 40% on ticket of `400 and 20% on ticket of `100, find how
much entertainment tax was collected from the programme.
5. In a debate competition, the judges decide that 20 percent of the total marks would be
given for accent and presentation. 60 percent of the rest are reserved for the subject matter
and the rest are for rebuttal. If this means 8 marks for rebuttal, then find the total marks.
Write two benefits of such competition.  [Value Based Question]
6. In an examination, there are three papers each of 100 marks. A candidate obtained
63 marks in first and 85 marks in the second paper. How many marks must the candidate
obtain in the third paper to get an overall of 80 percent marks?  (Life Skills Question)

Answers and Hints

I. A. 1. Let the number be x. x



⇒ # 1900 = 114
So 5% of x = 42 100
5 42 × 100 114 # 100
⇒ × x = 42 ⇒ x = = 840 \x=
= 6%
100 5 1900
2. Let the percent be x. 3. Let x% of radios are out of order.

x
So x% of 1900 = 114 \ x% of 25 = 16 ⇒
× 25 = 16
100

Comparing Quantities  141


16 × 100 13. His savings = `10000 – `8,000

⇒ x =
= 64%
25 = 2,000
4. 65% of students have a bicycle. Now, x% of 10000 = 2,000
⇒ (100 – 65) % i.e., 35% of students do x
⇒ ×10000 = 2000
not have a bicycle. 100
5. Percentage of males and females 2, 000 # 100
\ x = = 20%
= 30% + 40% = 70% 10000

Thus, percentage of children = (100 – 70%) B. 1. 80% 2. ` 192 3. 23 : 500


= 30% 4. denominator 5. 10% 6. ` 2100
6. Fraction Decimal Percent C. 1. True 2. True 3. False
4. False 5. True
1
0.1 10% II. 1. The number of apple trees
10 30
= × 360 = 108
100
7
0.7 70% The number of other trees = 360 – 108
10
= 252.
2 2 2
2. True because = × 100% = 66 %
18 3 3 3
0.72 72%
25
1 3
20 3. True e.g., 1 = × 100% = 150%
2 2
7. 20% of 400 = # 400 = 80
100 50
4. True, × 200 = 100
33 100
8. 33% of 700 = × 700 = 231
100 5. Number of students that did not go for
9. Let x% of 40 cm be 8 cm. the picnic = 300 – 63 = 237.
x 8 2 # 100 \ Percentage of students that did not go
⇒ # 40 = 8 ⇒ x = = 20% 237 # 100
100 40 for the picnic = = 79
300
10. Let the number be x.
15 So, 75% is the wrong answer.
So, x × = 270 2
100 6. False. × 1000 = 20 m = 2000 cm
270 90 # 100 20 100
⇒ x = = 1800 7. Total distance = 60 km + 240 km
15
11. Suppose x% of 160 = 96 = 300 km
x
⇒ × 160 = 96 Now, x% of 300 = 60
100 x
96 # 100 ⇒ × 300 = 60
\ x = = 60% 100
160 60 × 100
12. Increase = 10% of `500 \x= = 20%
300
10 Thus, distance travelled by car = 20%
= × 500 = `50
100 and distance travelled by train
\ New price = `500 + `50 = `550.
= 100 – 20 = 80%

142 Mathematics–7
750 family, educating a family means educating
III. 1. Sunita’s percentage = × 100 the nation.
800
= 93.75% IV. 1. Amount donated to the charitable trust
10
540 = 10% of `75,000 = × `75,000
Priya’s performance = × 100 = 90% 100
600
 93.75% > 90%, so Sunita’s performance = `7500
is better. Rest of savings = `75000 – `7500 = `67500
2. Let the ratio constant be x. Amount received by each child
So, the content of calcium, carbon and ` 67500
= = 22500.
oxygen are 10x, 3x and 12x. 3
Total = 10x + 9x + 12x = 25x 2. Let the number of oranges be x.
(i) Percentage of carbon in chalk Now, = 20% of x + 25% of x + 22
3x 20 25
= × 100 = 12% ⇒ x = ×x + × x + 22
25x 100 100
x x x x
(ii) To find the weight of chalk stick, we ⇒ x = + + 22 ⇒ x – – = 22
have to find 12% of x = 3 5 4 5 4
20x – 4x – 5x
12 ⇒ = 22 ⇒ 20 – 9x = 440
⇒ ×x =3 20
100
3 × 100 11x = 440
\x= = 25g
12 440
\ x = = 40
3. Suppose the number = x 11
20
 15% of 80 –25% of x = 4
\ Rotten oranges = × 40 = 8
15 25 100
⇒ × 80 – ×x =4 25
100 100 Oranges kept for guests = × 40 = 10
25 100
⇒ 12 – 4 = ×x 3. Number of literate population
100
100 80
\ x =8 × = 32 = × 8000 = 6400
25 100
Number of women = 40% of 6400
4. Let the length of the largest reticulated
40
python be x. So, 60 = × 6400 = 2560
60% of x = 5.58 ⇒ × x = 5.58 100
100 Ratio of the number of literate women to
5.58 × 100
⇒x= = 9.3 m 2560 8
60 the total population = = = 8:25
5. Land area of Asia = 30% of earth's 8000 25
total land area 4. Cost of 250 tickets of `400
30 = 250 × `400 = `100000
= × 148428950 = 44528685 sq. km
100 40
6. Number of girls = 40% of 4000 \ Entertainment tax = × `100000
100
40 = `40000
= × 4000 = 1600
100 Cost of 500 tickets of `100 = 500 × `100
\ Number of boys = 4,000 –1600 = 2400 = `50000
Yes, educating a girl means educating a

Comparing Quantities  143


20 20x – 12x

\ Entertainment tax =
× 50000
⇒ 8 =
100 8x 25
= `10000
⇒8=
25
Thus, total entertainment tax 8 × 25
\ x = = 25
= `40000 + `10000 = `50000 8
Two benefits of debate competition:
5. Consider total marks = x
Thus, the marks given for accent and (i) To popularise debate competition,
presentation = 20% of x. (ii) To provide healthy means for accent
20 x improvement.
= ×x= 6. Let the marks obtained by the candidate
100 5
x 5x – x 4x in the third paper be x. So,
Rest of marks = x – = = .
5 5 5
63 85 x
60 12 4 x × 100 + × 100 + × 100
Marks for subject matter = × 100 100 100
100 25 5 80
= × 300
12x 100
=
25 ⇒ 63 + 85 + x = 240 ⇒ 148 + x = 240
4x 12x
Marks of rebuttal = – ⇒ x = 240 – 148 = 92.

5 25
Thus, the candidate must obtain 92% marks.

8.3  Profit and Lost


• Profit = S.P. – C.P. (if S.P. > C.P.), Loss = C.P. – S.P. (if S.P. < C.P.)
• Overhead charges such as transportation, repairs, etc., are included in the cost price.
Profit Profit % # C.P. Profit
• Profit % = # 100 ⇒ Profit = and C.P. = # 100
C.P. 100 Profit %
Loss Loss% # C.P. Loss
• Loss % = # 100 ⇒ Loss = and C.P. = # 100
C.P. 100 Loss %
• The money borrowed from a lender is called the principal.
• The additional money paid by the borrower to the lender after a specified period of time is
called interest.
• The total money paid by the borrower to the lender is called amount.
• Amount = Principal + Interest.
If the interest is calculated on the original money borrowed throughout the period of loan,
i.e., there is no change in the principal, then the interest is called Simple Interest.
P#R#T
• Interest on a given sum can be calculated by using the formula S.I. =
100
100 # S.I. 100 # S.I. 100 # S.I.
T = ,P= ,R=
P#R R#T P#T

144 Mathematics–7
Exercise 8.3
I. Very Short Answer Type Questions (1 Mark)
A. Answer the following.
1. Find the profit if a book costing `160 is sold for `200.
2. Find S.P. if C.P. = `1,000 and Loss = `120.
3. Find the profit or loss percent if an article costing `3,000 is sold for `2,400.
4. Find the interest on `1,000 at 6% p.a. for 1  year.
5. Find the amount on `12,000 at 10% p.a. for 4  years.
6. Why is amount always more than the principal? How much is 33% of 700?
7. If C.P. is `700 and loss is `140, find the loss %.
8. Find simple interest on P = `5,000, R  =  12% p.a. and T = 3 years.
9. Find gain when C.P. = `450, S.P. = `510.
10. Find S.P. when C.P. = `500, Loss = 40.
B. Fill in the blanks.
1. Brijesh bought a music system for `18,000 and sold it at a loss of `1,800. The selling
price of the music system is ............... .
2. Imran buys a table for `1,400 and sells it for `1,600, he earns a profit of ...............% in
the transaction.
3. Khyati bought a pen for `70 and sold it for `63. Her ............... is ...............%.
4. Interest on `4,000 at 10% per annum for a period of 4 years is ............... .
5. Amount obtained by depositing `10,000 at 8% per annum for six months is ............... .
6. Jagat bought 240 oranges at `4 each. He sold 60% of the oranges at `5 each and the
remaining at `3.50 each. His ............... is ...............%.
7. Kalpana purchased a house for `25,29,850 and spent `20,150 on its repair. To make a
profit of 5%, she should sell the house for ` ............... .
8. A fruit seller purchases 20 kg of oranges at `60 per kg, out of these, 5% of the oranges
were found to be rotten. If he sells the remaining oranges at ` 60 per kg, then his ...............
is ............... %.
C. Say True or False
1. If C.P. = `5000, S.P. = `6,500, octroi = `200 and transportation charges = `300 his gain
is `500.
2. If C.P. = `300 and gain = `15, then gain % is 5%.
3. If P = `2000, R = 5% p.a., T = 1 yrs., then amount is `2100.
4. If P = `900, I = `324, R = 12% p.a., then Time = 3 years.
5. If P = `350, A = `455, T = 6 yrs, then R is 10% per annum.
6. The interest on `700 at 5% p.a. for 73 days is `70.
7. By selling a book for `25, a shopkeeper suffers a loss of 10%. The cost price of the book
is `30.
8. Amount received after depositing `400 for a period of 3 years at the rate of 12% p.a. is
`448.
9. Rachna sold a watch for `6640 at a gain of `640. For earning a gain of 10%, she should
have sold the watch for `6,600.

Comparing Quantities  145


10. C.P. = `300 and profit = 12% then profit is `36.
II. Short Answer Type Questions–I (2 Marks)
1. Arun bought a car for `3,50,000. The next year, the price went upto `3,70,000. What was
the percentage of price increase?
2. What rate gives `280 as interest on a sum of `56,000 in 2 years?
3. If Meena pays an interest of `45 for one year at 9% rate p.a. What is the sum she has
borrowed?
4. Mohan bought a calculator for `300 and sold it for `390. Find his gain or loss.
5. A shopkeeper bought a saree for `425 and sold it at a profit of `80. Find the S.P. of the
saree.
6. A vendor bought bananas for `85 and sold for `70. Find his gain or loss.
7. After selling an article for `280, a shopkeeper suffered a loss of `205. Find the cost price
of the article.
8. A sum of `500 is lent for 2 years at the rate of 5% p.a.. Find the interest. Justify your
answer.
III.  Short Answer Type Questions–II (3 Marks)
1. In how many years will `150 double itself at 4% simple interest?
2. At what rate of simple interest will `350 amount to `455 in 6 years?
3. If the simple interest on a certain sum of money for 2 years be one-fifth of the sum, then
find the rate of interest per annum.
4. Anita bought a second hand scooter for `7,500 and spent `500 on its repairs. Then she
sold it to Deepak at a loss of 12%. What is Anita’s loss?
5. A man buys 3 balloons for a rupee and sells them at a 2 for a rupee; find his gain percent.
6. Find the cost price if the selling price of an article is `8 and the loss is 20%.
7. Find the selling price if the cost price is `20.25 and gain is 10%.
8. Amina buys a book for `  275 and sells it at a loss of 15%. How much does she sell it
for?
IV. Long Answer Type Questions (4 Marks)
1. Raghav bought a second-hand car for `57,100. He sold this car for `70,000. Find his net
profit and profit percent.
2. Krishna borrowed `5,000 from Pushpa at 9% per annum. After 4 years Krishna cleared
the account by giving `5,500 and a cycle to Pushpa. Find the cost of the cycle.
3. Shammi sold two washing machines at `12,000 each. On one he gains 25% and on the
other he loses 25%. How much does he gain or lose in the whole transaction?
4. By selling an article for `500, a man gains `25. At what price should he sell the article
to gain 20%?
5. Ashraf donates `5000 to a school, the interest of which is to be used for awarding 10
scholarships of equal value every year. If the donation earns an interest of 11% p.a., find
the value of each scholarship.
6. Radhika borrowed `12000 from her friends. Out of which `4000 were borrowed at 18%
and the remaining at 15% rate of interest per annum. What is the total interest after
3  years?
7. Bhavya earns `50,000 per month and spends 80% of it. Due to pay revision, her monthly
income increases by 20% but due to price rise, she has to spend 20% more. Find her new
savings.

146 Mathematics–7
8. Juhi invested money for a period from May 2004 to April 2006 at the rate of 12% p.a.
If interest received by her is `1620, find the money invested.  [Life Skills Question]

Answers and Hints


I. A 1. C.P of books = `160, SP = `200 = `500 – `40 = `460
 SP > CP 2
B. 1. `  16,200 2. 14 %
\ Profit = SP – CP = `200 – `160 = `40 7
2. CP = `1,000, Loss = `120 3. Loss; 10% 4. `1600 5. `  10400
SP = CP – Loss = `1000 – `120 = `880. 6. Profit; 10% 7. `  2677500 8. Loss; 5%
3. CP = `3,000, SP = `2,400 C. 1. False 2. True 3. True
 CP > SP. Then, there is a loss. 4. True 5. False
Loss = CP – SP = `3,000 –`2,400 = `600 700 × 5 × 73
4. P = `1000, R = 6% p.a. Time = 1 year. 6. False because SI = =7
100 × 365
SI = ?
P×R×T 1000 × 6 × 1 7. False because 10% of 30 = 3.
SI = = = `60
100 100
S.P. = 30 – 3 = 27 not 25.
5. P = `12,000, R = 10% p.a. T = 4 years.
400 × 3 × 12
P×R×T 12, 000 ×10 × 4 8. False. I = = `144
SI = = = `4800 100
100 100
A = `544 not `448
Amount = P + SI
9. True because S.P. = `6640 and
= `12,000 + `4,800 = `16,800.
gain = `640
6. Because amount is the sum of principal C.P. = S.P. – gain = `6,000
and simple interest. gain = 10% of `6,000 = `600 then
33 S.P. = `6,600
33% of 700 = × 700 = `231
100 10. True, C.P. = `300
12
7. CP = `700, loss = `140, loss % = ? Profit = × 300 = `36
loss 100
loss % = ×100
CP II. 1. Increase in price
140
= # 100 = 20%
700 = `3,70,000 – `3,50,000 = `20,000
8. P = `5,000, R = 12% pa, T = 3 years. \ Percentage of price increase
P×R×T 20000 40 5
SI = = × 100 = =5 %
100 3, 50, 000 7 7
5000 × 12 × 3 SI × 100
= = `1800 2. Rate =
100 P×T
9. CP = `450 and S.P. = `510 where SI = 280, P = 56,000, T = 2 years.
 SP > CP, then there is a gain. 280 × 100 1
SI = = = 0.25%
gain = SP – CP = `510 – `450 = `60 56, 000 × 2 4
10. CP = `500 and Loss = 40 3. SI = `45, T = 1 year, R = 9% p.a.
SP = CP – Loss Principal = ?

Comparing Quantities  147


SI × 100 45 × 100 1
Principal = = = `500 5. CP of one balloon = , and
R×T 9 ×1 3
4. CP = `300, SP = `390 1
SP of one balloon =

 SP > CP ⇒ There is a gain. 2
Gain = SP – CP = `390 – `300 = `90 1 1× 2 2
= =
3 3× 2 6
5. CP = `425, profit = `80. SP = ?
1 1× 3 3
SP = CP + Profit = `425 + `80 = `505. = =
2 2×3 6
6. CP = `85, SP = `70
3 2
CP > SP, there is a loss.
 > So, there is a gain.
6 6
Loss = CP – SP = `85 – `70 = `15. Gain = SP – CP
7. SP = `280, Loss = `205, CP = ? 3 2 1
= – =
CP = SP + loss = `280 + `205 = `485. 6 6 6
8. P = `500, T = 2 years, R = 5% p.a. 1
Gain
P×R×T 500 × 5 × 2 Gain% = × 100 = 6 × 100
SI = = = `50 CP 1
100 100
1 3 3
III. 1. P = `150, Amount = `300; = × × 100 = 50%
Rate = 4% p.a. 6 1
SI = Amount – Principal 6. SP = `8, loss% = 20.
= `300 – `150 = `150 Suppose CP of the article = x
SI × 100 150 ×100 Now, x – 20% of x = 8
T = = = 25 years 20 100x – 20x
P×R 150 × 4
⇒ x– × x = 8 ⇒ =8
2. P = `350, A = `455, T = 6 years 100 100
Rate = ? 80x 8 × 100
SI = A – P = `455 – `350 = `105 ⇒ = 8 \ x = = `10
100 80
SI × 100 105 5 × 100 2 Thus, CP = `10
Rate = =
P×T 350 7 × 6 3
7. CP = `20.25, gain % = 10, SP = ?
= 5% p.a.
x
3. Suppose P = x and SI = , SP = CP + gain % of CP
5 10
T = 2 years, R = ? = `20.25 + × 20.25
100
SI × 100 x × 100 = `20.25 + `2.025
R = = = 10%
P×T 5× x × 2
= `22.275 ª 22.28 (approx.)
4. CP = `7,500, overhead charge = `500
8. CP = `275, Loss% = 15. SP = ?
 Actual CP = `7500 + `500 = `8000.
SP = CP – Loss% of CP
Loss % = 12%, loss = ?
15
12 = `275 – × ` 275
Loss = loss% of CP = × 8000 = `960 100
100
= `275 – `41.25 = `233.75

148 Mathematics–7
IV. 1. CP = 57,100, SP = `70,000 Total SP = `12,000 + `12,000 = `24,000
Profit = SP – CP = `70,000 – `57,100  CP > SP = There is a loss in the
transaction.
= `12,900
Loss = CP – SP = `25,600 – `24,000
Profit 12, 900
Profit % = × 100 = × 100 = `1600.
CP 57100
4. Let the CP be x. So,
= 22.59%
x + `25 = `500 ⇒ x = `500 – `25
2. P = `5,000, R = 9% p.a., T = 4 years Thus, CP = `475
SI = ? To gain 20%, his SP = CP + 20% of CP
P×R×T 5000 × 9 × 4
SI = = = `1800 20
100 100 = `475 + × 475 = `475 + `95 = `570
100
Amount = P + SI 5. P = `5000, R = 11%p.a., T = 1 year. SI = ?
= `5,000 + `1,800 = `6,800
Now, `5,500 + cost of cycle = `6,800 P#R#T 5000 × 11× 1
SI = = = `550
⇒ Cost of cycle = `6,800 – `5,500 100 100
= `1300 Number of scholarship = 10.
3. Suppose the CP of two washing machines So, the value of each scholarship.
are x and y respectively. 550
= ` = `55.
Now, x + 25% of x = `12,000 10
25 6. Simple Interest for P = `4000 , R = 18%,
⇒x+
× x = `12,000 T = 3 years.
100
P×R×T 4000 × 18 × 3
100x + 25x SI = = = `2160

⇒ = 12,000 100 100
100
Simple Interest for P = `12,000 – `4,000
125x

⇒ = 12000 = `8,000
100
R = 15%, P = 3 years.
12000 × 100 8000 × 15 × 3
\x=
= `9600.
\ SI = = `3,600
125 100
Again Thus, total interest after 3 years
y – 25% of y = `12000
= `2,160 + `3,600 = `5,760.

25y 7. Earnings = `50,000.


⇒y–
= `12,000
100 Expenditure = 80% of `50,000
75y 80
⇒ = 12,000 = × ` 50, 000 = `40,000
100 100
Savings = `50,000 – `40,000 = `10,000.
` 12000 × 100
\y = = `16,000
75 Increase in income = 50,000 + 20% of
Total CP = x + y = `9,600 + `16,000 `50,000
20
= 25,600 = `50,000 + × 50, 000
100

Comparing Quantities  149


= `50,000 + `10,000 = `60,000 New savings = `60,000 – `48,000 = `12,000
New expenditure 8. Suppose the money invested = P
20 R = 12% p.a. T = 2 years, SI = `1620.
= `40,000 + ` 40, 000
100 SI × 100 1620 × 100
= 40,000 + `8,000 = `48000 P = = = `6,750.
T×R 2 × 12

Subject Enrichment Activities


I. Crossword
Complete the following crossword using the given clues.

Across Down
1. ......................is a way of comparing 6. Total money paid by the borrower to the lender
numbers by division and has no unit. is called ..................... .
2. ............... means per hundred. 7. The method to find the value of one unit first
and then the required number of unit is called
..................... method.
3. The first and fourth terms of a propor- 8. The method to know how many times one
tion are termed as ..................... . quantity is of the other is called ..................... .
4. If two ratios are equal, then they are 9. The additional money paid by a borrower
said to form a ..................... . to the lender after a specified time is called
..................... .
5. Money borrowed from a lender is called 10. The charges paid for transportation, repairs,
..................... . etc. and included in the cost price are called
..................... charge.
II. Activity
Objective: To find the number of diagonals that can be drawn from 1 vertex of a polygon of x sides.
Materials required: Chart paper, sketch pens, scale.

150 Mathematics–7
Procedure:
1. Draw different polygons on chart paper. Say a triangle, a quadrilateral, a pentagon, a hexagon
and an octagon.
2. Draw diagonals from one vertex of each figure.

(i) (ii) (iii)

(iv) (v)

3. Now count the number of non-overlapping triangles in each figure and check from the following
table:
Polygon Number Number of Number of
of sides triangles non-overlapping
triangle
Triangle 3 1 3–2
Quadrilateral 4 2 4–2
Pentagon 5 3 5–2
Hexagon 6 4 6–2
Octagon 8 6 8–2
III. Match the columns
Column I Column II
1. 30% of `360 (i) 30
2. 120% of 50 (ii) 250
3. 15 is .......... % of 50 (iii) 10
4. 2.5 = .......... % (iv) `108
5. S.P. = `440, C.P. = `400 and Profit % is = .......... (v) 60

IV. Multiple Choice Questions


1. 20% of 1400 m is
(a) 1120 m (b) 140 m
(c) 420 m (d) 280 m

Comparing Quantities  151


2. What percent of 3000 is 1500?
1
(a) 200 (b)
2
(c) 2 (d) 50
3. The ratio 7 : 8 is equal to
(a) 8.75% (b) 87.5%
(c) 0.875% (d) 623%
4. A book was purchased for `400 and was sold for `800. Then profit earned is
(a) 100% (b) 16%
(c) 50% (d) 60%
5. A farmer bought a buffalo for `44,000 and a cow for `18,000. He sold the buffalo at loss of 5%
but made a profit of 10% on the cow. The net result of the transaction is
(a) loss of `200 (b) profit of `400
(c) loss of `400 (d) profit of `200
6. The interest on ` 15,000 for 3 years at the rate of 15% p.a. is
(a) `22,500 (b) `4,500
(c) `9,000 (d) `6,750
7. Amount received on ` 6,000 for 2 years at the rate of 11% p.a. is
(a) `4,680 (b) `7320
(c) `8,640 (d) `6,660
8. 45% of x is 630 km, then the value of x is
(a) 1400 km (b) 1300 km
(c) 1620 km (d) 1360 km
9. The sum which will earn a S.I. of `252 in 2 years at 14% p.a. is
(a) `788 (b) `790
(c) `900 (d) `1,080
10. The percent that represents the shaded region in the figure.

(a) 55% (b) 40%


(c) 70% (d) 27.5%

152 Mathematics–7
Answers
I. III. 1—(iv) 2—(v) 3—(i)
4—(ii) 5—(iii)
IV. 1. (d) 280 m 2. (d) 50
3. (b) 87.5% 4. (a) 100%
5. (c) loss of `400
6. (d) `6,750
7. (b) `7320 8. (a) 1400 km
9. (c) `900 10. (b) 40%

qqq

Comparing Quantities  153


9 Rational Numbers
Topics Covered
9.1 Rational Numbers 9.2  Operations on Rational Numbers

Let’s Remember
• Fraction: Fraction represents equal part of a whole thing (or a group).
e.g., may mean 1 part out of 3 equal parts.
or 1 out of 3 i.e.,

9.1 Rational Numbers


p
• A number which can be expressed in the form , where p and q are integers and q  0 is
q
called a rational number.
4 5 -6
• Every integer can be expressed as a rational number, e.g., 4 = ,5 = , -6 = etc.
1 1 1
• Every fraction is a rational number.
0 0 0 0
• 0 is a rational number as it can be written as , , , ,...
1 2 3 4
• Rational numbers other than 0 are either positive or negative.
• When the numerator and denominator, both of a rational number have like signs, i.e., both
positive or both negative, it is a positive rational number.
• When the numerator and denominator have opposite signs, i.e., one positive and one negative,
it is a negative rational number.
• A rational number is said to be in its standard form if its denominator is a positive integer
and the numerator and denominator have no common factor other than 1.
3
• Every rational number can be written in the standard form. e.g., is not in standard form as its
5
3 9
denominator is negative. But it can be written as . Also 15 is not in the standard form as
5
its numerator and denominator have a common factor other than 1, that is 3. It can be written in
3
the standard form as .
5
• Two rational numbers are said to be equal if their standard forms are equal.
• Every rational number can be represented on a number line.
• On a number line all rational numbers to the left of 0 are negative and to the right of 0 are
positive.
• Every rational number has a point corresponding to it on the number line but the converse
is not true.
154
a c a c a
• If and are two rational numbers and + = 0 , then is called the additive inverse
b d b d b
c
or negative of and vice-versa. On the number line, points corresponding to these are equal
d
distance apart from 0 but on the opposite sides of 0.
• There are countless rational numbers between any two rational numbers.
• If the numerator and denominator of a given rational number are multiplied or divided by the
same non-zero integer, then the new rational number thus formed is said to be equivalent
to the given rational number.

Exercise 9.1
I. Very Short Answer Type Questions (1 Mark)
A. Answer the following.
4
1. Write the rational number with a positive denominator.
7
2. Define a rational number.
3. What is the standard form of a rational number? Explain.
4. How many rational numbers are there between any two rationals?
B. Fill in the boxes with ‘>’, ‘<’ or ‘=’.
5 2 4 5
1. 2.
7 3 5 7
7 14 8 7
3. 4.
8 16 5 4
C. Which of the following pairs represents the same rational number.
7 3  16 20
1. and 2. and
21 9 20  25
2 2 1 1
3. and 4. and
3 3 3 9
5 5
5. and
9 9
D. Fill in the blanks.
1. There are infinite number of ............... between any two rational numbers.
2. The rational number 0 is neither ............... nor ............... .
c
3. In any rational number , denominator cannot be ............... .
5 d
4. is a ............... rational number.
7
15
5. The standard form of = ............... .
25
9 x
6. If  , then the value of x is ............
10 50

Rational Numbers  155


E. Say True or False.
7 7
1. Zero is a rational number. 2. is less than .
9 9
5
3. On a number line, is to the right of zero (0).
4
18 9 3 3
4. The standard form of is . 5. Additive inverse of is .
24 −12 5 5
II.  Short Answer Type Questions–I (2 Marks)
A. Answer the following.
25
1. Express as a rational number with numerator –50.
17
2. Express 21 as a rational number with denominator –28.
7
49
3. Express as a rational number with denominator 20.
140
 5  10 y
4. Find the values of x and y if  
8 x 40
21
5. Express in the standard form.
 35
7
6. Find the reciprocal of  .
15
8 19
7. A city received cm rainfall on Saturday and cm on Sunday. How much rainfall
15 3
did it receive in two days? [Multidisciplinary Question]
B. Say True or False. Justify your answer.
1. All whole numbers are rational numbers.
7 4
2. is greater than .
4 3
III. Short Answer Type Questions–II (3 Marks)
1. List any three rational numbers between –2 and –1.
2. Write four more rational numbers in each of the following patterns.
3 6 9 2 4 6
(a) , , , , , , (b) , , , , , ,
5 10 15 3 6 9
3
3. Draw the number line and represent on it.
4
4. The points P, Q, R, S, T, U, A and B on the number line are such that, TR = RS = SU
and AP = PQ = QB. Name the rational numbers represented by points P, Q, R and S.

156 Mathematics–7
5. Write the following rational numbers in ascending order.
1  2  4
, , .
3 9 3
3 3
6. A dragonfly flies from a point A, km towards East and 2 km towards West. At what
5 4
distance and in which direction will it be now from the point A?
 [Multidisciplinary Question]
IV. Long Answer Type Questions (4 Marks)

1. Represent 3 2 on the number line.


5
2. In the following rational numbers, a is a positive integer. Order these numbers from least
to greatest.  [HOTS]
a a a a a a
a, , , , , , .
5 8 8 6 2 4
p
3. If a rational number q < 1, where p and q are both positive integers, then which of the
following is greater than 1?  [HOTS]
p p
(a) (b) 2 (c) q (d) p – q
3q q p

Answers and Hints


−4 E. 1. True 2. True 3. True
I. A. 1. 4. False 5. True
7
2. A number which can be expressed in −25 −25 × 2 −50
p II. A 1. = =
the form where p and q are integers 17 17 × 2 34
q
21 21 × 4 84
and q ≠ 0 is called a rational number. 2. = =
3. A rational number is said to be in its −7 −7 × 4 −28
standard form if its denominator is a −49 −49 ÷ 7 −7
3. = =
positive integer and the numerator and 140 140 ÷ 7 20
denominator have no common factor 4. x = –16, y = 25
other than 1. −5 −10 25
4. Infinite Hint: = =
−8 −16 40
B. 1.  < 2. < 3. =
21 21 ÷ 7 −3
4. > 5. = =
C. 1. No 2. Yes 3. No −35 −35 ÷ 7 5
4. No 5. Yes 7 7
6. –
=–
D. 1. rational numbers 15 15
2. positive, negative −7 1 −15
= =
3. zero 4. negative −7
3 Reciprocal of 15 7
5. 6. –45 15
5

Rational Numbers  157


8 19 8 + 19 103 13 1 7
7. + = = =6 cm 4. Point P represents 2 = ,
15 3 15 15 15 3 3
B. 1. True; as each whole number can be 2 8
p Point Q represents 2 =
expressed in the form 3 3
q
2. False. 1 −4
Point R represents −1 =
–7 –21 + 4 –4 –16 3 3
As = and = =
4 12 –3 3 12 2 −5
Point S represents −1 =
−21 −16 3 3
Now, < –4 –1 –2
12 12 5. < <
3 3 9
−7 −3 −5
III. 1. , , 3
4 2 4 6. 2 m to the west of point A.
20
−12 −15 −18 −21 2 2
2. (a) , , , IV. 1. –3 =3
20 25 30 35 5 5

8 10 12 14
(b) , , , 0 1 2 3 4
−12 −15 −18 −21 2 17
3 =
2 2 5 5
3. –3 = 3 −a −a −a a a a
5 5 2. < < < < < <a
4 6 8 8 5 2
q
0 3 1 3. (c)
4 p

9.2 Operations on Rational Numbers


• Sum of two or more rational numbers with the same denominator can be obtained by adding
their numerators, keeping the denominator same.
• Sum of two or more rational numbers with different denominators can be obtained by first
taking the LCM of the denominators and then converting the rational numbers to their
equivalent forms having their LCM as the denominator.
• For subtracting one rational number from the other, we add the additive inverse (i.e., negative)
of the rational number to be subtracted to the other rational number.
• To find the product of two rational numbers, we separately multiply their numerators and
denominators.
Product of numerators
i.e., Product 
Product of denominators
a c
e.g., if and are any two rational numbers, then
b d
a c a c a#c
product of and = # =
b d b d b#d
a c a c a c
• If and are two rational numbers and   1, then is called the reciprocal of
b d b d b d
and vice-versa.
• To divide a rational number by another non-zero rational number, we find the product of the
first rational number with the reciprocal of the second rational number.

158 Mathematics–7
p r r p r p s
e.g., if and are two rational numbers and  0, then    .
q s s q s q r
Exercise 9.2
I. Very Short Answer Type Questions (1 Mark)
A. Answer the following.
–5
1. Find the additive inverse of the rational number
9
11 7 –2 3
2. Add – and 3. Subtract from
5 5 7 4
11 –1
4. Multiply by 5. Write reciprocal of –2.
13 4
5 6
6. Simplify: '
9 7
7. Is the whole number 0 a rational number? Give reasons.
B. Fill in the blanks.
4 5 3
1. Additive inverse of is ............... . 2.  = ............... .
5 7 7
3. The reciprocal of –1 is ............... .
II.  Short Answer Type Questions–I (2 Marks)
A. Answer the following.
2 3 5 4
1. Find  . 2. Find the sum of and .
7 7 7 7
3 8
3. Find the difference of and .
11 11
46
4. Express the rational number as the sum of an integer and a rational number.
9
5 8 7 5
5. Evaluate:  . 6. Multiply by .
11 11 10 8
1 4
7. Find the reciprocal of 1  . 8. Divide 8 by .
8 9
9. Express the rational number –129.6 in the simplest form.

B. Express the given rational numbers as a decimal.


9 1 64
1. 2. 3.
40 8 80
C. Say True of False. Justify your answer.
1 2
1. – and are additive inverse of each other.
5 10
2. Sign of product of a negative rational number and a positive rational number may be
negative or may be positive.

Rational Numbers  159


III.  Short Answer Type Questions–II (3 Marks)
8 1
1. The sum of two rational numbers is . If one of the numbers is , find the other.
7 14
35 5
2. The product of two rational numbers is . If one of the numbers is , find the other.
18 12
55 22
3. By what number should  be divided to get  ?
18 9
1
4. From a rope 68 m long, pieces of equal size are cut. If length of one piece is 4 m , find
4
the number of such pieces.
3  4 
5. Find the value of  .
13  65 
7 28
6. By what rational number should we multiply to get the product as ?
11 33
7 5
7. What number should be subtracted from to get ?
8 12
2 5
8. Satpal walks km from a place P towards East and then from there 1 km towards
3 7
West. Where will he be now from P?
3 5
9. Kuber spends hours in doing homework, and his sister Manavi spends hours in
8 8
doing homework. Who spends more time in doing homework and how much more time?
Do you think doing homework sincerely helps?  [Value Based Question]
3 1 1
10. Pooja spent of her pocket money. If she spent of it for a video game, for
4 3 6
stationery and the rest for a dress, what part of her pocket money did she spend on the
dress?  [Life Skills]
IV.  Long Answer Type Questions (4 Marks)
1. Simplify and express the result in the form of a rational number:
 3 2   6 1   1 9   2 
         1     7   21
11 9 21 42 3 24
4
2. By taking x = 3 and y = , find out whether (x + y)–1 = x–1 + y–1 is true or false.
9
–5 72 11 34 28 –52
3. Simplify: d # n–d # n+d # n
9 –125 17 55 –13 21
 1 1 1
4. What should be added to     to get 4?
 2 3 5

5. Simplify: 5  7  11  1  0  7
6 8 12 6 16

160 Mathematics–7
 1 3  8 7
6. What should be added to    of  so that the sum is the product of and
 2 4 15 50
1
1 ?  [HOTS]
14

Answers and Hints

–5 –5 5
=–d n=
−648
I. A. 1. Additive inverse of 8. –18 9.
9 9 9 5
–11 7 –4 B. 1. –0.225 2. 0.125
2. + =
5 5 5
3. 0.8
3 –2 3 2 21 + 8 29
–d n =
C. 1. True
3. + = =
4 7 4 7 28 28 2. False, it is always negative.

11 –1 11 # (–1) –11 8 1 16 − 1 15
4. #d n = = III. 1. Other number = − = =
13 4 13 # 4 52 7 4 14 14
1 –1 −35 5
5. Reciprocal of –2 = = 2. Other number =
÷
–2 2 18 12
5 6 5 7 35 −35 12 −14
6. ' = # = = × =
9 7 9 6 54 18 5 3
7.  Yes because 0 can be written as 3. Let the number be x.
0 0 0 −55 −22 −55 1 −22
, , , etc. which are rationals. So, ÷x= ⇒ × =
1 2 3 18 9 18 x 9
4 8
B. 1. − 2. − 3. –1 −55 9 5
5 7 \ x = × = .
−2 3 −2 + 3 1 18 −22 4
II. A. 1. + = =
7 7 7 7 1 17 4
= 68 ÷ = 68 × =  4 × 4 =  16
4. 68  ÷  4
−5 −4 −5 − 4 −9 4 4 17
2. + = =
7 7 7 7 3  −4  3 65 −15
−3 8 −3 − 8 −11 5. ÷  = × =
3. − = = =–1 13  65  13 −4 4
11 11 11 11
28 −7
 1 8 6. Rational number =
÷
4. −5 +  −  or − 6 + 33 11
 9 9 28 11 −4
−5 8 −5 8 −5 + 8 3 = × =
5. − = + = = 33 −7 3
11 −11 11 11 11 11
−7 5 −21 − 10 −31
−7 5 −35 −7 7. − = =
6. × = = 8 12 24 24
10 8 80 16
1 1 1 8 −1 7 12 2 36 − 14 22
7. 1 − = − = =
8. − = = km towards West
8 1 8 8 8 7 3 21 21
7 1 8 from P.
\ Reciprocal of = 7 = 5 3
8 7
9. Q > , So, Manavi spends more
8 8 8

Rational Numbers  161


5 3 2 1 P
 ooja spends three-eighth of her pocket
time – = = hrs, i.e., 15 money on the dress.
8 8 8 4
minutes.
433 37
1 3 1 IV. 1. =6
10. Expenditure on video game = × = 66 66
3 4 4
9
2. False because ( x + y ) =
−1
1 3 1   
Expenditure on stationary = × = 31
6 4 8 31
1 1 2 +1 3 or  x −1 + y −1 =
Total expenditure = + = = 12
4 8 8 8 19 89 29 11
3 3 3. 5 4. =2 5.
\ Expenditure on dress = − 75 30 30 16
4 8 1
6−3 3 6. −

= = 20
8 8

Subject Enrichment Activities


I. Crossword
Complete the following crossword using the given clues.
5
6
3 4

1
2

Across
2 2
1. and are .......... inverse of each other.
3 3
2. Every fraction is a .......... number.
3. Product of two negative rational numbers is always a .......... rational number.
Down
4
4. Point representing on the number line lies to the .......... of zero.
5
7 9
5. is the .......... of .
9 7
6. A rational number with a positive denominator and HCF of its numerator and denominator
1 is said to be in its .......... form.

162 Mathematics–7
II. Match the columns
Column I Column II

6 8 33
(i) (a) is the product of and
 31 11 40

3 38
(ii) (b) is the multiplicative inverse of
5 22

5 3
(iii) (c)
8 7

 11
(iv) (d) is a positive rational number
19

3 (e) lies between –1 and 1


(v)  0 is
7

III. Multiple Choice Questions


4
1. When  is written as a rational number with numerator 16, then the denominator is
8
(a) 32 (b) –32 (c) –64 (d) 64
2. Which of the following can be written as a rational number with denominator 5?
9 16 36
(a) (b) (c) (d) 1
10 20  40 15
5 6
3.  is not equal to
6 7
1 6   5  6  5  5  6 
(a) (b)   (c)    (d)  
42 7  6 7  7  6  7
4. To rewrite a rational number in its standard form, we divide its numerator and denominator by
their
(a) Product (b) LCM (c) Common factor (d) HCF
5. The denominator of a rational number in its standard form is always a
(a) negative (b) positive (c) 1 (d) 0
7
6. Which of the following is equivalent to ?
8
8 64 21
(a) (b) (c) (d) None of these
7 49 64
7. Which is the smallest rational number in the following?
−1
(a) (b) 0 (c) (d) –3
3

Rational Numbers  163


8. Which of the following rational numbers is not positive?
 5  7 11
(a)    (b) (c) (d) 4
9 9 9 7
1
9. The reciprocal of is
5
(a) 6 (b) 5 (c) – 1 (d) 0

Answers
I. 5 II. (i)—(d), (ii)—(a), (iii)—(e),
R
6
(iv)—(b) , (v)—(c)
3 P O S I T I V E 4
16
T C L III. 1. (b) –32 2. (b)
1 A D D I T I V E 20
−6  −5 
2 R A T I O N A L P F 3. (b) +   4. (d) HCF
D R T 7  6
A O 5. (b) positive 6. (d) None of these
R C 4
7. (d) –3 8. (d)
D A −7
L 9. (b) 5
qqq

164 Mathematics–7
10 Practical Geometry
Topics Covered
10.1 Construction of Lines 10.2  Construction of Triangles

Let’s Remember
• A ruler is a scaled straight edge used to draw and measure the line segments.
• Set squares are used to construct parallel and perpendicular lines.
• A protractor is a semi-circular disc used to draw and measure angles.
• Two lines are said to be perpendicular if one of the angles formed by them is right angle.
• Bisector of an angle is its axis of symmetry.
• One and only one perpendicular can be drawn to a line from a point (i) on the line and
(ii) outside the line.

10.1 Construction of Lines
• We can construct a line parallel to a given line through a given point not on it by using the
alternate interior angles property (i.e., if a pair of alternate interior angles are equal, then
lines are parallel) or corresponding angles property (i.e., if a pair of corresponding angles
are equal, then lines are parallel).
• Using the above two properties we can construct a line parallel to the given line at a given
distance from it.
Construction of Parallel Lines Using Ruler and Set Squares
Example: Draw any line AB and mark a point P outside it. Draw the line through P parallel to
AB using a ruler and set square.
Solution. Steps of construction
1. Draw any line AB. 2. Mark any point P outside AB.
3. Place either of the set squares with one arm of the right angle along AB.
4. Hold the set square firmly and place a ruler or the other set square along the other arm of the
right angle as shown in figure (i) shown below.

165
5. Holding the ruler firmly slide the set square along the ruler until the point P is on the first arm
of the right angle.
6. Now holding the set square firmly in this position draw the line PQ through P along the edges
as shown in figure (ii)
7. Remove the set square.
Then PQ is the required line through the point P.
Construction of Parallel Lines Using Ruler and Compass
Example: Draw any line AB and a point P outside AB. Draw a line through P and parallel to AB
using ruler and compass.
Solution. Steps of construction
1. Draw any line AB.
2. Mark any point P outside AB [Figure (i)].
3. From P draw a line segment PC [Figure (ii)].
4. At P, draw –RPC = –PCB [Figure (iii)].
5. Produce RP to Q.
Then RQ is the required line through P parallel to AB [Figure (iv)].

Construction of a Line Parallel to a given Line at a given Distance


xample: Draw any line AB and draw a line CD || AB at a distane of 2.5 cm from AB.
E
Solution. Steps of construction
1. Draw any line AB and at A draw AX ^ AB [Figure (i) on the next page].

166 Mathematics–7
2. From AX, cut off AC = 2.5 cm [Figure (ii)].
3. At C, draw CD ^ AX [Figure (iii)].
Thus, CD || AB is the required line.

X X X

C D
C

A B A B A B
(i) (ii) (iii)

Exercise 10.1
I. Very Short Answer Type Questions (1 Mark)
A. Answer the following.
1. How many lines can be drawn parallel to a give line?
2. How many lines can be drawn parallel to a given line at a given distance from it?
B. Say True or False.
1. We can construct only two lines parallel to a given line at a given distance from it.
2. If two parallel lines are cut by a transversal, then the alternate interior angles are equal.

II.  Short Answer Type Questions–II (3 Marks)


1. Draw a line l. Draw a perpendicular to l at any point on l. On this perpendicular choose
a point X, 4 cm away from l. Through X, draw a line m parallel to l.
2. Draw a line AB. Draw another line parallel to AB at a distance of 2.5 cm from AB. How
many such lines can be drawn?
III.  Long Answer Type Questions (4 Marks)
1. Draw any DPQR using a compass and scale. Through P, draw a line l parallel to QR.
2. Draw any triangle PQR. Let S be the midpoint of side PQ. Using a ruler and compass
draw the line through S parallel to QR to meet PR at T. Measure ST, PT and TR. Is T a
1
midpoint of side PR? Is the measure of ST = QR?  [HOTS]
2
3. Draw an equilateral triangle ABC of any side length. Now, through the vertex A
draw a line l  ||  BC, m || AC through B and n || AB through C to form a DDEF. Is DDEF
also equilateral? Now measure the side of DDEF. Is it twice AB? [HOTS]

Practical Geometry  167


Answers and Hints
I. A 1. only one 2. Two
B. 1. True 2. True
III. 1.
II . 1.
X
m

P
P
2
T S T
PX = 4 cm S
l || m 2.
Q
Rough diagram
R 1
Q R

l ST || QR (Q ∠1 = ∠2)
P
Yes, PT = TR.
1
Yes, ST = QR
2
2.
D A E
l
D A E l
4 cm 4 cm
P D Q
3.
B 4 cm C
B C

AB || PQ n Fm F
CD = 2.5 cm Rough diagram n m

DE = EF = DF = 8 cm
∴ DDEF is an equilateral triangle.
A C B
Yes, sides of DDEF are twice the sides of DABC.
10.2 Construction of Triangles
• A triangle has 6 parts, i.e., 3 sides and 3 angles.
• If two sides of a triangle are unequal, the angle opposite to the longer side is greater.
• To construct a triangle we should know at least three of its elements.
Note: It is always advisiable to draw a rough sketch before drawing actual figure.
Case 1. (SSS Triangle Construction) — To Construct a Triangle when the Lengths of its
three Sides are given
Example: Construct a DABC in which AB = 2 cm,
BC = 4.5 cm and AC = 3.5 cm.
Solution. Steps of construction.
1. Draw a rough sketch and mark the given measures.
2. Draw BC = 4.5 cm [Figure (i)].
3. With B as centre and radius 2 cm draw an arc [Figure (ii)].
Rough Sketch

168 Mathematics–7
4. With C as centre and radius 3.5 cm draw another arc cutting the previous arc at point A [Figure (iii)].
5. Join A to B and A to C [Figure (iv)].

Then DABC is the required triangle.


Note: Sum of any two sides of a triangle must be greater than the third side

Exercise 10.2
I. Very Short Answer Type Questions (1 Mark)
A. Answer the following.
1. Can you construct a triangle with sides measuring 8 cm, 5 cm, 3 cm?
2. In the triangle ABC sides in descending order are AB > BC > AC. Which is the largest
angle?
3. In the given figure, name the shortest side.

B. Fill in the blanks.


1. A triangle can be constructed if any of its three ............... are known.
2. A triangle ABC can be constructed with sides BC = 5.2 cm and CA = 4.2 cm and side
AB. Length of side AB should be less than ............... cm but greater than ............... cm.
C. Say True or False.
1. It is possible to construct a triangle with sides 7 cm, 5 cm and 3 cm, respectively.
2. To construct a triangle we need all the six elements, i.e., three sides and three angles.
II. Short Answer Type Questions–I (2 Marks)
1. Construct an equilateral triangle of side 5.5  cm.
2. Draw DPQR with PQ = 4 cm, QR = 3.5  cm and PR = 4 cm. What type of triangle is  this?
3. Construct a DPQR in which PQ = 4 cm, QR = 3 cm and PR = 5 cm. Measure –Q. What
type of triangle is it?
4. Construct an equilateral triangle in which each side is 6 cm. Measure each angle.

Practical Geometry  169


Answers and Hints
I. A. 1. No 2. ∠C 3. AB 3.
B. 1. parts but not three angles
2.   9.4 cm, 1 cm
C. 1. True 2. False
II. 1.

5.5
cm

cm
5.5

∠Q = 90°, right-angled triangle.

cm
5.5

5.5
cm

5.5 cm
Rough diagram 5.5 cm
4.

2.

6c
m

6c
4c

3.5
4c

cm

m
m

c
4

3.5 cm
Rough diagram 4 cm
6 cm
Each angle = 60°
Case 2. (SAS Triangle Construction) — To Construct a Triangle when the Lengths of two

Sides and the measure of included Angle are given
Example.
Construct a DABC, if AB = 5 cm, –B = 60° and
BC = 6 cm
Solution. Steps of construction
1. First we draw a rough sketch. It will help us to draw the
actual figure.
2. Draw BC = 6 cm and at B draw a ray BD making an angle
of 60° with BC [Figure (i)].
3. With B as centre and radius 5 cm, draw an arc cutting BD
at A [Figure (ii)].
4. Join A and C [Figure (iii)].

D
A
D
m
5c

60° 60°
B C B C
6 cm 6 cm
(i) (ii)

170 Mathematics–7
Thus, DABC is the required triangle.

Exercise 10.3
I. Very Short Answer Type Questions (1 Mark)
A. Fill in the blanks.
1. A triangle can be constructed when its two ............... and an included angle are given.
2. A triangle PQR can be constructed such that PQ  =  4 cm, QR = 5 cm, PQR = 120°
using ............... triangle construction.
B. Say True or False.
1. We can construct a triangle with equal sides measuring 5 cm each and angle between
them as 60°.
II. Short Answer Type Questions–II (3 Marks)
1. Construct an isosceles triangle in which the lengths of each of its equal sides is 6.5 cm
and the angle between them is 110°.
2. Construct DDEF such that DE = 5 cm, DF  =  3 cm and mÐEDF = 90°.
3. Construct DABC with BC = 7.5 cm, AC  =  5  cm and mÐC = 60°.
4. Construct a triangle ABC in which AB = 5.2 cm, BC = 5.2 cm ÐB = 30°. Measure angle
C. What type of triangle, according to sides is this?
5. Construct a DPQR having PQ = 4.8 cm, QR = 5.2 cm, and Q = Ð40°. How many triangles
are possible with a common base.

Answers and Hints


I. A. 1. sides 2. SAS II. 1.
B. 1. True

Practical Geometry  171


2.
4. C
C
5.2 cm 5.2 cm
F
D

3 cm
3 30°
c m cm
5 90° A 5.2 cm B 30°
E D Rough diagram A 5.2 cm B
Rough diagram D 5 cm E
3. ∠C = 75°, isosceles triangle.
R
5. R
5.2 5.2
cm cm
40°
P 4.8 cm Q 40°
Rough Diagram P 4.8 cm
Q

Many triangles can be constructed with a


common base.
Case 3. (ASA Triangle Construction) — To Construct a Triangle when the Measures of its
two Angles and the length of included Side are given
Example: Construct a DPQR, in which PQ = 4 cm,
R
–P = 60° and –R = 50°.
Solution. Steps of construction
1. Draw a rough sketch and mark the given measures.
2. Draw PQ = 4 cm [Figure (i)]. 60° 50°
3. At P draw –APQ = 60° [Figure (ii)]. P
4 cm
Q

4. At Q draw –RQP = 50° [Figure (iii)]. Rough Diagram

S A
A R

60° 50°
60° P Q
P Q P Q 4 cm
4 cm 4 cm
(i) (ii) (iii)
5. Let PA and QS intersect each other at R, then DPQR is the required triangle.

Exercise 10.4
I. Very Short Answer Type Questions (1 Mark)
A. Say True or False.
1. DPQR with PQ = 6 cm, P = 60°, Q = 45° can be constructed using ASA criterion.
2. It is possible to construct a triangle DEF with EF  = 4.2 cm, E = 110° and F = 80°.
3. We can draw unique triangle whose angles are 75°, 25° and 80°.

172 Mathematics–7
II. Short Answer Type Questions–II (3 Marks)
1. Construct DPQR if PQ = 5 cm, mÐPQR = 105° and mÐQRP = 40°.
(Hint: Recall angle-sum property of a triangle.)
2. Examine whether you can construct DDEF such that EF = 6.2 cm, mÐE = 120° and
mÐF = 80°. Justify your answer.
III. Long Answer Type Questions (4 Marks)
1. Construct a triangle XYZ with Y = 80°, Z = 55° and YZ = 5.6 cm. Draw the
perpendicular bisector of side YZ.
2. Construct DFGH in which FG = 7.4 cm, F = 45°, G = 68°. Construct bisector of G.
3. Construct DPQR with QR = 5.8 cm, Q = 100°, P = 40°. Draw the perpendicular from Q
to PR.
4. Construct DPQR in which QR = 6.2 cm, Q = 45°, P = 35°. Measure R. What kind of
triangle is this?
5. Is it possible to construct DPQR in which PQ = 6 cm, P = 105° and Q = 80°? If not,
why?

Answers and Hints


I. A. 1. True 2. False 3. False
P R
II. 1.

m
5c

105° 105°
40°
Q R P 35°
Rough diagram 5 cm Q

Hint: ∠P = 180° – (105° + 40­°) [ASP]


= 180° – 145° = 35°
2. No. Triangle with two angles, 120° and
80° cannot be constructed as sum of
angles is more than 180°.
III. 1. 2.

Practical Geometry  173


H P
4.
35°

X
45°
68° 45° Q R
G F 6.2 cm
7.4 cm Rough diagarm
3. P

100°
45°
Q 6.2 cm R

–R = 180° – (45° + 35°)


= 180° – 80° = 100°
DPQR is an obtuse-angled triangle.
5. No, the sum of angles i.e., –P + –Q
= 185° and we know that the sum of
Hint: ∠R = 180° – (100° + 40°)  [ASP] angles of a triangle cannot be more
= 180° – 140° = 40° than 180°.

Case 4. (RHS Triangle Construction) — To Construct a Right-angled Triangle whose Length


of Hypotenuse and Length of one Side are given
Example.
Construct a right-angled triangle DEF having
given hypotenuse DF = 4.5 cm and side
DE = 3.5 cm
Solution: Steps of construction
Since DF is the hypotenuse, so –E = 90°.
1. Draw a rough sketch.
2. Draw DE = 3.5 cm and at E draw –DEP = 90° [Figure (i)].
3. With D as centre and radius 4.5 cm, draw an arc meeting EP at F
[Figure (ii)].
4. Join DF [Figure (iii)].

174 Mathematics–7
Thus, DDEF is the required triangle.

Exercise 10.5
I. Very Short Answer Type Questions (1 Mark)
A. Fill in the blank.
1. For constructing a right-angled triangle, its hypotenuse and one ............... should be known.

II. Short Answer Type Questions–II (3 Marks)


1. Construct a right-angled triangle whose hypotenuse is 6 cm long and one of the legs is
4 cm long.
2. Construct an isosceles right-angled triangle ABC, where mÐACB = 90° and AC = 6 cm.
3. Construct the right-angled DPQR, where mÐQ = 90°, QR = 8 cm and PR = 10 cm.
4. Construct DDEF right-angled at E in which DE = 5 cm and EF = 7 cm.
5. Construct a right-angled triangle in which base is 7.5 cm and hypotenuse makes an angle
of 30° with the base.
6. Construct a right-angled triangle ABC, in which AB = 3 cm, BC = 4.5 cm and ÐB = 90°.

Answers and Hints


I. A. 1. side/angle ∆ABC is an isosceles triangle with
II. 1. A
A AC = BC = 6 cm.
6 cm 6 cm 3.

P
B 4 cm C P
Rough diagram B 4 cm C
10 cm
A A 10 cm
2.
6 cm

90°
6 cm

90°
Q 8 cm R Q 8 cm R
C B Rough diagram
6 cm B 6 cm C
Rough diagram

Practical Geometry  175


4 . D D 6.
A A

5 cm

3 cm
5 cm

3 cm
E C B 4.5 cm C
7 cm
Rough diagram E 7 cm F Rough diagram B 4.5 cm C
5. qqq
A A


30°
90°
B 7.5 cm C 30°
Rough diagram B 7.5 cm C

176 Mathematics–7
11 Perimeter and Area
Topics Covered
11.1  Perimeter and Area of a Square and a Rectangle
11.2  Area of a Parallelogram and a Triangle 11.3 Circumference and Area of a Circle
11.4 Conversion of Units and Applications

Let’s Remember
• Perimeter of a closed figure is the distance covered along the boundary, when you go round
the figure once.
• The area of a plane figure is its magnitude, measured in square units of length.

11.1  Perimeter and Area of a Square and a Rectangle


• Perimeter of a rectangle = 2(length + breadth) = 2 (l + b) units
• Perimeter of a square = 4 × side = 4a units
• Area of a rectangle = length × breadth = l × b sq. units
• Area of a square = Side × Side = (Side)2 = a2 sq. units
1
• Length of a rectangle = (Perimeter) – Breadth Rectangle b Square a
2
1
• Breadth of a rectangle = (Perimeter) – Length l a
2
1 Area
• Side of a square = (Perimeter) • Length of a rectangle =
4 Breadth
Area
• Breadth of a rectangle = • Side of a square = Area
Length
Exercise 11.1
I. Very Short Answer Type Questions (1 Mark)
A. Answer the following.
1. What do you mean by the perimeter of a closed figure?
2. What is the perimeter of the following figures?
6m
4m

3m 14 m
(i) 2m (ii) 12 m
4m

1.5
m 12 m
6m
14 m
4m

6m

177
3. How many units would you move if you travel around the following figures once?

(i)           (ii)


4. Do two congruent figures always have equal area?
5. Do two figures with equal perimeter will always be congruent?
6. What do you mean by area of a plane figure?
7. Area of a square is 289 cm2. Find its side.
8. The perimeter of a rectangle is 144 m. Its length is 40 m. What is its breadth?
9. The perimeter of a square garden is 44 cm. What is its area?
10. Anita wants to fence her garden on three sides with lengths 20 m, 12 m and 12 m. Find
the length of the fence required.
B. Fill in the blanks.
1. Area of a square of side 9 cm is equal to the area of ............. squares of each side 1 cm.
2. ................. of a regular polygon = Number of sides × length of one side
3. Perimeter of a .......................... = 4 × side
C. Say True or False.
1. Two congruent figures are equal in area.
2. Two rectangles with equal area have equal perimeter also.
3. Two figures can have equal areas but different perimeters.
II.  Short Answer Type Questions–I (2 Marks)
1. The area of a rectangular plot is 415 m2. If its breadth is 7.5 m, find its length.
2. The area of a square garden is 12321 m2. Find its perimeter.
3. The perimeter of a square is 40 cm. Find its area.
4. An increase in area of a figure always increases its perimeter. Justify whether true or false.
III.  Short Answer Type Questions-II (3 Marks)
1. The area of a square park is the same as of a rectangular park. If the side of the square
park is 60 m and the length of the rectangular park is 90  m, find the breadth of the
rectangular park.
2. A wire is in the shape of a rectangle. Its length is 40 cm and breadth is 22 cm. If the
same wire is rebent in the shape of a square, what will be the measure of its side? Also,
find which shape encloses more area?
3. The length and breadth of a playground are 80 m and 35 m, respectively.
(i) Find the cost of levelling it at ` 4.50 per square metre.
(ii) How long will a boy take to go three times round the field, if he walks at the rate of
1.5 m per second?
4. Find the length and breadth of a rectangle given that its perimeter is 960 cm and length
is 4 times its breadth.

178 Mathematics–7
IV.  Long Answer Type Questions (4 Marks)
1. A man covers a distance of 48 m while going around a square field twice. Find the side
of the square. How much wire would be needed for fencing around it once? Find the cost
of the wire if 1 m of it costs `1.75?
2. The perimeter of a triangle is 64 m. Two of its sides measure 15 m and 24 m, respectively.
Find the length of the third side of the triangle. Also, find the cost of putting the fence
at the rate of `15/m.

Answers and Hints


I. A. 1. Perimeter of a closed figure is the \ P = 2(L + B) = 2(x + 4x) = 2×5x = 10x
distance covered along the boundary, ⇒ 10x = 960 ⇒ x = 96 cm
when you go around the figure once. So, B = 96 cm and L = 4 × 96 cm = 384 cm
2. (i) 16.5 m (ii) 72 m IV. 1. Distance around the square field twice
3. (i) 24 m (ii) 15 m = 48 cm.
4. Yes 5. No, not always Distance around the square field once
6. The magnitude of a plane region is 48
called its area. = = 24 cm = Perimeter.
2
P 24
7. 17 cm 8. 32 m Side of the square = = = 6 m.
2 4 4
9. 121 cm 10. 44 m
Length of wire needed for fencing 1 round
B. 1. 81 2. Perimeter 3. Square = 24 m
C. 1. True 2. False 3. True Cost of 1 m = `1.75
II. 1. 55.33 m 2. 444 m Cost of 24 m = `24 × 1.75 = `42.
3. 100 sq cm 2. Let the third side of the triangle be x
4. True because increase means size P = 64 = 15 + 24 + x
increases and hence perimeter increases.
⇒ 64 = 39 + x
III. 1. 40 m
⇒ 64 – 39 = x
2. 31 cm, square encloses more area
⇒ 25 = x
3. (i) `12600 (ii) 460 sec, i.e., 7 min 40 sec
⇒ x = 25 m.
4. Let breadth of the rectangle be x cm.
Cost of putting the fence
Then length of the rectangle is 4x cm.
= ` 25 × 15 = ` 375
11.2 Area of a parallelogram and a triangle
• Area of a parallelogram = (base × corresponding height) sq. units d1 d2
h
1 æ1 ö
• Area of a rhombus = 2 ´ product of diagonals = ççç 2 ´ d1 ´ d 2 ÷÷÷ sq. units b
è ø Parallelogram Rhombus
1
• Area of a triangle = × base × height (i.e., altitude) = c bh m sq. units
1
2 2
1 
• Area of a right-angled triangle =  × product of its legs sq. units h
2  b
Example 1. Find the area of a parallelogram whose base and corresponding altitudes are
20 m and 5 m respectively.

Perimeter and Area  179


Solution. Area of a parallelogram = base × corresponding attitude
= 20 m × 5 m = 100 m2
Example 2. Find the area of the following triangles.

5 cm
3 cm
4 cm 12 cm
(i) (ii)
1
Solution. (i) Area of the given triangle = × base × height
2
1
= × 4 × 3 = 6 cm2
2

(ii) Area of the given right-angled triangle
1
= × product of its legs × height
2
1
= × 12 × 5 = 30 cm2
2
Example 3. Find the area of a rhombus, the lengths of whose diagonals are 36 cm and 22.5 cm.
1
Solution. (i) Area of the rhombus = × product of diagonals
2
1
= × 36 × 22.5 = 405 cm2
2

Exercise 11.2
I. Very Short Answer Type Questions (1 Mark)
A. Answer the following.
1. Find the area of the given parallelogram.

2. Find the area of the triangle LMN. (NCERT)

B. Fill in the blanks.


1. To find the area of a parallelogram, any of its ............. can be chosen as its base.

180 Mathematics–7
2. If the base of triangle is tripled and the height is reduced to one-third, then its area is
............. to the original area.
3. If a wire in the shape of a square is rebent to form a triangle, then ............. of both shapes
remain same, but ............. may vary.
4. The adjacent sides of a parallelogram are 10 cm and 8 cm. If the perpendicular distance
between the smaller sides is 6 cm, the distance between the greater sides is ............. .
5. Altitude of a triangle whose area is 40 cm2 and the base is 8 cm is ............. .
C. Say True or False.
1. The area of a right triangle is 80 cm2. The two sides containing the right angles can be
20 cm and 4  cm.
2. If each side of a rhombus is 15 cm and its area is 75 cm2, then its altitude is 5 cm.
3. All triangles equal in area are congruent.
4. Triangles with equal base have equal area.
II.  Short Answer Type Questions–I (2 Marks)
1. The side of a rhombus is 7.2 cm and its altitude is 5 cm. Find its area.
2. Area of a parallelogram is 24x2 m2. If height is 6x  m, find the base.
3. The two equal sides of a right-angled isosceles triangle are 10 cm each. Find its area.
4. All triangles have the same altitude. Justify whether the statement is true or false.
5. In the given figure, PQRS is a parallelogram, ST ^ PQ, PQ = 30 cm, ST = 15 cm, and
QR = 25 cm. Find the length of SU, where SU ^ QR.
S R

15 cm 25 cm
U

P T Q

III. Short Answer Type Questions-II (3 Marks)


1. DL and BM are the heights on sides AB and AD,
respectively of parallelogram ABCD. If the area of
the parallelogram is 1470  cm2, AB = 35 cm and
AD = 49 cm, find the length of BM and DL.
2. DABC is right-angled at A. AD is perpendicular to
BC. If AB = 5 cm, BC = 13 cm and AC = 12 cm,
find the area of DABC. Also find the length of AD.

3. The ratio of two adjacent sides of a parallelogram is 2 : 3. Its perimeter is 50  cm. Find
its area if altitude corresponding to longer side is 10 cm.

Perimeter and Area  181


4. A field in the form of a parallelogram, has one of its diagonals 42 m long and the
perpendicular distance of this diagonal from either of the outlying vertices 10 m and 8 m.
Find the area of the field.
5. Find the area of an equilateral triangle whose one side is 7 cm. (Hint: Draw altitude to
one side, prove two triangles formed are congruent.)
6. The area of a triangle is equal to that of a square whose each side measures 60 m. Find
the side of the triangle whose corresponding altitude is 90 m.
7. The base and corresponding height of a triangle are in the ratio 3 : 4 and its area is
96 cm2. Find its base and height.
IV.  Long Answer Type Questions (4 Marks)
1. If the area of a square with side a units is equal to the area of a triangle with base a units,
then find the altitude of the triangle.  [HOTS]
2. In two triangles, the ratio of their areas is 16 : 12 and that of their heights is 6 : 8. Find
the ratio of their bases.  [HOTS]
3. In a parallelogram, the attitude is twice the length of the base. Its area is 450 cm2. Find
the length of the base and the altitude.
4. A square and a parallelogram have the same area. If a side of the square is 40 m and
the height of the parallelogram is 20 m, find the length of the corresponding base of the
parallelogram.
5. Find the area of parallelogram PQRS, if PR = 24 cm, and ST = 8 cm = QU.

6. A room is 25 m long and 10 m wide. A triangular carpet is laid at the centre with
base = 8 m and height = 12 m. How much area is not carpeted?
Answers and Hints
I. A. 1. 24 sq cm 2. 3 sq cm of base and area are same, then altitude
B. 1. sides 2. equal will be same.
3. perimeter, area 4. 4.8 cm 5. PQ × ST = QR × SU
5. 10 cm ⇒ 30 × 15 = 25 × SU
30 6 # 15 3
C. 1. False 2. True 3. False ⇒ SU = =18 cm
25 5 1
4. False III. 1. DL = 42 cm, BM = 30 cm
II. 1. 36 sq cm 2. 4x m 3. 50 sq cm 60 8
2. 30 cm2, cm = 4 cm
4. False because length of altitude depends 13 13
upon length of base and area. If length 3. 150 cm2 4. 378 m2

182 Mathematics–7
5. 21.21 cm2 6. 80 m Area of the parallelogram = B × A
7. 12 cm, 16 cm ⇒ 1600 = x × 20
IV. 1. h = 2a 2. 16 : 9 1600 80
⇒ x = = 80 m
3. Let base = x cm 20 1
Base of the parallelogram = 80 m
Then, altitude = 2x cm
5. Area of parallelogram
Area = B × A ⇒ 450 = x × 2x
= area (DPQR) + area (DPSR)
⇒ 2x2 = 450 ⇒ x2 = 225
1 1
⇒ x = 15 cm ⇒ B ase = x = 15 cm = e # 24 # 8 4 + # 24 # 8 4 o
2 2
Altitude = 2x = 2 × 15 cm = 30 cm
= (96 + 96) cm = 192 cm2
2
4. Area of the square = (side)2 = (40)2

= 1600 m2 6. 202 m2

11.3 Circumference and Area of a Circle


• The perimeter of a circle is called its circumference.
• The ratio of the circumference to the diameter of a circle is a constant quantity. This constant
22
is denoted by p whose approximate value is or 3.14.
7
• Circumference of a circle with radius r is 2pr.
• Area of a circle of radius r is pr2.
1 2
• Area of a semi-circle of radius r is r .
2
1 2
• Area of a quadrant of a circle is r .
4
Exercise 11.3
I. Very Short Answer Type Questions (1 Mark)
A. Answer the following.
1. What is the perimeter of a circle called?
2. What is the circumference of a circle with radius 7 m?
3. Is the ratio of the circumference of a circle to its diameter always constant?
4. If diameter of a circle is doubled, by what factor will the perimeter change?
5. If radius of a circle is halved, what will be the corresponding change in its area?
6. Find the circumference of a circle of diameter 10 cm. (Use p = 3.14)
7. Find the perimeter of the given figure.

8. Find the area of a circle whose radius is 3.5 cm.


9. Find the circumference of a circle of diameter 14 cm.

Perimeter and Area  183


B. Fill in the blanks.
1. The distance around a circle is its ............. .
2. The circumference of a circle whose area is 81pr square units is ............. .
3. Value of p is approximately ............. in fraction.
4. Product of the diameter of a circle and the constant p is ............. of the circle.
5. The area of a circular ring with inner and outer radii r1 and r2, respectively is ............. .
6. Radius of a circle whose circumference is 4pr is ............. .
C. Say True or False.
a
1. The perimeter of a quadrant of a circle with radius ‘a’ is  2a.
2
2. The ratio of areas of two circles is 16 : 81, then the ratio of their radii is 4 : 9.
3. The exact value of p is 3.14.
II.  Short Answer Type Questions–I (2 Marks)
 22 
1. Find the circumference of a circle when the radius is 14 cm.  Use    .
 7
 22 
2. Find the radius of a circle if the circumference is 44 cm.  Use    .
 7
3. Find the diameter of the circle whose area is 7546  m2.
4. If the circumference and area of a circle are numerically equal, then find its diameter.
 22 
5. How many times a wheel of radius 28 cm must rotate to go 352 m?  Take   
 7
6. The ratio of the radii of two wheels is 3 : 4. What is the ratio of their circumference?
7. The circumference of a circle is always more than three times its diameter. Justify whether
True or false.
III.  Short Answer Type Questions–II (3 Marks)
1. From a circular sheet of radius 4 cm, a circle of radius 3 cm is removed. Find the area
of the remaining sheet. (Take p = 3.14)
2. Shazli took a wire of length 44 cm and bent it into the shape of a circle. Find the radius
of that circle. Also find its area. If the same wire is bent into the shape of a square, what
will be the length of each of its sides? Which figure encloses more area, the circle or the
æ 22 ö
square? ççTake  = ÷÷÷
èç 7ø
3. From a circular card sheet of radius 14  cm, two circles of radius 3.5 cm
each and a rectangle of length 3 cm and breadth 1 cm are removed (as
æ 22 ö
shown in the figure). Find the area of the remaining sheet. ççTake  = ÷÷÷
èç 7ø
4. A circular flower garden has an area of 314  m2. A sprinkler at the centre of the garden
can cover an area that has a radius of 12  m. Will the sprinkler water the entire garden?
(Take p = 3.14)

184 Mathematics–7
5. Find the circumference of the inner and the outer circles, shown in the figure? (Take p = 3.14).

6. Find the perimeter of the given figure.

7. The circumference of the front wheel of a car is 10 dm and that of the hind wheel is
16 dm. How many revolutions more will the first wheel make than the second in covering
a distance of 96  km?
8. The area of a circle of radius 10 cm is numerically what percent of its circumference?
 [HOTS]
IV.  Long Answer Type Questions (4 Marks)
1. From a rectangular metal sheet of sides 30 cm and 40 cm, a circular sheet as big as
possible is cut off. Find the area of the remaining sheet. (Take p = 3.14)
2. A 7 m wide path is to be constructed all around, and outside a circular garden of diameter
112 m. Find the cost of constructing the path at ` 50 per square metre.
3. ARB represents a semicircular plate where AB = 14 cm. A semicircle
ASO is cut away (O is the mid-point of AB). What is the area of
 22 
the remaining portion?  Take   
 7
4. The diameter of the given circles are 21  cm, 14 cm and 7 cm (see
figure). What is the area of the shaded region?

5. If a wire is bent into the shape of a square, the area of the square is
3969 cm2. When the wire is re-bent into a semicircular shape, find
 22 
the radius of the semicircle.  Take     [HOTS]
 7
6. A Billiard/Snooker table has dimensions
1
as th of its actual size as shown in the
10
figure. The portion excluding six holes each
of diameter 0.5 cm needs to be polished at
the rate of `200 per m2. Find the cost of
polishing.

Perimeter and Area  185


7. Pizza house sells pizzas in two shapes—square and circle. A square pizza of side 4.5 cm
costs `300 and a circular pizza of diameter 25 cm costs `160. Which pizza is a better deal?

Answers and Hints


I. A. 1. Circumference 2. 44 m II. 1. 88 cm 2. 7 cm 3. 98 m
3. Yes 4. 4 units 5. 2 times 6. 3 : 4
4. Perimeter will become double 7. Yes, because circumference of a circle
5.  Area = pr2 = A is πd and π is approximately 3.14.
r 2 πr 2 III. 1. 21.98 cm2
\ New Area = π d n =
2 4 2. r = 7 cm, 154 cm, 11 cm, circle encloses
A 1 more area
= = th of original area
4 4 3. 536 cm2
6. 31.4 cm 7. 25.7 cm 4. Yes, because sprinkler can water 452.16 m2
22
8. Area = × 3.5 × 3.5 cm2 5. 56.52 m, 119.32 m 6. 12.5 cm
7 7. 36000 8. 500%
= 22 × 0.5 × 3.5 cm2 = 38.5 cm2
22 14 IV. 1. 493.5 sq cm
9. C = 2pr = 2 # # cm = 44 cm 2. Area = 21618 m2, Cost = ` 130900
7 2
B. 1. circumference 3. 57.75 sq cm 4. 154 sq cm
22
2. 162 p units 3. 5. 49 cm 6. ` 497.60
7
4. circumference 7. Circular pizza as its area is more than
5. p(r22 – r12) sq. units 6. 2r square pizza.
C. 1. True 2. True 3. False

11.4 Conversion of Units and Applications


Conversion of Units
• We know that 1 cm = 10 mm
So, 1 cm2 = 1 cm × 1 cm = 10 mm × 10 mm = 100 mm2
Similarly 1 m2 = 10,000 cm2
and 1 hectare = 10,000 m2
Applications
Sometimes we need to find the area of the paths in or around park or also at times between as cross
paths. Now we will discuss some such cases.
Example 1: A garden is 90 m long and 75 m broad. A path 5 m wide is to be built outside
and around it. Find the area of the path. Also
5m

find the area of the garden in hectare.


90 m
Solution. Area of the garden = l × b
75 m

5m 5m
= 90 m × 75 m
= 6750 sq m
6750
= = 0.675 hectare
5m

10, 000

186 Mathematics–7
Now, length of the garden with the path, L = (90 + 5 + 5) = 100 m
and breadth, B = ( 75 + 5 + 5) m = 85 m
Area of the garden with the path = 100 × 85 sq m = 8500 sq m.
Area of the path = Area of the garden with path – Area of the garden
= (8500 – 6750) sq m = 1750 sq m.
Example 2: A path 1 m wide is built along the border and inside a square garden of side 30 m.
Find:
(i) the area of the path
(ii) the cost of planting grass in the remaining portion of the garden at the rate
of `40 per m2.
Solution. Area of the square garden = 30 × 30 = 900 m2
Side of the grassy portion = (30 – 1 – 1) m = 28 m
Area of the grassy plot = (28 × 28) m2 = 784 m2
Area of the path = Area of the garden – Area of the grassy plot
= (900 – 784) m2 = 116 sq. m.
30 m
1m

30 m

1m 1m
1m

Cost of planting grass on 1 sq m = ` 40


Cost of planting grass on 784 sq m = ` 784 × ` 40 = ` 31360
Example 3: A field 150 long and 120 m wide has two crossroads in its centre. Each road is
3.2 m wide. Find the area of the crossroads.
Solution. Lengths of the crossroads are 150 m and 120 m, respectively and width is 3.2  m.
150 m
Road 1

3.2
120 m
3.2

Road 2

Area of the crossroads = Area of road 1 + Area of road 2


– Area of the common square portion
= (150 × 3.2) + (120 × 3.2) – (3.2 × 3.2)
= 480 + 384.0 – 10.24 = 853.76 m2

Perimeter and Area  187


Exercise 11.4
I. Very Short Answer Type Questions (1 Mark)
Fill in the blanks.
1. One-fourth of a hectare = ............. m2
2. 50 cm2 = ............... mm2
3. 1000 cm2 = ............... m2.
II.  Short Answer Type Questions–I (2 Marks)
1. Find the area of the given figure.

5 cm 3 cm
1.5 cm

9 cm

2. Find the area of the shaded region.


10 m
8m

2m

III.  Short Answer Type Questions–II (3 Marks)


1. Find the area of the given figure.

2. A rectangular field is 24 m long and 15 m


wide. How many triangular flower beds,
80 cm
D R C
each of base 3  m and altitude 4 m can be
laid in this field?
3. ABCD is a given rectangle with length as
80 cm and breadth as 60 cm. P, Q, R, S are
the mid points of sides AB, BC, CD, DA
60 cm

S Q
respectively. A circular rangoli of radius
10  cm is drawn at the centre as shown in
the figure. Find the area of shaded portion.
[Use p = 3.14] [Multidisciplinary Questions]
A P B

188 Mathematics–7
IV. Long Answer Type Questions (4 Marks)
1. A verandah of width 2.25 m is constructed all along outside a room which is 5.5 m long
and 4 m wide. Find:
(i) the area of the verandah.
(ii) the cost of cementing the floor of the verandah at the rate of ` 200 per m2.
2. A path 1 m wide is built along the border and inside a square garden of side 40 m. Find:
(i) the area of the path.
(ii) the cost of planting grass in the remaining portion of the garden at the rate of ` 50 per m2.
3. Two crossroads, each of width 10 m, cut at right angles through the centre of a rectangular
park of length 700 m and breadth 300 m and parallel to its sides. Find the area of the
roads. Also find the area of the park excluding crossroads. Give answer in hectares.
4. Pragya wrapped a cord around a circular pipe of radius 4 cm and cut off the length required
of the cord. Then she wrapped it around a square box of side 4 cm (also shown). Did
she have any cord left? (Take p = 3.14)

5. In the following figures, find the area of the shaded portions:


E
A 10 cm 8 cm B P Q
6 cm

20 cm
10 cm

T
10 cm

F
4 cm

S R
10 cm U 10 cm
D 18 cm C
(i) (ii)
6. Architects design many types of buildings. They draw plans for houses, such as the plan
shown in the figure.  [Multidisciplinary Questions]

Family Kitchen
Room 3.65 m × 4.57 m
4.57 m × 5.48 m
Dining
Room
5.41 m × 5.48 m

Bathroom
2.75 m
×2m

Living
Bedroom 1 Bedroom 2 Room
3.04 m × 3.04 m × 3.81 m ×
3.04 m 2.43 m 7.53 m

An architect wants to install a decorative moulding around the ceilings in all the rooms. The
decorative moulding costs ` 500/metre.

Perimeter and Area 189


(a) Find how much moulding will be needed for reach room.
(i) Family room (ii) Living room (iii) Dining room (iv) Bedroom 1
(v) Bedroom 2
(b) The carpet costs `200/m2. Find the cost of carpeting each room.
(c) What is the total cost of moulding for all the five rooms?
7. Amrita is a nature lover. She has marked some land for her pets and plants. Find the
ratio of the areas kept for pets and plants to the living area. Write the values shown by
Amrita.  [Value Based Questions]

8. Manu bought a new field that is next to one she already owns. This field is in the shape
of a square with side 70  m. She makes a semi-circular lawn of maximum area in this
field.
(i) Find the perimeter of the lawn. (ii) Find the area of the square field excluding the lawn.
 [Life Skills]

Answers and Hints


I. 1. 2500 2. 5000 (v) 10.94 m
3. 0.1 (b) (i) Bedroom 1: ` 1848
II. 1. 19.5 cm2 2. 67.43 m2 Bedroom 2: ` 1477
III. 1. 36 cm2 2. 60      3. 2086 cm2 Living room: ` 5735.86
IV. 1. (i) 63 m2 (ii) ` 12600 Dining room: ` 5929.36
2. (i) 156 m2 (ii) ` 72200 Family room: ` 5008.72
3. 0.99 hectare, 20.01 hectare (c) ` 43830
4. Yes, 9.12 cm 7. 529 : 225, caring, kind hearted, nature
5. (i) 110 sq cm (ii) 150 sq cm lover, etc.
6. (a) (i) 20.10 m (ii) 22.68 m 8. (i) 180 m (ii) 2975 m2
(iii) 21.78 m (iv) 12.16 m qqq

190 Mathematics–7
12 Algebraic Expression
Topics Covered
12.1 Types and Terms of Algebraic Expressions 12.2  Operations on Algebraic Expressions
12.3 Finding the value of an Expression 12.4 Using Algebraic Expressions – Formula
and Rules

Let’s Remember
• We can use letters to represent numbers when you don’t know what the numbers are. These
letters are known as variables.
• We write 2 times x as 2x, 6 times y as 6y etc. 1 time z is written as z.

12.1 Types and Terms of Algebraic Expressions


3
• The expressions like 15 – 3x + 5y – 6z, 9a, 6 p , 7u2 – 3v, etc., are composed of numerals,
q
variables and four fundamental arithmetic operations +, –, × and ÷, are called algebraic
expressions. Each part of the algebraic expression, separated by the sign of ‘+’ and ‘–’, is
called a term.
In the expression 15 – 3x + 5y – 6z; 15, – 3x, 5y and –6z are the terms of the algebraic
expression.
• An algebraic expression with one term is called a monomial.
7
The algebraic expressions 5x,  x 2 y, 8a3, 11xyz, etc. are all monomials as they all have
only one term. 9
• An algebraic expression with two terms is called a binomial and the one with three terms
is called a trinomial.
9
• Term s like 6a, –7a, a and 3xy, –7xy, etc., using the same literal factor are called like
11
terms.
• Terms having different literal factors are called unlike terms, e.g., 5x, 6x2, – 7y, 8a3, 9y3z,
etc, are unlike terms.
• In the term like 120xy2z which is a product of the numeral 120, and literals x, y2 and z,
the numeral 120 and literals x, y2 and z are called the factors of the term 120xy2z. Either
factor is called the coefficient of the other. The constant factor 120 is called the numerical
coefficient. The factors x, y2 and z are called literal coefficients.
• A term containing no literal factors is called a constant term.

191
Example 1: Identify which of the following algebraic expression are polynomials.
p 2 – 3p + 1 1
(a) 4x2 – 3x + 1 (b) (c) – 3q
p 8
Solution. (a) and (c)
Example 2: Identify the following as monomials, binomials and trinomials.
9 3 5
(a) 4a – 7b (b) – abc (c) 2p2 – p + (d) 7
3 2 3
Solution. (a) binomial (b) monomial (c) trinomial (d) monomial
Example 3: Write the constant term in the following expressions.
(a) 9 – 8x2 + 4y2 (b) 3p2 + 4q – 2pq
Solution. (a) Constant term 9 (b) Constant term 0
Example 4: Write the degree of the polynomial a3b – 2ab4 + 1.
Solution. Highest power of 2ab4 = 2a1b4, i.e., 1 + 4 = 5
Example 5: Identify the like term in the expression
p2q + 4qp2 – 4pq + 3p2q2 – p2q
Solution. p2q – p2q
Example 6: Write the numerical coefficient of each term in the following.
12a2 – 7ab + 2c2
Solution. Coefficient of 12a2 = 12
Coefficient of –7ab = –7
Coefficient of 2c2 = 2
Example 7: Write coefficients of z in 4zx.
Solution. 4x
Example 8: Write the algebraic language of “thrice cube a number x subtracted from six times
the sum of y and 4 is 10”.
Solution. 6(y + 4) – 3x3 = 10

Exercise 12.1
I. Very Short Answer Type Questions (1 Mark)
A. Answer the following.
1. What are the like terms in 5a(5 – 2b) and 6(ab + a2)?
2. What is the coefficient of bc in 9a2bc + 7a2cb  – 2bca2?
3. What are the factors of the term –pq2?
4. Is it necessary that in like terms the numerical coefficients should also be the same?
5. In an algebraic expression what is a constant term?
6. Are 3a2b and – 7ba2 like terms?
7. What is the algebraic variable in the expression pr2?
8. How many terms a trinomial has?
9. Write the coefficient of x2 in x3 – 2x2 + 3x + 1.
10. Write the degree of the polynomial 4p2 + 6p3 + 8p8.

192 Mathematics–7
B. Fill in the blanks.
1. 5x3y and – 15yx3 are ............. terms.
2. 20p2q and – 20q2p are ............. terms.
3. The expression 43 + 62 is a ............. .
4. The speed of a train is 120 km/h. The distance covered in x hrs. is ............. .
5. The value of 3x2 – 5x + 3 when x = 2 is .............  .
6. Simplified form of 7x3 – 3x2y + xy2 + x2y – y3 is ............. .
C. Say True or False.
1. 3x + 32x2 + 4x3 – 26x2 – 6x2 is a binomial.
2. A trinomial has exactly three terms.
3. Numerical coefficient of –63x3y is 63.
4. –36cab is a monomial.
5. 13x – 5 = 8x
6. For any natural number p; 2p – 1 is an odd number.
7. Difference of a binomial and a monomial is always a monomial.
II.  Short Answer Type Questions–I (2 Marks)
1. Identify terms and factors in the expressions given below:
(i) –4x + 5 (ii) –4x + 5y (iii) 5y + 3y2 (iv) xy + 2x2y2 (v) pq + q
3 1
(vi) 1.2ab – 2.4b + 3.6a (vii) x + (viii) 0.1p2 + 0.2q2
4 4
2. Identify the numerical coefficients of terms (other than constants) in the following
expression: –p2q2 + 7pq
3. State whether a given pair of terms is of like or unlike terms.
1, 100
Justify whether True or False.
4. An algebraic expression with more than two terms is a trinomial.
5. (5x – y + 7) – (x + y) is a binomial.
6. In like terms, variables and their powers have to be same.
7. The terms of the expression 3x + 23x2 + 6y2 + 2x + y2 are 3x, 23x2, 6y2, 2x, y2.
8. 14p is the numerical coefficient of q2 in –14pq2.
9. The coefficient of x2 in –138x2y is –138y.
10. –7a2b and –7ba2 are like terms.
11. n2 and 6n are the like terms in the expression n(n + 1) + 6(n – 1).
III.  Short Answer Type Questions–II (3 Marks)
1. Fill in the blanks to make the given addition sentence true  [HOTS]
2 2 2 2 2 2
x – y + ....... – 3xy + 4y + 3y + ....... – x = 2x – 8xy + .......

Algebraic Expression  193


2. Art. Colour the adjoinings figure as per the instructions:
[Multidisciplinary Questions]
(a) Scalene triangle blue
(b) Isosceles triangles yellow
(c) Equilateral triangles green 
IV. Long Answer Type Questions (4 Marks)
1. Volume of a cuboid is expressed as Volume = Length × Breadth
× Height. Express the volume of a cuboid algebraically if its length
is three times the breadth and height is twice the breadth.
2. The area of a rectangle is expressed as Length × Breadth. The
length of a rectangle is thrice its breadth. Find the area of the
rectangle in term of its breadth p.
3. Write down the degree of each term of the algebraic expression
x3 – 7x2y + 5xy2 – 2.

Answers and Hints


I. A. 1. –10ab, 6ab 2. 9a2 + 7a2 – 2a2 4. False; a2 + b2 + 2ab – 6 has more than
3. –1, p, q, q2 4. No 2 terms but is not a trinomial.
5. Term without a literal factor 5. False; 5x – y + 7 – x – y = 4x – 2y +
7 is a trinomial.
6. Yes 7. r 8. three
9. –2 10. eight 6. True

B. 1. like 2. unlike 7. False; terms are 5x, 23x2, 7y2
3. monomial 4. 120x 8. False; because it is – 14p
5. 3 × 22 – 5 × 2 + 3 = 12 –10 + 3 = 5 9. True
6. 7x3 – 2x2y + xy2 – y3 10. True; because –7ba2 = –7a2b, i.e., literal
factors are same.
C. 1. False 2. True 3. False
11. False; n2 and 6n do not have same
4. True 5. False 6. True
literal factor.
7. False
III. 1. LHS
II. 1. (i) terms –4x, 5; Factors of –4x are –1,
2, 4, x x2 – y2 + ....... – 3xy + 4y2 + 3y2 –
(ii) terms –4x, 5y ....... – x2
(iii) terms 5y, 3y2 = 6y2 + ....... – 3xy – .......
(iv) terms xy, 2x2y2 (v) terms pq, q {Q x2 – x2 = 0; 4y2 + 3y2 – y2 = 6y2}
(vi) terms are 1.2ab, –2.4b, 3.6a 6 y 2 + −5 xy − 3 xy + 2 x 2

3 1
(vii) terms are x, = 2x2 – 8xy + 6y 2
4 4

(viii) terms are 0.1p2, 0.2q2
2. –1, 7 3. like

194 Mathematics–7
2. Art IV. 1. Let breadth of the cuboid be = x
Length of the cuboid be = 3x
\ Height of the cuboid be = 2x
Volume = l × b × h = 3x × x × 2x
= 6x3
2. Area of a rectangle = l × b
Breadth = p
Length = 3p
\ Area = l × b = 3p × p = 3p2
3. Degree of term x3 = 3
Degree of term –7x2y = 2 + 1 = 3
Degree of term 5xy2 = 1 + 2 = 3
Degree of term –2 = 0

12.2  Operation on algebraic Expressions


• The process of finding the value of an algebraic expression by replacing the variables by
their particular values is called substitution.
• To add or subtract two or more algebraic expressions, collect different groups of like terms
and find the sum or difference (as sign of terms suggests) of like terms in each group.
• Only like terms can be added or subtracted. Sum or difference of two like terms is a like
term with the coefficient equal to sum or difference of the coefficients of the two like terms.
• To add two expressions we add their like terms and leave unlike terms as they are.
• To subtract one expression from the other, we change the sign, i.e., from + to – and – to
+ of each term of the expression to be subtracted and the resulting expressions are added
together.
Example 1: Add the following.
p + q + 3 and 3p + 2q + 5
Solution. Horizontal Method Column Method
(p + q + 3) + (3p + 2q + 5) p+q+3
= (p + 3p) + (q + 2q) + (3 + 5) + 3p + 2q + 5
= 4p + 3q + 8 4p + 3q + 8
Example 2: Subtract a2 + b2 from 5a2 – 3b2
Solution. Horizontal Method Column Method
(5a2 – 3b2) – (a2 + b2) 5a2 – 3b2
= 5a2 – a2 – 3b2 – b2 – a2 – b2
= 4a2 – 4b2 4a2 – 4b2
= 4 (a2 – b2) = 4(a2 – b2)

Algebraic Expression  195


Exercise 12.2
I. Very Short Answer Type Questions (1 Mark)
A. Answer the following.
1. Subtract –7p2qr from –3p2qr. 2. Add p2 –7pq – q2 and – 3p2 – 2pq + 7q2.
2 2 2
3. Add the term x y, 5x y, –7x y 4. Subtract x2 + y2 from 5x2 + 3y2 + 2
5. What is the sum of x and y? 6. Add the following term x2, 4x2, –7x2
B. Say True or False.
1. Sum of 3a and 7b is 3a + 7b. 2. Sum of 4 and x is 4x.
3. If Raghav has 3ab toffees and Manav has 20ba toffees, then Manav has 17ab more toffees.
II.  Short Answer Type Questions–I (2 Marks)
A. Answer the following:
1. Simplify combining like terms: p – (p – q) – q – (q – p)
2. Add: 3mn, –5mn, 8mn, –4mn 3. Subtract: (a – b) from (a + b)
4. Add: t – 8tz, 3tz – z, z – t
5. From the sum of 3x – y + 11 and –y – 11, subtract 3x – y – 11.
6. Simplify: 8r3 – 10r3 + 6r3 7. Add x2 + 2 and –6x2 + 3
8. Subtract: 4a + 3 from 5a + 6 9. Simplify: [3c – (2c + 5d) – d]
B. Justify whether True or False.
1. The sum or difference of two like terms may not be like term to the given terms.
2. The perimeter of a square of side 2p + 3 is given by the expression 8p + 12.
III.  Short Answer Type Questions–II (3 Marks)
1. From the sum of x2 – 2xy + y2 and x2 + 2xy + y2 subtract x2 – 4xy + 4y2.
2. Add x3 + 6x2, 5x2 + 3x and 5x – 7.
3. If A = 3x2 – 4x + 1, B = 5x2 + 3x – 8 and C = 4x2 – 7x + 3 then find: (A + B) – C.
4. What should be added to x3 + 3x2y + 3xy2 + y3 to get x3 + y3?
5. How much does 99p2 – 55p + 4 exceed 13p3 – 5p2 + 17p – 90?
6. The rate of planting the grass is `x per square metre. Find the cost of planting the grass
on a triangular lawn whose base is y metres and height is z metres.
7. Find the perimeter of the figure given below:

8. A wire is (9x – 5) m long. A length of (7x – 6) m is cut for use. Now, answer the following
questions:
(i) How much wire is left?
(ii) If this left out wire is used for making an equilateral triangle, what is the length of
each side of the triangle so formed?

196 Mathematics–7
IV. Long Answer Type Questions (4 Marks)
1. If A = 2x2 – 5, B = x2 – 3x + 6 and C = x2 – 4x +  2, find A + 2B + 2C.
2. From the sum of 4 + 3x and 5 – 4x + 2x2, subtract the sum of 3x2 – 5x and –x2 + 2x + 5.

3. If then find the value of:


4. Subtract the sum of 12ab – 10b2 – 18a2 and 9ab + 12b2 + 14a2 from the sum of ab +
2b2 and 3b2 – a2.
5. Kunj’s mother gave him `3ab2 and his father gave her `5(ab2 + 2). Out of this total
money he spent `(10 – 3ab2) on her birthday party. How much money is left with her?
 [Life Skills]

Answers and Hints


I. A. 1. 4p2qr 2. –2p2 – 9pq + 6q2 2. True; because perimeter of a square =
3. x y + 5x y + (–7x2y)
2 2 4 side = 4(2p + 3) = 8p + 12
= 6x2y – 7x2y = – x2y III. 1. x2 – 2xy + y2 + x2 + 2xy + y2
4. 5x2 + 3y2 + 2 – (x2 + y2) – (x2 – 4xy + 4y2)
= 5x2 + 3y2 + 2 – x2 – y2 = 2x2 + 2y2 – x2 + 4xy – 4y2
= (5x2 – x2) + (3y2 – y2) + 2 = x2 – 2y2 + 4xy
= 4x2 + 2y2 + 2 2. x3 + 6x2 + 5x2 + 3x + 5x – 7
5.  x+y = x3 + 11x2 + 8x – 7
6. x2 + 4x2 + (–7x2) = 5x2 – 7x2 = –2x2 3. (A + B) – C = 3x2 – 4x + 1 + 5x2
B. 1. True 2. False 3. True + 3x – 8 – (4x2 – 7x + 3)
II. A. 1. p – p + q – q – q + p = p – q = 8x2 – x – 7 – 4x2 + 7x – 3
2. 3mn + (–5 mn) + 8 mn + (–4 mn) = 4x2 + 6x – 10
= 3mn + 8 mn – 5mn – 4mn 4. Let A be added to x3 + 3x2y + 3xy2
= 11 mn – 9mn = 2 mn + y3 to get x3 + y3
3. a + b – (a – b) = a + b – a + b = 2b ∴ A = x3 + y3 – (x3 + 3x2y + 3xy2 + y3)
4. t – 8tz + 3tz – z + z – t = –5tz = x3 + y3 – x3 – 3x2y – 3xy2 – y3
5. (3x – y + 11) + (–y – 11) = –3x2y – 3xy2
– (3x – y – 11) 5. Let 99p2 – 55p + 4 exceed 13p3 – 5p2
= 3x – y + 11 – y – 11 – 3x + y + 11 + 17p – 90 by A
= 11 – y i.e. 99p2 – 55p + 4 = 13p3 – 5p2
6. 8r3 – 10r3 + 6r3­= 14r3 – 10r3 = 4r3 + 17p – 90 + A

7. x2 + 2 + (– 6x2 + 3) ⇒ A = 99p – 55p + 4 – 13p3 + 5p2
2
= x2 – 6x2 + 2 + 3 = –5x2 + 5
– 17p + 90
8. 5a + 6 – (4a + 3) = 5a + 6 – 4a – 3 3 2
A = –13p + 104p – 72p + 94
= a + 3
6. Here base = y m and height = z metres
9. 3c – (2c + 5d) – d
Area of triangular lawn
= 3c – 2c – 5d – d = c – 6d
1 1
B. 1. False; it will always be a like term = × base × height = × y × z sq m
unless it becomes zero. 2 2

Algebraic Expression  197


cost of putting grass on 1 sq m = `  x
yz xyz 3.
cost of putting grass on sq m = `
2 2
7. (i) Perimeter = sum of all four sides 3 9 + 14 23
= 5x – y + 2(x + y) + 5x – y + 2(x + y) 3 = ´3 + 7 = =
2 2 2
= 10x – 2y + 2x + 2y + 2x + 2y
= 14x + 2y units
8. (i) Wire left is original length – length
of piece cut out
= 9x – 5 – (7x – 6) = 2x + 1
(ii) Let side of an equilateral be a units
Perimeter of ∆ = 3a 23 60 + 23 + 4 87
∴ 2x + 1 = 3a ⇒ a = ( 2 x + 1)
1 = 30 + − ( −2) = =
2 2 2
3
IV. 1. A = 2x2 – 5, B = x2 – 3x + 6 and 4. Sum of 12ab –10b2 – 18a2 and
C = x2 – 4x + 2 9ab + 12b2+ 14a2
A + 2B + 2C = 2x2 – 5 + 2 (x2 – 3x + 6) is 21ab + 2b2 – 4a2 ...(1)
+ 2(x2 – 4x + 2) = 2x2 – 5 Sum of ab + 2b2 and 3b2 – a2 is
+ 2x2 – 6x + 12 + 2x2 – 8x + 4 ab + 5b2 – a2 ...(2)
= 6x2 + 11 – 14x Subtracting (1) from (2)
2. Sum of 4 + 3x and 5 – 4x + 2x2 is ab + 5b2 – a2 – (21ab + 2b2 – 4a2)
2x2 – 4x + 5 + 3x + 4 = ab + 5b2 – a2 – 21ab – 2b2 + 4a2
= 2x2 – x + 9 ...(1) = 3b2 – 20ab + 3a2.
2 2
Sum of 3x – 5x and – x + 2x + 5 is 5. Mother gave = `3ab2
–x2 + 2x + 5 + 3x2­ – 5x Father gave = 5(ab2 + 2)
= 2x2 – 3x + 5 ...(2) = 5ab2 + 10
Subtracting (2) from (1) Total money with Kunj = 8ab2 + 10
2x2 – x + 9 Kunj spent `10 – 3ab2
2x2 – 3x + 5 Money left with her
– (+) (–) = 8ab2 + 10 – (10 – 3ab2)
2x + 4 = 8ab2 + 10 – 10 + 3ab2 = 11ab2.

12.3 Finding the value of an Expression


• The process of finding the value of an algebraic expression by replacing the variables by
their particular values is called substitution.
Example 1: Find the value of the expression p2 + 3p – 1 when p = 3.
Solution. When p = 3, p2 + 3p – 1 = 32 + 3 × 3 – 1 = 9 + 9 – 1 = 17

198 Mathematics–7
Exercise 12.3
I. Very Short Answer Type Questions (1 Mark)
A. Answer the following.
1. Find the value of the expression 3x2 – 5x + 3 (at x = –1)
2. Find the value of m3 + n3 + p3 at m = 1, n = –1 and p = –2.
x
3. Find the value of 4y – 1 for y = 1 4. Find the value of + 5 – 8 for x = 6
2
5. For x = 4 find the value of 2x + 7 – 15.
B. Fill in the blanks.
1. Value of 5x + 9 for x = 2 is ............ .
2. Value of x + y – 3 for x = 1, y = –2 is ............ .
y
3. Value of – 4 for y = – 3 is ............ .
3
II. Short Answer Type Questions–I (2 Marks)
1. Evaluate: 4a2 + 4a – 2 (at a = –3)
2. Given m = –2 and n = 6, evaluate 2m2 + 6n.
3. Find the value of the expression x2 + 3x – 1 when x = 2.
4. Find the value of the expression –x3 + 1 when x = –1
III. Short Answer Type Questions–II (3 Marks)
1
1. The sum of squares of first n natural numbers is given by (2n3 + 3n2 + n). Find the
6
sum of squares of the first 10 natural numbers.
2. Will the value of 17x for x = –5 be greater than 17 or less than 17? Explain.
3. What should be the value of a if the value of 2x2 + x – a equals to 5, when x = 0?
IV. Long Answer Type Questions (4 Marks)
1. Area: A misha bought a rectangular plot with length x m and breadth y m then sold a
triangular portion of it whose base is y m and height is z m. Find the area of the remaining
part of plot.  [Multidisciplinary Questions]
2. Nutrition: Observe the following nutritional chart carefully:
Food item Carbohydrates
(Per unit = 100 g)
Rajma 60 g
Cabbage 5g
Potato 22 g
Tomato 4g
Apples 1.4 g
Carrots 11 g

Algebraic Expression  199


Write an algebraic expression for the amount of carbohydrate in ‘g’ for:
(i) p units of tomatoes and 3 units of rajma.
(ii) 2x units of carrots and y units of apples.  [Multidisciplinary Questions]
2
3. Find the value of a if the value of 2p + p – a equals 5, when p = 2.

Answers and Hints


I. A. 1. 3(–1)2 – 5(– 1) + 3 = 3 + 5 + 3 = 11 1
2. 13 + (–1)3 + (–2)3 = 1 – 1 – 8 = –8 = [2000 + 300 + 10]
6
3. 4 × 1 – 1 = 4 – 1 = 3 1
6 = × 2310 = 385
4. + 5 – 8 =3 + 5 – 8 =8 – 8 =0 6
2 2. 17x]x = –5 = –85 < 17
5. 2 × 4 + 7 –15 = 8 – 8 = 0
3. 2x2 + x – a]x = 0 = 5
B. 1. 5 × 2 + 9 = 19
–a = 5 ⇒ a = –5
2. 1 + (–2) – 3 = 1 – 2 – 3 = 1 – 5 = – 4
–3 IV. 1. Area

3. – 4 = –1 – 4 = –5 Area of rectangle = xy
3
1
II. 1. 4a2 + 4a – 2]a = –3 Area of triangle = zy
2
= 4(–3)2 + 4 × (–3) – 2
= 4 × 9 – 12 – 2 = 36 – 12 – 2
= 24 – 2 = 22 z y
2. 2m2 + 6n]m = –2, n = 6
= 2 × (–2)2 + 6 × 6 = 2 × 4 + 36
x
= 8 + 36 = 44 1
Area of remaining plot = xy − zy
3. 22 + 3 × 2 – 1 = 4 + 6 – 1 = 9 2
4. –(–1)3 + 1 = –(–1) + 1 = 1 + 1 = 2 2. Nutrition
III. 1. Sum of squares of first 10 natural p units of tomatoes and 3 units of rajma
numbers is = 4p – 180
1 2x units of carrots and y units of apples
 2n3 + 3n 2 + n 
6  n = 10 = 22x + 1.4y
1 3. 2p2 + p – a = 5
= [2 × 103 + 3 × 102 + 10] ⇒ 2 × 22 + 2 – a = 5
6
1 ⇒ 8 + 2 – a = 5
= [ 2 × 1000 + 3 × 100 + 10]
6 ⇒ 10 – a = 5
⇒ a = 5
12.4 Using Algebraic Expressions–Formula and Rules
Rules for perimeter and area
1. If s is the side of an equilateral triangle then perimeter of the triangle = 3s.
2. If a is side of the square, then area of the square = a2.
3. If x is the base of a parallelogram and y the corresponding altitude, then area of the parallelogram
= xy.

200 Mathematics–7
Rules for number pattern
• Just like the formula for perimeter and area of plane figures, we can find some rules for
number patterns.
1. If x is any natural number, then (x – 1) is its predecessor, e.g., x = 40, then x – 1 = 40 – 1
= 39, is the predecessor of x.
2. 2x + l is an odd number, e.g., if x = 3
then 2x + 1 = 2 × 3 + 1 = 6 + 1 = 7, which is an odd number.
Pattern in Geometry

1 3 6 10 15

1, 3, 6, 10, 15, ... are called triangular numbers. If n is taken as the nth triangular number, then
n (n + 1)
the number of dots required to make the nth triangle is given by .
2
Exercise 12.4
I. Very Short Answer Type Questions (1 Mark)
A. Answer the following.
1. The side length of the top of a square table is x. Find its perimeter.
1
2. What is the number of scarves of length  m that can be cut from x metres long cloth?
2
3. Write the rule for the pattern.

Number of sides 4 5 6 7
Number of non-overlapping triangles 2 3 4 5

B. Fill in the blanks.


1. In the formula, circumference of a circle = 2pr numerical coefficient is ............. .
2. In the formula, area of a circle = pr2, the variable is ............. .

Algebraic Expression  201


1
3. The formula for the area of a triangle = bh is a ............. . (monomial, binomial, trinomial)
2
II.  Short Answer Type Questions–I (2 Marks)
1. Observe the patterns of digits made from line segments of equal length. You will find
such segmented digits on the display of electronic watches or calculators.
... ...

6 11 16 21 ... (5n + 1) ...


If the number of digits formed is taken to be n, the number of segments required to form
n digits is given by the algebraic expression appearing on the right of each pattern. How
many segments are required to form 5, 10, 100 digits of the kind ?
2. Find the number of triangles.


Matchsticks 3 5 7 9 11
Triangles
III. Short Answer Type Questions–I (3 Marks)
1.       +  D 
  D D  +   represents 4a2 + 3b2 + c, then write the algebraic expression for


D D D D  +      +  
  [HOTS]
2. Ranjita has a triangular lawn  whose  base  is y  m and height is z m. Find the  cost  of  planting
the grass at the rate of ` x per square metre. These days most of the houses don’t have
lawns. What does this say about Ranjita?  [Value Based Question]

3.

No. of digits (9) 1 2 3 n

No. of matchsticks 5 9 13 _____?


IV. Long Answer Type Questions (4 Marks)
1. Each symbol given below represents an algebraic expression:



The symbols are then represented in the expression:

202 Mathematics–7
Find the expression which is represented has a playground? Give two reasons?
by the above symbols.  [Value Based Question]
2. A school has a rectangular playground
with length x and breadth y and a square
lawn as shown in the figure given here.
What is the total perimeter of both of them
combined together? Why every school
Answers and Hints
I. A. 1. P = 4x 3. monomial
x
2. No. of scarves = = 2x II. 1. Patterns of 5 = 5 × 5 + 1 = 26
 1
  Patterns of 10 = 5 × 10 + 1 = 51
2
3. n –2 where n ≥ 4 Patterns of 100 = 5 × 100 + 1 = 501
B. 1. 2π 2. r

2.
Matchsticks 3 5 7 9 11
Triangles 1 2 3 4 5
2×1+1 2×2+1 2×3+1 2×4+1 2×5+1
III. 1.  
  
 = 4a2 means  = a2
  3.
2 2
D 
D 
D  = 3b means D = b No. of 1 2 3 n
=c digits (9)
∴ D D D = 4b2,  = 3c,  = a2
D  No. of 5 9 13 4n + 1
matchsticks 4 × 1
D 
D 
D D +  +  = 4b2 + 3c + a2 4×2 4×3 4×n
1 +1 +1 +1 +1
2. Area of triangular lawn = × y×z
2 IV. 1.
xyz
Cost of planting grass = `  2x2 + 3y + 5x2 + 3x – (8y­2 – 3x2 + 2x + 3y)
2
Ranjita is environment friendly. She = 7x2 + 3y + 3x – 8y2 + 3x2 – 2x – 3y
encourages greenery by setting an = 10x2 + x – 8y2
example. 2. Perimeter = 4x + 2y
(i) To keep the student physically fit.
(ii) To use students extra energy in the
positive way.
qqq

Algebraic Expression  203


13 Exponents and Powers
Topics Covered
13.1 Exponents 13.2  Laws of Exponents
13.3 Decimal Number System

Let’s Recall
• Product of any number of positive numbers is positive.
ïì-ve if n is odd
• Product of n negative numbers = ïí
ïïî+ve if n is even
For example, (–4) × (–5) × (–6) = –120 Q 3 negative numbers (i.e., odd) are multiplied
(–4) × (–5) × (–6) × (–2) = 240Q 4 negative numbers (i.e., even) are multiplied

13.1 Exponents
• A rational number x when multiplied by itself repeatedly n times can be written in the form xn.
For example, x × x × x ... n times = xn. This form is called exponential form. Here, x is
called the base and n is called the exponent or index or power.
(–1)odd positive integer = –1
and (–1) even positive integer = 1
a a k a –k
• For a rational number the reciprocal of d n = d n , where k is a natural number.
b b b
Example 1: Express the following in product form.
(a) (–12)6 (b) 48
Solution. (a) (–12)6 = (–12) × (–12) × (–12) × (–12) × (–12) × (–12)
(b) 48 = 4 × 4 × 4 × 4 × 4 × 4 × 4 × 4
Example 2: Find the value.
(a) 83 (b) (–6)3 ÷ (–3)3 (c) (–4)3 × (–5)3
(d) (–1) (e) (–1)16
25

Solution. (a) 83 = 8 × 8 × 8 = 512


(– 6) # (–6) # (–6) –216
(b) (–6)3 ÷ (–3)3 = = =8
(–3) # (–3) × (–3) –27
(c) (–4)3 × (–5)3 = (–4) × (–4) × (–4) × (–5) × (–5) × (–5)
= (–64) × (–125) = 8000

204
(d) (–1)25, since power of –1 is an odd integer \ (–1)25 = – 1
(e) (–1)16, since power of –1 is an even integer
\ (–1)16 = 1

Exercise 13.1
I. Very Short Answer Type Questions (1 Mark)
A. Answer the following.
1. Is 40 + 50 + 60 and (4 + 5 + 6)0 equal?
3
 2
2. What is the reciprocal of   ? 3. Find value of 44.
 5
4. Express in exponential form 5 × a × a × a × a × a × a × a
5. Which is greater 29 or 92? 6. Express 3600 as a product of prime factors.
7. Compare 12 × 105 and 3 × 107.
8. In what short way can we write repeated multiplication? Give an example.
9. In the statement “6 raised to power 3”, which number is the base and which number is the
exponent?
10. What other terms can be used for the word ‘exponent’?
11. What is the exponential form of 8?
12. What is the value of 37?
B. Fill in the blanks.
1. 1 lakh = 10.......
C. Say True or False.
1. Ten million = 107 2. 23 > 32
3. 00 is not defined. 4. (10 + 10 + 10)10 = 1010 + 1010 + 1010
5. pm + pm = p2m, where p is a non-zero rational number and m is a positive integer.

II.  Short Answer Type Questions–I (2 Marks)


1. Express the following as a product of prime factors only in exponential form:
108 × 192
2. Find the number from the following expanded form:
9 × 105 + 2 × 102 + 3 × 101.
Justify whether true or false.
3. 32 < 15 4. 63 < 44

III.  Short Answer Type Questions-II (3 Marks)


1
1. The number of diagonals of an n-sided polygon is n(n – 3). Express it in exponential
2
form.  [Multidisciplinary Question]
2. Divide 6,561 by 10,000 and express the result in exponential form.
5 3 2 1 4 128 2 –125
3. Simplify: d n × 9 2 × 4. Simplify: d – n × d n ×d n
3 9 8 5 4

Exponents and Powers  205


IV. Long Answer Type Questions (4 Marks)
Simplify:
–5 2 36 1 3 1 2 3 2 2 3
1. (92 – 43) × d n × 2. =d n – d n G # d n # d – n
18 25 2 3 5 3
3 3 16 2 3
3. =d n × = –d nG
4 27 3
33 3 2
4. Distinguish between the rational number 5 and d n . Which one is greater?
3 5
Answers and Hints
I. A. 1. No. 40 + 50 + 60 = 1 + 1 + 1 = 3 II. 1. 108 × 192 = (2 × 2 × 3 × 3 × 3)
and (4 + 5 + 6)0 = 150 = 1 × (2 × 2 × 2 × 2 × 2 × 2 × 3)
3 3
 2  5 53 125 = (2 × 2 × 2 × 2 × 2 × 2 × 2 × 2)
2. Reciprocal of   =   = 3 =
 5  2 2 8 × (3 × 3 × 3 × 3) = 28 × 34
3. Value of 44 = 4 × 4 × 4 × 4 2. 9 × 105 + 2 × 102 + 3 × 101
= 16 × 16 = 256 = 900000 + 200 + 30 = 900230
4. 5 × a × a × a × a × a × a × a = 5a7 3. True as 32 = 9 and 9 < 15
5. 2­9 = 2 × 2 × 2 × 2 × 2 × 2 × 2 × 2 4. True 63 = 216 44 = 16 × 16 = 256
× 2 = 512 and 216 < 256
92 = 81, So 29 > 92.
1 1 1 3n
6. 3600 = 2 × 2 × 2 × 2 × 3 × 3 × 5 × 5 III. 1. n × n − × n × 3 = n2 −
2 2 2 2
7. 12 × 105 = 1200000; 3 × 107 = 30000000
6561 38
So, 12 × 105 < 3 × 107 2. =
10, 000 10 4
8. 
In exponential form, e.g., 3 × 3 × 3 ×
3 × 3 = 35
9. Base = 6; Exponent = 3
10. Index or power
11. 8 = 2 × 2 × 2. In exponential form
8 = 23
12. 37 = 3 × 3 × 3 × 3 × 3 × 3 × 3 = 2187
B. 1. 5

C. 1. True
5 3
2
2. False as 23 = 8 and 32 = 9 and 8 < 9 3. d n # 9 2 #
3 9
3. True 5 5 5 2
= d n # e o # e o # 9 3 1 # 9 #
4. False as (10 + 10 + 10)10 = (30)10 3 3 3 9
= (3 × 10)10 = 310 × 1010 125 # 2 250
= =
and 1010 + 1010 + 1010 = 3 × 1010 and 3 3
1 4 128 2 –125
3 × 1010 ≠ 310 × 1010 4. d – n # d n #d n
8 5 4
5. False pm + pm = (1 + 1)pm = 2pm ≠ p2m

=

206 Mathematics–7
–1 –1 –1 –1 128 16 # 128 16 3 3 3 16 2 2 2
3. d# # n< – # # F
2 2

# # # #
8 8 8 8 5#5 4 4 4 27 3 3 3
27 16 8 27 16 – 8
–125 = < – F = #< F
# 64 27 27 64 27
4 1
27 8 1 1
2 # 2 –125 25
5
= × =
= # = –5 64 8 27 1 8
5#5 42 33 3 # 3 # 3 27
^ –5 h–1 ( –5 ) –1 36 2
1
4. = = and
IV. 1. ( 9 × 9 – 4 × 4 × 4) × # # 5 5 5
18 9 18 1 25 5
3 2 3 3 9
d n = # =
1

1 1 17
= (81 – 64) × = 17 × d n = 5 5 5 25
9 9 9
27 27 # 5 135
1 1 1 1 1
2. < × × – × F
But = =
5 5#5 25
2 2 2 3 3
135 9
1 1 –2 –2 –2
×d n× d n× e o× e o× d n
We know that in and
25 25
5 5 1 1 3
1 1 1 –8 135 > 9.
= < – F # #d n
135 9 33 3 2
> d n .
8 9 25 3
So > or
9–8 –8 1 –8 –1 25 25 5 5
= < F#< F = #d n =
72 75 72 75 675
13.2  Laws of Exponents
• Laws of exponents: If x and y be any non-zero rational number, and let m and n be any natural
numbers, then

Law 1. xm . xn = xm + n
Example: Simplify and express in exponential form
5 4 5 5 –12 3 –12 5
(a) d n # d n (b) d n #d n
7 7 13 13
Solution. We know that for any non-zero rational number xm. xn = xm + n
5 4 5 5 5 4+5 5 9
(a) \ d n # d n = d n = d n
7 7 7 7
–12 3 –12 5 –12 3 + 5 –12 8
(b) d n #d n = d n =d n
13 13 13 13
xm
Law 2 = x m–n , if m > n
xn
6 6 6 4 6 6–4 6 2
Example: d n ' d n = d n = d n
17 17 17 17
1
Law 3 xm ÷ xn = n – m if m < n
x
–2 7
–2 15
Example: d n ' d n
7 7

Exponents and Powers  207


1
–2 7 –2 15
d n 'd n =
1 m n
Solution. [ a ÷ a = a n–m
, if m < n]
7 7 –2 15–7
d n
1 7 8 7
= = d n
–2 8 –2
d n
7
Law 4 (xm)n = xmn
Example: (32)5 = 32 × 5 = 310
Law 5 xm × ym = (xy)m
Example: 34 × 54 = (3 × 3 × 3 × 3) × (5 × 5 × 5 × 5)
= (3 × 5) × (3 × 5) × (3 × 5) × (3 × 5) = (3 × 5)4
1 3 1 3
Example: d n # 93 = d # 9 n = 33 = 27
3 3
m
xm x
Law 6 =e o
ym y

23 2 # 2 # 2 2 2 2 2 3
Example: = = # # = d n
53 5 # 5 # 5 5 5 5 5
Law 7 x0 = 1 (where x is any non-zero rational number)
19 0 –13 0
Example: (a) d n = (b) d n =
87 15
19 0 13 0
Solution. (a) d n = 1 (b) d – n = 1
87 15
Exercise 13.2
I. Very Short Answer Type Questions (1 Mark)
A. Answer the following.
1. Write in exponential form 33 × 34 × 37.
2. Evaluate: 47 ÷ 42 3. Evaluate: (32)4
4. Find the value of (42)3 in exponential form.
B.  Fill in the blanks. 6 5 11
 13     13 
9 4 ........
1. (–5) ÷ (–5) = (–5) 2.      
 
14 14
3
  1
5
  1
........ 13
  1   8  4   8  ........
3.      
  4.       
7 7 7  15  15

15 5 2
 7   7  3   7 ........   1 18   1
........
5.        6.       
11   11  11   8    8

208 Mathematics–7
7 3 4
 15     15 
7.      
  8. p9 × p5 × p0 = ............... .
16   16
C.  Say True or False.
1. a0 × a0 = a0 ÷ a0 is true for all non-zero values of a.
3 3 2 5 10
 3  7  8  8  8
2.      1 3.      
 
 7  3 5 5 5
9 9 9
 6  5  6 5
4. 60 × 360 × 2160 = (60)6 5.        
 7 4 7 4
II.  Short Answer Type Questions-I (2 Marks)
Evaluate (Q1 to Q5)
 7  4  7  5   7  7  p6  5 0
12
1. (5 ÷ 5 ) × 5 7 5
2.         
 3.  4   p  p
 9 9  9 p 
34  53  4
4. 5. (– 3)3 × (– 10)3
5  81
6. Simplify and write in exponential form  (33 ) 2  26   76
Justify whether True or False.
7. 30 + 40 + 50 = (3 + 4 + 5)0
8. am × an = am + n, where a is a non-zero rational number and m, n are positive integers.

III.  Short Answer Type Questions-II (3 Marks)


Simplify: (Q1 to Q5)
 5  3  5  8   5  2  5  4  98 × x10 y 7 z12 68 × 93 × 253
1.   ×    ÷   ×    2. 3.
 7 7   7 7  96 × x8 y 4 z12 33 × 43 × 153
64  52  x 7 213  105  125
4. 5.
103  x 4 25  33  58
6. Find the value of [(–3)–3]–3.  [HOTS]

IV.  Long Answer Type Questions (4 Marks)


1. Simplify using laws of exponents  7294  7293   36 .  [HOTS]
2. 6x = 216, then find the value of 53x – 8.  [HOTS]
3 3 125
d– n # d n
4 27 (4 0 – 3 0 + 6 0)
3. Simplify: +
5 2 9 2 0
d n # d n
3 16 5
2
p 3 18
3 16
p
4. If e o = d n ' d n find e o
q 4 4 q

Exponents and Powers  209


Answers and Hints
I. A. 1. 33 × 34 × 37 = 33 + 4 + 7 = 314 7. False as 30 + 40 + 50 = 1 + 1 + 1 = 3
2. 47 ÷ 42 = 4­7 – 2 = 4­5 = 1024 (3 + 4 + 5)0 = (12)0 = 1 ∴ 3 ≠ 1
3. (32)4 = 38 = 6561 8. True as am × an = am + n
4. (42)3 = 42 × 3 = 46 When we multiply two numbers with
13 same base we add the exponents.
B. 1. 5 2. 3. 8
5 3 5 8 5 2 5 4
III. 1. =d n # d n G ' =d n # d n G
14
4. 12 5. 0 6. 36 7 7 7 6
15 5 3+8
5 2+4
5 11
5 6
7.
16
8. p14 = d n ' d n = d n ' d n
7 7 7 6
C. 1. True 5 11– 6
5 2+4
3 3 3 3 6 = d n = d n
 3  7  3  3  3 7 7
2. False   ÷   =   ×   =  
 7  3  7  7  7 8
9 x y z
# 10 7 12
98 # x10 y 7 z12
 8
2
 8
5
 8
2+5
 8
7 2. 6 =
3. False   ×   =   =   9 # x8 y 4 z12 96 # x8 y 4 z12
 5  5  5  5
= 98–6x10–8y7–4z12–12 = 92x2y3z0
0 0 0 06
4. True 6 × 36 × 216 = (6 ) as 1 × 1 ×
= 81x2y3.1 = 81x2y3
1 = 1 and (1)6 = 1 68 × 93 × 253 ]2 # 3g8 × 93 × ]5 2g3
3. 3 3 = 3 3
3 × 4 × ]5 # 3g3
9 9 9
 6  5 6 5
3 × 4 × 153
5. False as   ×   =  × 
2 × 3 × ]3 g × 56
 7  4  7 4 8 8 2 3
12 7 5
II. 1. (5 ÷ 5 ) × 5 = 5 12 – 7
× 55 = 3 3
3 × 4 × 33 × 53
= 55 × 55 = (5 × 5)5 28 × 314 × 56
= (5 × 5) × (5 × 5) × (5 × 5) × = 3 + 3 6 3 = 28–6 × 314–6 × 56–3
3 ×2 ×5
(5 × 5) × (5 × 5) = 9765625
= 22 × 38 × 53
 7 7   7   7 7
4 5 7 4+5
7
2.   ×    ÷   =   ÷ 
= 4 × 6561 × 125 = 3280500
 9   9    9   9    9  64 × 52 × x 7 26 × 52 × x 7 − 4
4.
=
 7  7
9
 7
7 9−7
 7 49
2 103 × x 4 ( 2 × 5)3
=   ÷  =   =  = 26 × 5 2 × x3
 9  9  9  9 81 = = 26–3 × 52–3 × x3

23 × 53
 p6  5 0
3.  4  × p × p = p
p 
6− 4
× p5 × 1 ( ) 3 −1
= 2 × 5 × x = x
3 8 3
5
= p2 × p5 = p2+5 = p7 5. 21 3
× 10 5
× 125
=
(3 × 7)3 × ( 2 × 5)5 × 53
34 × 53 × 4 81 53 25 × 33 × 58 25 × 33 × 58
4.
= × ×4 33 × 7 3 × 25 × 55 × 53
5 × 81 81 5 =
2 5 × 3 3 × 58
= 53–1 × 4 = 52 × 4 = 25 × 4 =100 58
5. (–3)3 × (–10)3 = –27 × (–1000) = 27000 = 73 × 8 = 73 = 343
5
6. [(33)2 × 26] × 76 = [33 × 2 × 26] × 76 −3
 
= [36 × 26] × 76 6. ( −3)−3  =  1 3 
−3

 
= (3 × 2)6 × 76 = 66 × 76  ( −3) 
= (6 × 7)6 = 426. = [(–3) ] = (–27)3 = –19683
3 3

210 Mathematics–7
IV. 1. [7294 ÷ 7293] ÷ 36 = [7294 – 3] ÷ 36 –1 –125
# 125
= 729 ÷ 36 = 36 ÷ 36 = 36 – 6 = 30 = 1
= 64 + 1 = 64 + 1
2. 6x = 216 ⇒ 63 = 63 ⇒ x = 3 1 25
25 #
⇒ 53x – 8 = 53 × 3 – 8 = 59 – 8 = 51 = 5 16 16
(–3) 3 125 –125 16 –5 –1
# = # +1 = +1 =
43 27 1 –1 + 1 64 25 4 4
3.
+
p 3 18
3 16
3 18–16
3 2
= d n 'd n = d n = d n
5 2
9 1
# 4.
3 2
16 q 4 4 4 4
p 2 3 22 3 2#2 3 4
\ e o = =d n G = d n = d n
–27 1 125
# 1
64 27 1 q 4 4 4
= + 4
25 9 1
1 3 81
# = 4 =
9 16 4 256

13.3 Decimal Number System


• Let us write 592764 in the expanded form.
592764 = 5 × 100,000 + 9 × 10,000 + 2 × 1,000 + 7 × 100 + 6 × 10 + 4 × 1
= 5 × 105 + 9 × 104 + 2 × 103 + 7 × 102 + 6 × 101 + 4 × 100
Observe how the exponents of 10 start from a maximum value of 5 and go on decreasing at
every step from the left to the right up to 0.

Expressing large numbers in the standard form


Any number can be expressed as a decimal number between 1.0 and 10.0, i.e., 1 ≤ x < 10
multiplied by a power of 10. Such form of a number is called its standard form or scientific
notation.
Example: The distance between Earth and Moon is 3,84,000,000 m = 3.84 × 108 m

Exercise 13.3
I. Very Short Answer Type Questions (1 Mark)
A.  Answer the following.
1. Express in expanded form using standards form:
8605734
2. Write the number from the given expanded form
6 × 106 + 4 × 104 + 3 × 103 + 2 × 101 + 9
B.  Fill in the blanks.
1. 3.95 × 10....... = 39500000
2. 560700000 = ............... × 108

Exponents and Powers  211


II.  Short Answer Type Questions–I (2 Marks)
1. Compare: 2.7 × 1012; 1.5 × 108
2. Express the number appearing in the following statement in standard form.
The distance between Earth and Moon is 384,000,000 m.
3. In the standard form, a large number can be expressed as a decimal number between 0 and
1 multiplied by a power of 10. Justify your answer.
4. Express in standard form: 6 0230000000000000000000

III.  Short Answer Type Questions–II (3 Marks)


1. The temperature of the Sun varies from 10,900°F on the surface to 27 billion °F at its core.
Write these in scientific notation. Write two benefits of sunlight. [Value Based Questions]
2. The needle of a record player moves 7.2 cm when playing a particular record. If the record
makes 900 revolutions, find the width of the grove. Give your answer in standard form.

IV.  Long Answer Type Questions (4 Marks)


1. Geography
Given below are areas of five deserts of the world. Express the information in scientific
notation and rearrange in descending order of their size.
S.No. Desert Country Area in sq. km.
1. Great Victoria Australia 647,500
2. Gibson Australia 155,400
3. Sahara North Africa 8,598,800
4 Kalahari South Africa 932,400
5. Thar India 199,430
 [Multidisciplinary Questions]
2. The star Sirus is about 8.1 × 1013 km from the Earth. Assuming that light travels at
3.0 × 105 km per second, find how long does it take for light from Sirus takes to reach the
Earth.

Answers and Hints


I. A. 1. 8 × 10 6 + 6 × 10 5 + 5 × 10 3 Hence, 2.7 × 1012 > 1.5 × 108
+ 7 × 102 + 3 × 101 + 4 × 100 2. 3.84 × 108m

2. 60,00,000 + 40,000 + 3000 + 20 + 9 3. False. In standard form a large number is
= 6043029 expressed as a decimal number between
B. 1. 7 2. 5.607 1 and 10 multiplied by a power of 10.
4. 60230000000000000000000
II. 1. 2.7 × 10 = 2.7 × 104 × 108
12
= 6023 × 1019
= 27000 × 108
= 6.023 × 103 × 1019 = 6.023 × 1022
Now, We have 27000 × 108 , 1.5 × 108
III. 1. Temperature at the surface of the Sun
27000 > 1.5 = 1.0900 × 104°F

212 Mathematics–7
Sun’s temperature at the core = 27 4 Kalahari South 9.32400 × 105
billion °F Africa
= 27 × 1000000000 = 27 × 109 °F 5. Thar India 1.99430 × 105
= 2.7 × 1010 °F Descending order:
Sunlight gives us Vitamin D. Sunlight 8.598800 × 106, Sahara,
also helps plants, which give us fruits 9.32400 × 105, Kalahari,
and vegetables.
6.47500 × 105, Great Victoria,
2. Distance moved by record player needle 1.99430 × 105, Thar,
= 7.2 cm
1.55400 × 105, Gibson
Number of revolutions = 900
2. Speed of light = 3.0 × 105 km/sec
7.2 cm
Width of the grove = Distance = 8.1 × 1013 km
900 Distance 8.1 # 1013
72 1 1 8#1 Time = = seconds
Speed 3.0 # 105
= × 2 = 8 # 1+2 =
9 # 10 10 10 103
= 2.7 × 1013 – 5 seconds
–3 –2
= 8 × 10 = 0.8 × 10 cm
= 2.7 × 108 seconds
IV. 1. Geography
Since 1 hour = 3600 seconds
S.No. Desert Country Area in
sq. km. = 3.6 × 103 seconds
1. Great Australia 6.47500 × 105 2.7 × 108
\ 2.7 × 108 seconds =
Victoria 3.6 × 103
3 3
2. Gibson Australia 1.55400 × 105 = # 108 – 3 = # 105
4 4
3. Sahara North 8.598800 × 106 1
= 0.75 × 105 = 0.75 × 10 × 105 ×
Africa 10
= 7.5 × 105 – 1 = 7.5 × 104 hours.
qqq

Exponents and Powers  213


14 Symmetry
Topics Covered
14.1 Lines of Symmetry 14.2  Rotational Symmetry
14.3 Line Symmetry and Rotational Symmetry

14.1  Lines of Symmetry


• If we fold a figure in half such that the two halves match exactly, then we say that the figure
is symmetrical about the line of fold. The line of fold is called the line of symmetry.
• Two points A and A′ are said to be symmetric with respect to a line l, if the line l is
perpendicular bisector of line segment AA′.
• A figure can have one or more lines of symmetry.
• Number of lines of symmetry of a regular polygon is equal to the number of its sides.

Shape Number of lines of symmetry


Scalene triangle Not symmetric
Isosceles triangle 1
Equilateral triangle 3
Rectangle 2
Square 4
Rhombus 2
Kite 1
Isosceles trapezium 1
Circle Infinite

Exercise 14.1
I. Very Short Answer Type Questions (1 Mark)
A. Answer the following.
1. How many number of lines of symmetry an angle has?
2. What is the line of symmetry of an angle?
3. A line of symmetry divides a figure into how many congruent parts?
4. How many lines of symmetry a regular hexagon has?

214
For Q5 and Q6: Draw the axis of symmetry for the figures.

5. 6.

In Q7-Q8 part of geometrical figure is given. Complete the figures so that the dotted
line in each case is a line of symmetry of the completed figure.

7. 8.

B. Say True or False.


1. A
n angle has one line of symmetry.
2. Two parts of figure with a line symmetry are mirror images of each other.
3. The number of lines of symmetry of a regular polygon is equal to the number of angles of
the polygon.
4. A circle has four lines of symmetry.

II.  Short Answer Type Questions–I (2 Marks)


1. Draw the axes of symmetry for the figure given below:

Justify whether True or False.


2. Angle of rotation is a divisor of 360°.
3. Less the order of rotational symmetry, less the angle of rotation.
4. The order of rotational symmetry of a regular pentagon is 10.

III.  Short Answer Type Questions–II (3 Marks)


1. How many diameters a circle has? Is the diameter of a circle also its axis of symmetry?
For Q2 and Q3: Identify multiple lines of symmetry, if any, in each of the following
figures:

Symmetry  215
2. 3.

4. Draw the reflection of the given shape in the given mirror line (shown as dotted line).

IV.  Long Answer Type Questions (4 Marks)


1. Copy the diagram and complete each shape to be symmetric about the mirror line(s):

(i) (ii)

(iii) (iv)

216 Mathematics–7
2. Draw a reflection in the given mirror line.

Mirror line

The reflection of each part is marked directly ..................... and .....................distant from
the mirror line.

Answers and Hints


I. A. 1. One 2. True, as this means number of times the

angle divides 360°.
2. Line of symmetry of an angle is its
3. False, as less the order of rotation more
angle bisector.

3. Two 4. 6the angle of rotation.
4. False, as order of rotational symmetry
of a regular polygon is equal to the
number of sides it has.
5. 6. III. 1. A circle has infinite number of
diameters. Yes, the diameter of a circle
is also its axis of symmetry.

7.

2.

8. 3 lines l, m and n.

B. 1. True 2. True 3. True 3.


4. False

II. 1.
2 lines l and m.

Symmetry  217
4.
(iii)

(iv)

2.

IV. 1. (i)


(ii)
Opposite, equally

14.2  Rotational symmetry


• If a shape can fit exactly into itself after a certain rotation (not a full rotation) about a fixed
point, then it is said to have rotational symmetry.
• The fixed point is called the centre of rotation.
• The number of times a shape will fit into itself in one complete turn is called the order of
rotational symmetry.
• Rotation means turning a figure through an angle about a fixed point.
• Direction of rotation is assumed to be anti-clockwise if not mentioned.
• Rotation is specified by the angle of rotation, and the centre of rotation.
• A half-turn means rotation by 180°. A quarter-turn means rotation by 90°.
• Every figure has a rotational symmetry of order 1. (i.e., a rotational symmetry of angle 360°).
In such case it is considered that the figure has no rotational symmetry.

218 Mathematics–7
Example: This figure has a rotational symmetry of order 4.

Solution. As this needs to be turned to through one-fourth of a complete turn to look the
same, then it will need three more such turns to return to its original position.
Therefore, starting from the original position, it takes four turns, each one-fourth
of a revolution to return to its original position.

Exercise 14.2
I. Very Short Answer Type Questions (1 Mark)
A. Answer the following.
1. What is rotational symmetry?
2. What is centre of rotation?
3. What is angle of rotation?
4. What is the order of rotation?
5. What is the order of rotational symmetry of the following letters ‘H’ ‘N’ ‘S’ and ‘Z’?
6. What is the order of rotational symmetry of a regular polygon of n sides?
7. What will be the order of rotational symmetry of a figure which has no line of symmetry?
8. What is the angle of rotational symmetry for the letter ‘S’?
9. What is the angle of half a rotation?
10. What is the order of rotational symmetry of  
 ?

B. Fill in the blanks.
1. In the word ‘SOAR’ the letter ........... has a rotational symmetry of order 2.
2. The angle of rotation for a figure with rotational symmetry of order 4 is ........... .
3. The fixed point through which a figure is rotated is called its ........... .
4. ........... triangle has a line symmetry but no rotational symmetry.
5. ........... triangle has both type of symmetry, line and rotational.
6. Order of rotational symmetry of a circle is ........... .
7. A square has a rotational symmetry of order ........... .
8. Every shape has rotational symmetry of order ........... .
9. Rotating a figure through 90° clockwise means rotating it through ........... anti-clockwise.

Symmetry  219
C. Say True or False.
1. Order of rotational symmetry of a semicircle is two.

II.  Short Answer Type Questions–I (2 Marks)


1. Write the order of the rotational symmetry of the given figure.

For Q2-Q5: State the order of rotational symmetry.

2. 3.

4. 5.

For Q6-Q9: Which of the following figures have a rotational symmetry about the marked
point?

6. 7.

8. 9.

10. An isosceles triangle has a rotational symmetry of order 2. Justify whether true or false.

220 Mathematics–7
III.  Short Answer Type Questions-II (3 Marks)
For Q1 and Q2: Find the order of rotational symmetry of the flower shown below:

1. 2.

3. Give the order of rotational symmetry of the figure given below.

IV.   Long Answer Type Questions (4 Marks)


1.
Give the order of rotational symmetry for each figure:

(i) (ii)

Answers and Hints


I.  A. 1. If a shape can fit exactly into itself 5. H has rotational symmetry of order 2
after a certain rotation (not a full N has rotational symmetry of order 2
rotation) about a fixed point, then it S has rotational symmetry of order 2
is said to have rotational symmetry.
Z has rotational symmetry of order 2
2. The fixed point about which a figure is
6. Order of rotation of symmetry of a
rotated is called its centre of rotation.
regular polygon of side n is n.
3. The angle through which a figure is
rotated is called its angle of rotation. 7. 0 8. 180° 9. 180°
4. The number of times a shape will fit 10. 4

into itself in one complete turn is called B. 1. S 2. 90°
the order of rotational symmetry. 3. Centre of rotation 4. Isosceles

Symmetry 221
5. Equilateral 6. Infinite 7. 4 7. Yes 8. No 9. Yes
8. 1 9. 270° 10. False, as an isosceles triangle has
C. 1. False rotational symmetry of order one.
II. 1. Five 2. 3 3. 4 III. 1. 3 2. 11 3. 1
4. 5 5. 4 6. Yes IV. 1. (i) 2 (ii) 3

14.3  Line Symmetry and Rotational Symmetry


• You have observed many shapes and their symmetries in the previous two exercise. By now
you would have understood that some shapes have only line symmetry, some have only
rotational symmetry and some have both rotational and line symmetry,
Example. A circle has line and rotational symmetry both.
1. We know that the each diameter of the circle is its line of symmetry and a circle has
infinite number of diameters, hence infinite number of lines of symmetry.
2. Circle can also be rotated through any angle and retain its shape. So, it has rotational
symmetry around its centre for every angle.

Exercise 14.3
I. Very Short Answer Type Questions (1 Mark)
A. Answer the following.
1. Define rotational symmetry.
2. Name a quadrilateral that has four lines of symmetry as well as rotational symmetry of
order 4.
3. Do the following figures have rotational symmetry of the same order? If yes, state the
order.

4. Does the adjacent figure have both line and rotational symmetry?
5. Name a quadrilateral which has line and rotational symmetry both.
B. Fill in the blanks.
1. ........... and ........... are the capital letters of English alphabets that have no line of symmetry
but they interchange to each other when rotated through 90°.

222 Mathematics–7
C. Say True or False.
1. The letter H has both line and rotational symmetry.
I I.  Short Answer Type Questions–I (2 Marks)
1. Draw a triangle with no line of symmetry and rotational symmetry of order 1.
2. Draw axis of symmetry of an isosceles triangle. Does this have rotational symmetry also?
What other name can you give to the axis of symmetry in this case?
3. The order of rotational symmetry of a regular pentagon is 10.

III.  Short Answer Type Questions–II (3 Marks)


1. List all letters of English alphabet which have both line symmetry and rotational symmetry.
2. Which of the following figures have both line symmetry and rotational symmetry?

Also, draw the line of symmetry and write order of rotational symmetry.
3. For each of the following shapes state:
(i) The number of lines of symmetry (ii) The order of rotational symmetry


(a) (b)
IV. Long Answer Type Questions (4 Marks)
1. Draw the object and its image under rotation given. In each case take X as the centre of
rotation.

(a)
(b)
X

180° clockwise

Symmetry  223
(c) (d)

X
120° clockwise
X
90° clockwise

For each of the above shapes state:


(i) The number of lines of symmetry (ii) The order of rotational symmetry

Answers and Hints


I.  A. 1. If a shape can fit exactly into itself
after a certain rotation (not a full
rotation) about a fixed point, then it
is said to have rotational symmetry.
2. Square
3. Yes, order 3
4. No, has only line symmetry
5. Square 3. (a) (i) 0 (ii) order 2
B. 1. Z and N (b) (i) 3 (ii) order 3
C. 1. True
IV. 1. (a) X (b)
II. 1. Hint: Draw a scalene triangle
2. No, Median or altitude X
3. False, as order of rotational symmetry
(i) 1 (ii) 1 (i) 1 (ii) 1
of a regular polygon is equal to the number
of sides it has (c) (d)
III. 1. H, O, X, I
X
2. Figure (i)
Four lines of symmetry. Rotational X
symmetry of order 4. (i) 3 (ii) 3 (i) 4 (ii) 4
qqq

224 Mathematics–7
15 Visualising Solid Shapes
Topics Covered
15.1 Plane Figures and Solid Shapes 15.2  Drawing-Solids on a Flat Surface
15.3 Visualising Solid Objects

15.1  Plane Figures and Solid Shapes


• Plane figures have two dimensions (2-D), i.e., length and breadth.
• Solid shapes have three dimensions (3-D), i.e., length, breadth and height.
• Solid shapes like cubes, cuboids, cylinders, cones, spheres, prisms and pyramids, etc., occupy
space and are three dimensional.
• A 3-D shape bounded by six rectangular faces is called a cuboid. It has 8 vertices and
12 edges.
• A cube is a cuboid with equal length, breadth and height.
• A cylinder is a 3-dimensional shape with two plane faces (circles) and one curved surface
with no vertex.
• A cone is 3-D shape having curved surface, a plane face (circle) and a vertex.
• A 3-D shape with two opposite faces that are identical polygons, and whose other faces are
parallelograms is known as a prism.
• A pyramid has a polygon as a base and all other faces are triangles.
• A sphere is a 3-dimensional shape bounded by one curved face. It has no edges or vertices.

Cube Cuboid Cylinder

Cone Sphere Triangular Prism


• The corners of a solid shape are called its vertices.
• Edges are the line segments/curves of the figure which form its skeleton.

225
• Face of a solid is its flat surface.
• A net is the outline of a solid that can be folded to make
the solid. In other words, a net is a sort of skeleton-
outline in 2-D, which, when folded, results in a 3-D
shape.
• Each solid shape can be drawn on a flat surface. This
is called a 2-D representation of a 3-D solid shape.
If we join together six identical squares, edge to edge, we get a cube or the outside surface
of the cube.

Suppose we want to make a cube out of a cardboard or a piece of paper. We need a pattern
giving us the shape of the cardboard or the piece of paper to make the cube.
Figure (i) below shows, the shape of the pattern of six squares. When this shape is folded
along the edge, a cube is formed.
Top Back
Back

Side Side
Side Base Side Top
Base
Front Front

(i) (ii)

A cross plan such a shown in figure (i) above, which can be folded to form a cube is called
the net of the cube.
Cube
A cube has 8 vertices.
In the figure, the vertices are
A, B, C, D, E, F, G, and H.
A cube has 12 edges.
In the figure, the edges are
AB, BC, CD, AD, EF, FG, GH, HE, BF,
AE, CG and DH.
A cube has 6 faces – top, bottom, front,
back and 2 side faces.
In the figure, the faces are
ABCD, EFGH, ABFE, DCGH, BCGF and
ADHE.

226 Mathematics–7
Cuboid
A cuboid has 8 vertices, 12 edges and 6 faces. Using the adjoining figure, name all the
vertices, edges and faces of the cuboid.

Cylinder
A cylinder has 3 faces, 1 curved face and 2 flat faces. It has 2 curved edges. It has no vertex.

Cone
A cone has 1 vertex, 1 curved edge and 2 faces (1 curved and 1 flat face).

Sphere
A sphere has 1 curved face. It has no vertex and no edge.

Visualising Solid Shapes  227


riangular Prism
T
• A triangular prism has 6 vertices.
• In the figure, the vertices are A, B, C, D, E and F.
• A triangular prism has 9 edges. In the figure, the
edges are AB, BC, AC, DE, EF, FD, BD, CF and
AE.
• A triangular prism has 5 faces, 2 triangular and 3
rectangular.
• In the figure, ABC and DEF are triangular faces and
ABDE, BDFC, AEFC are rectangular faces.
Triangular Pyramid
• A triangular pyramid has 4 vertices.
• In the figure, the vertices are O, A, B and C.
• It has 6 edges. In the figure, the edges are AB, BC, AC,
OA, OB and OC.
• A triangular pyramid has 4 triangular faces.
• In the figure, the faces are ABC, OAB, OBC and OAC.
Note : A triangular pyramid is also called a tetrahedron
(tetra = four and hedron = face)
Square Pyramid
• A square pyramid has 5 vertices. In the figure, the
vertices are O, A, B, C and D.
• A square pyramid has 8 edges.
• In the figure, the edges are AB, BC, CD, AD, OA,
OB, OC and OD.
• It has 5 faces, 4 triangular faces and 1 square face. In
the figure, ABCD is a square face and OAB, OBC,
OCD and OAD are triangular faces.
Net of a Cube
• There are another arrangement of six squares that can
be folded up to make a cube. For example, a cube
can also be made from the nets shown below.

Top Side

Side Back Base Front

Back Base Front Side

Side Top

(a) (b)

228 Mathematics–7
Net of a Cuboid
• The figure below shows the net of a cuboid of dimensions 6 cm × 4 cm × 3 cm.

Net of an Open Cylinder


• The net for a cylinder without a top and a bottom is shown. The length of the net is equal
to the circumference of the cylinder.

Exercise 15.1
I. Very Short Answer Type Questions (1 Mark)
A. Answer the following.
1. Name the plane shape(s) needed to draw the net of a cube.
2. Label the net given below.

3. Name a solid that has 4 faces.


4. What is the other name of plane figures?
5. What is the number of faces a rectangular prism has?
6. What is the number of edges a triangular prism has?
7. What is a net?

Visualising Solid Shapes  229


B. Fill in the blanks.
1. Flat surface of a ................ is called its face.
2. ................ is solid with no vertex. 3. A square pyramid has ................ faces.

4. Cuboid is a ................ . 5. is a net of ................


C. Say True or False.
1. Cylinder is a solid with no vertex.
2. A rectangular pyramid has 5 rectangular faces.
3. A pyramid has only one vertex. 4. A prism has three bases.
II.  Short Answer Type Questions–I (2 Marks)
1. Which 3-D shape does the given net represent?

2. Which solid shape has a circle as its base and a point as its top?
3. Draw the net of a tetrahedron.
Justify whether True or False.
4. The solid shapes are two-dimensional.
5. Circle is a 3-D shape whereas a sphere is a 2-D shape.
III.  Short Answer Type Questions–II (3 Marks)
1. Match the nets with appropriate solids:

(i) (a)

(ii) (b)

(c)
(iii)

(iv) (d)

230 Mathematics–7
2. Which of the following arrangements of six squares are the nets of a cube?

Side

Back Base Front


(a) (b)
Side

Top

Top Back Back Side Back Back Side


(c) (d)
Side Front Side Front Top

3. Sketch the net of a triangular prism whose ends are right-angled triangles of sides
3 cm × 4 cm × 5 cm and whose length is 6 cm.
IV.  Long Answer Type Questions (4 Marks)
1. Crazy cubes
Make four cubes with paper and tape, numbering each face as shown.

The goal is to line up the cubes so that 1, 2, 3 and 4 can be seen along the top, bottom,
front and back of the row of cubes. They can be in any order, and the numbers do not
have to be right side up.
2. The dimensions of a cuboid are 3 cm × 2 cm × 1 cm.
(i) How many faces does this cuboid have?
(ii) Sketch all faces of the cuboid showing their measurements.
(iii) Draw a net of the above cuboid.
3. Do you know that numbers of a die on opposite faces add up to 7. Draw a net of the
dice and number the correct faces.
4. Each net is indicated by a letter. Each figure of a solid is indicated by a number. Which
of the lettered net best matches with a numbered solid?

Visualising Solid Shapes  231


C
A 2
B
1

E 4
3

D F

5 6

G I

H 7
9 8

Answers and Hints


I. A. 1. Square 2. Tetrahedron IV. 1. Do yourself 2. (i) Six
3. Triangular pyramid (ii)
 4. Two dimensional figures 5. Six
6. Nine
7. A net is the outline of a solid in 2-D
that can be folded to make the solid.
B. 1. solid 2. sphere 3. five
4. parallelopiped 5. prism
C. 1. False 2. False 3. False
4. False
II. 1. Cuboid 2. Cone 3.

4. False. Solid shapes are three dimensional


as they occupy space.
5. False. Circle is 2-D shape whereas a
sphere is a 3-D shape.
III. 1. (i)—(b), (ii)—(c), (iii)—(d), (iv)—(a) 3.
2. a, b
6 cm
4. A—2; B—3; C—7; D—5; E—1; F—4;
G—6; H—9; I—8;
4c

3.
5 cm
m

m
3c
3 cm

232 Mathematics–7
15.2  Drawing Solids on a Flat Surface
• Each solid shape can be drawn on a flat surface. This is called a 2-D representation of a
3-D solid shape. Sketches of a solid are of two types:
(a) An oblique sketch. This sketch does not have a proportional measurement.
(b) An isometric sketch.
This kind of a sketch is drawn on isometric dot paper. In this sketch of the solid, the
measurements are proportional.
• On an isometric drawing, a 3-D shape will make the shape appear slightly distorted. On an
isometric drawing of a 3-D shape, lengths which are equal on the shape are also equal on
the drawing.
• While drawing 3-D shapes on isometric graph paper or dot paper, vertical edges should be
drawn as vertical lines.
Following steps are taken to draw a cube on an isometric dot paper.

Steps involved while drawing


a cuboid of dimensions 4 cm ×
1 cm × 3 cm on an isometric
dot paper as shown here:
Using similar steps we can
draw a cube on an isometric
graph paper. Cuboids can also
be drawn in a similar way.

Cube of edge 3 cm Cuboid of dimensions 3 cm × 2 cm × 1 cm

Visualising Solid Shapes  233


Exercise 15.2
I. Very Short Answer Type Questions (1 Mark)
A. Answer the following.
1. Draw a pattern of isometric dot paper.
2. How can we represent a 3-D object in 2-D?
B. Say True or False
1. An oblique sketch of a solid has proportional measurements.
2. A pyramid is a solid figure.
II.  Short Answer Type Questions-II (3 Marks)
1. Three cubes each with 2 cm edge are placed sidesby side to form a cuboid. Sketch an
oblique or isometric sketch of this cuboid.
2. Make an oblique sketch for each one of the given isometric shapes.

Answers and Hints


I.  A. 1. Do it yourself II. 1.
2. By drawing its net.
B. 1. False 2. True
2. Do it yourself.

15.3  Visualising Solids Object


• A solid can be viewed in many ways:
(a) By cutting or we can say slicing the shape to obtain a cross section of the solid.
(b) Looking at the shape from different positions, the front view, the side view and the top
view.
Viewing different sections of a solid
• Here is a sliced cake. Its each piece is like a
cuboid. It has been sliced vertically. The face
of each piece is a square. This face is called
a cross section of the bread. The shape of the
cross section depends on the solid and on the
way of slicing. For example, if you slice a

234 Mathematics–7
whole cake with a knife obliquely, its cross section will not be square and if you slice a
round cake vertically, its cross section will be a circle.
You must have observed that if a coke can is kept just below a bulb its shadow will be a
circle and if a dice is kept its shadow will be a square.
Also, if we stand in the Sun our shadow size keeps on changing, depending on the position
of Sun.
Similarly, sometimes the shadow of a cube can be in the shape of a rectangle.
Thus, we see that we get different views of an object when seen from different angles.
Examples

Let us look at some solved examples.


Example 1. Look at the following figure and count the number of cubes.

Solution. Six
Example 2. Draw front, top and side (from right) view of the figure given below.

Solution. (a) Front  (b) Top   (c) Side

Visualising Solid Shapes  235


Exercise 15.3
I. Very Short Answer Type Questions (1 Mark)
A. Answer the following.
1. Draw front view of the solid shown here.

2. Name the shape of the cross section.


B. Fill in the blanks.
1. The number of cubes in the figure is ............................ .

(b)

2. Number of cubes in the figure given below is ............................ .

II.  Short Answer Type Questions–I (2 Marks)


1. Name the cross section obtained with a horizontal cut of a square pyramid.
2. Draw the front, side and top view of a cylinder.
III.  Long Answer Type Questions (4 Marks)
1. Find the number of cubes in each of the following figures and in each case give the top,
front, left side and right side view (arrow indicating the front view).

(i) (ii)
(iii) (iv)

236 Mathematics–7
(v)  (vi)   (vii)  (viii)

2. Draw the plane view, front and side elevation of the solid given below:

Answers and Hints


II. 1. Rhombus
I. A. 1. 2. Circle

2.

B. 1. 10 2. 9


III. 1. Top Front Left Right

(i), 6;

(ii), 8;

(iii), 7;

(iv), 8;

(v) , 6;

Visualising Solid Shapes  237


(vi), 8;

(vii), 7;

(viii), 8;
2. Plane Front Side

qqq

238 Mathematics–7
Term-I
Periodic Test-1 Maximum Marks: 20
Based on Chapters
1. Integers 2. Fractions and Decimals 3. Rational Numbers
4. Exponents and Powers

Section-A
1
1. Find of 24. (1)
4
2. Write 5384000 in standard form. (1)
3. Find 783 × (–97) + (–783) × 3 using some quicker grouping. (1)
3 x
4. Find the value of x if the rational number and are equivalent. (1)
7 42

Section-B
( −6) 0
− ( 2)
0

5. Find the value of 0 . (2)


 −3 
 
4
9 8 2
p  −2   −2   p
6. If =   ÷   , find the value of   .  (2)
q  3  3  q
2   1  1 1  
7. Simplify: 3 −  4 + 2 −  2 of 1 ÷ 1 + 1    (2)
3   2  3 9  

Section-C
8. Complete the magic square so that the product of the numbers in any row, column or
diagonal is –1728. (3)

9. Find x if 8 × [(–5) + x] = 8 × (–5) + 8 × 10. (3)

Section-D
10. Sourav bought 750 ml milk at ` 19 per litre, 1 l 500 ml cooking oil at ` 48.00 per litre.
He gave a ` 100 note to the shopkeeper. What change did the shopkeeper give him back?(4)

Test Papers  239


Term-I
Periodic Test-2 Maximum Marks: 20
Based on Chapters
1. Simple Equations 2. Algebraic Expressions 3. Symmetry
4. Visualising Solid Shapes

Section-A
1. Write “twice cube of a number x subtracted from five times the sum of y and z. (1)

2. Is the equation x2 + 2 = 18 a linear equation? (1)


3. Find the value of a if the value of 2x2 + x – a equals 5, when x = 0 (1)
4. Write an equation for the statement ‘8 times a number is 7 more than 9’. (1)

Section-B
5. Draw as many lines of symmetry as provide for the figure given below. (2)

6. Copy the figure given below and mark the approximate centre of rotation State the
order of rotational symmetry. (2)

7. Draw front, side (from right) and top views of the figure given here. (2)

Section-C
3x − 5 x + 4
8. Solve: =  (3)
2 3

240 Mathematics–7
9. (i) Which of the following figures has reflection, rotational or both reflection and
rotational symmetry?

(a) (b) (c)

(3)
(ii) Draw the lines of reflection symmetry.
(iii) Mark the centre of rotation and write the order of rotational symmetry.

Section-D
10. Name the following figures
(i) A figure which has 8 vertices, 6 congruent faces and 12 edges.
(ii) A figure whose base is a circle and has one vertex.
(iii) A solid with square base, 5 faces, 8 edges and 5 vertices.
(iv) A solid with 5 faces, 2 congruent triangles and 3 rectangles. (4)

Test Papers  241


Time: 3 hrs. Half-Yearly Test Paper-1 Maximum Marks: 80
Based on Chapters
1. Integers 2. Fractions and Decimals 3. Simple Equations
4. Lines and Angles 5. Rational Numbers 6. Algebraic Expressions
7. Exponents and Powers 8. Symmetry 9. Visualising Solid Shapes
General Instructions
1. All questions are compulsory.
2. Question paper consists 4 sections and 30 questions.
3. Questions 1-6 in Section A carry 1 mark each, questions 7-12 in Section B carry 2 marks
each, questions 13-22 in Section C carry 3 marks each and questions 23-30 in Section D carry
4 marks each.
Section-A
1. The product of 7 negative integers is a negative integer. Say True or False. (1)
5 x
2. If = , then find the value of x. (1)
7 28
3. Express the following in exponential form. (1)
8 × 8 × 4 × 4 × 4 × (– 6) × (– 6)
4. What is the order of rotational symmetry of an equilateral triangle? (1)
5. Look at the figure and count the number of cubes.  (1)

6. Multiply 2.3 by 4. (1)


Section-B
7. The sum of two integers is –100. If one of them is – 46, then find the other integer (2)
8. Arrange the following decimal in the ascending order. (2)
81.201, 30.1, 12.01, 13.10, 12.14, 10.65
21 14
The product of two rational numbers is − . If one of the numbers is
9. . Find the
81 27
other number. (2)

 3   16  2  
3 3


10. Simplify the following:    × −  (2)
 4   27  3  

242 Mathematics–7
11. Collect the like terms and simplify (2)
1 5
3x2 – x + 7 − 8 + x − 2 x + 5 x + 4 x + 2
2 2

2 3
12. An angle is equal to 5 times to its complement. Determine its measure. (2)
Section-C
13. Evaluate, using suitable properties:(3)
(i) 625 × (–35) + (– 625) × 65 (ii) (– 67) × 19 + (– 67) × 81
14. In a class test containing 12 questions, 5 marks are given for every correct answer and
(–2) marks are given for every incorrect answer and 0 marks are given for questions
not attempted.
(i) Aruna gets 7 correct and 5 incorrect answers. What is her score?
(ii) Tanshi attempted all questions but gets only 5 correct answers. What is her score? (3)
15. Reena plants 4 saplings in a row in her garden. The distance between the adjacent saplings
3
is m. Find the distance between the first and the last saplings. (3)
4
16. The cost of 1 m of cloth is ` 24.75. Find the cost of 3.6 m of cloth. (3)

17. Simplify:  1 + 4  ×  2 + −1  (3)


 3 9   5 15 
4
 2 216
 −  ×
3 125
18. Simplify: 2
 (3)
 6 4
  ×
5 9
19. If A = 5x3 + 3x2 – 4x + 1, B = 3x3 + 5x2 + 3x – 8 and C = 6x3 – 4x2 – 7x + 3, find
(A + B) – C. (3)
20. Find three odd consecutive numbers whose sum is 27. (3)
21. For the given figure
(i) mark the centre of rotation.
(ii) state the order of symmetry. (3)
22. Draw front, side (from right) and top view of the figure: (3)

Test Papers  243


Section-D
23. The temperature at noon was 12° C above zero. It descends at the rate of 2°C per hour
until mid-night. At what time would the temperature be 10°C below zero. What would
be the temperature at mid-night?(4)
3 1
24. Riya spends of income on household expenses and on personal expenses. If her
5 7
monthly income is ` 35,000. Find her savings. (4)
40
25. Find the value of ÷ (–20).
98
(i) Express as a rational number in the standard form.
(ii) Find the reciprocal of the number obtained. (4)
26. The star Sirus is about 8.1 × 1013 km from earth. Assuming that light travels at
3.0 × 105 km per second, find how long light from Sirus takes to reach the earth. (4)
27. (i) Find the value of expression when x = 3, y = 5.
3x2y + 2xy – xy2

(ii) Simplify: x – [y {z – (b – c)}]. (4)
28. Solve and check:
(i) 2(3x – 1) = 10
(ii) 5x – 6 = 12 – x (4)
29. Shiren asked Riya to think of a number. Then she asked to double it and add 5 to it. If
the result was 21, what number did Riya think? (4)
30. Find the sum : 0.27 + 0.849 + (–1.32) + 2.350
p
(i) As a decimal number and convert the sum into the form .
q
(ii) By converting them into rational numbers. (4)

244 Mathematics–7
Time: 3 hrs. Half-Yearly Test Paper-2 Maximum Marks: 80

Based on Chapters
1. Integers 2. Fractions and Decimals 3. Simple Equations
4. Rational Numbers 5. Perimeter and Area 6. Algebraic Expressions
7. Exponents and Powers 8. Symmetry 9. Visualising Solid Shapes

General Instructions
1. All questions are compulsory.
2. Question paper consists 4 sections and 30 questions.
3. Questions 1-6 in Section A carry 1 mark each, questions 7-12 in Section B carry 2 marks
each, questions 13-22 in Section C carry 3 marks each and questions 23-30 in Section D carry
4 marks each.
Section-A
1. Fill in the blanks: (1)
[12 + 14] + (– 25) = 12 + __ + (–25)
4
2. Multiply by 10  (1)
9
80
3. Express − with numerator –5. (1)
64
4. Find the value of (–2)4 × (–2)3. (1)

5. Does the following figure have rotational symmetry? If yes, find the order of rotational
symmetry. (1)

6. Write down the number of edges a triangular prism has. (1)

Section-B
3  2 1
7. Simplify: 3 ×  5 − 3   (2)
5  3 6
8.
Find the value of the expression given below using suitable rearrangement. (2)
–217 × 151 + (–217) × 349
3 a
Find a such that −
9. and are equivalent rational numbers. (2)
11 −33
10. Divide 6,561 by 10,000 and express the results in exponential form.  (2)

Test Papers  245


11. Write down the degree of each term of the algebraic expression. Also write the degree of
the expression p2q2 + 2pq2 – p2q + 1  (2)
12. Solve: y – 16 = 4  (2)
Section-C
13. Verify and name the property used: 20 × [(–19) + 5] = [20 × (–19) + 20 × 5](3)
14. Identify true and false statements.
(i) 0 is called the multiplicative identity of integers.
(ii) The greatest negative integer is –1.
(iii) If an integer is added to its additive inverse, the result is the additive identity of
integers. (3)
15. A sum of ` 3486.45 was equally distributed among 33 workers. How much did each
worker get?  (3)
1
16. Find the perimeter of a rectangular field whose length and breadth are 16 m and
2
3
12 m respectively. (3)
4
5 6 3
17. Arrange , and is descending order. (3)
−4 7 14
3 2
 1  2   1  3 
18. Simplify and express the result as a power of 5:    ÷     (3)
 4    4  
19. What should be subtracted from 2x3 – 3x + 6 to get 4x2 – x + 2? (3)
20. Solve: 5 (x + 4) = 25  (3)
21. For the given figure state
(i) The number of lines of symmetry.
(ii) The order of rotational symmetry.
(iii) Also, draw the lines of symmetry. (3)
22. The diagram shown is the net of dice. The number of dots on opposite faces add upto 7.
Write down the number of dots that would appear on each of the faces marked as A, B
and C(3)

B C
A

Section-D
23. Simplify – 8 – [7 – {2 + 5 – (6 – 7 – 3)}] (4)
1 1 1
24. A man sold of his land. He gave of the remaining portion to his son and of the
2 2 3
balance to his daughter. What fraction of his land is left with him? (4)

246 Mathematics–7
−5 1
25. By what rational number should we multiply to get ? (4)
19 5
p
26. Simplify and express the result as :
q

(6.25 + 0.36) – (17.2 – 8.97) (4)
 3 2
  3   −2 
2 3
27. Simplify:  1  −  1  ×  ×   (4)
 2   3    5   3 
28. If A = 3x2 – 4x + 1, B = 5x2 + 3x – 8, C = 4x2 – 7x + 3, find the value of A+ B – C. (4)
29. Solve 3(x + 6) = 4 (2x – 8) and check your answer. (4)
1 1
30. Divide 84 into two parts such that of one part is equal to of the other. (4)
3 4

Test Papers  247


Term-II
Periodic Test-1 Maximum Marks: 20
Based on Chapters
1. Data Handling 2. Comparing Quantities

Section-A
1. If 3, x and 27 are in proportion, then find x. (1)

2. What is 60% of 1 km? (1)


3. The first, third and fourth terms of a proportion are 6, 10 and 25 respectively. Find its second
term. (1)

4. What is the probability of a number when it is chosen from first 8 prime numbers? (1)
Section-B
5. A man borrowed ` 7,200 from a bank for 3 years at 8% p.a. Find the amount he will have to
pay after specified time.  (2)
6. Sarika obtained 432 marks out of 600 and Manohar obtained 525 marks out 750 in annual
examination. Whose performance is better? (2)
7. A shopkeeper bought a saree for ` 425 and sold it at a profit of ` 80. Find the selling price of
the saree. (2)
Section-C
8. The following table shows the interest paid by a company in lakhs. (3)
Year 1995-1996 1996-1997 1997-1998 1998-1999 1999-2000
Interest (in lakhs of rupees) 20 25 15 18 30
Draw the bar graph to represent the above information.
9. The heights of plants (in cm) in a nursery is given below. (3)
Height (in cm) 28 30 32 34 36
Number of plants 36 47 80 58 72
Find the mode of data.
Section-D
10. (i) Are 2.1, 13.5 and 22.5 in a proportion? (4)

(ii) Are the numbers 25, 45 and 81 in continued proportion.

248 Mathematics–7
Term-II
Periodic Test-2 Maximum Marks: 20
Based on Chapters
1. Lines and Angles 2. The Triangle and its Properties 3. Practical Geometry
4. Perimeter and Area
Section-A
1. Find the area of a circle of diameter d. (1)
2. When a transversal cuts two parallel lines, how many pairs of corresponding angles
are formed? (1)
3. In the given figure, l || m and t is a transversal. Find the values of y and z.
t

l
120°

y z
m

 (1)
4. Find the values of x and y in the given figure.
A
x

65° y 130°
B
C 
D
(1)
Section-B
5. The length of a square field is 35 m. What distance will a man walk in going around
it 3 times? (2)
6. One side of a parallelogram is 20 cm and the corresponding altitude is 12 cm. Find
the length of the adjacent side of the parallelogram if the height of the altitude to the
adjacent side is 10 cm. (2)
7. One angle of D ABC is 40° and the other two angles are equal. Find the value of each
equal angle.  (2)
Section-C
8. Construct a right DXYZ in which XY = 3 cm, YZ = 4.5 cm and ∠Y = 90°. (3)
OR
Draw a line segment AB = 5 cm. Take a point P outside AB. Using ruler and set
squares, draw a line parallel to AB and passing through P.

Test Papers  249


9. In the given figure, find –ACD and –AED.
A

25°
E


20° 10

D C B (3)
Section-D
10. A rectangular field is of length 94 m and breadth 32 m. Two roads each of width 2 m
pass through the field such that one is parallel to the breadth and another is parallel to the
length. Find the area of the field (4)
(i) covered by the roads
(ii) not covered by the roads

250 Mathematics–7
Time: 3 hrs. Annual Test Paper-1  Maximum Marks: 80

Based on Chapters
1. Integers 2. Data Handling 3. Simple Equations
4. Lines and Angles 5. The Triangle and Its Properties 6. Comparing Quantities
7. Practical Geometry 8. Perimeter and Area

General Instructions
1. All questions are compulsory.
2. Question paper consists 4 sections and 30 questions.
3. Questions 1-6 in Section A carry 1 mark each, questions 7-12 in Section B carry 2 marks
each, questions 13-22 in Section C carry 3 marks each and questions 23-30 in Section D carry
4 marks each.
Section-A
1. Find the ratio between 80 cm and 2 m.  (1)
2. Express 15.6% as decimals. (1)
3. Find the angle which is equal to its complement. (1)
4. In a right-angled triangle, one acute angle is 30°. Find the other acute angle.  (1)
5. Simplify: (–3) × (–6) × (–7)  (1)
6. Solve: 2x + 4 = 3x – 4 (1)

Section-B
7. If 10 boys consume 3 kg rice in a day, how much rice will be consumed by 15 boys in a
day?  (2)
8.
In an orchard, 30% are apple trees out of a total of 360. Find the number of other trees. (2)
9.
In the figure, ABC is a triangle with ∠B = 60° and ∠C = 50°. PQ is parallel to BC. Find
x and y. (2)

10. In the given figure, find the measures of x and y.  (2)


C

30°

y x 65°
A B D

Test Papers  251


11. The length of a rectangular field is three times its breadth. If the breadth of the rectangle is
1
3 m, find the perimeter of the rectangle.  (2)
2
12. The radius of a circular wheel is 14 cm. How many revolutions will it make to cover a
distance of 2228 m? (2)
Section-C
13. The floral design on the floor of a building consists of 2,800 tiles. Each tile is in the
shape of a parallelogram of altitude 3 cm and base 5 cm. Find the cost of polishing the
design at the rate of `25 per m2.(3)
14. Find the mean of the monthly rents of 30 shops from the following distribution table. (3)
Monthly Rent (in `) Number of shops
4,000 2
10,000 7
12,000 8
20,000 10
28,000 3
15. Simplify: −10 − −5 ÷ 30 − −25  (3)
16. Solve: 5(x – 3) = 4(x – 2).  (3)
17. The ratio of the number of men to that of women in a party is 5 : 4. If the number of men
is 40, find the number of women in the party. (3)
18. Find interest on ` 6,000 lent for a period of 4 years at the rate of 8% p.a. and also the
amount to be paid at the end of the period.  (3)
19. A person donates 10% of his savings to a charitable trust. He distributes the rest among
his three children equally. If the total savings of the person is ` 75,000, find the amount
received by each child. (3)
20. In the given figure, find the values of x, y and z. (3)

y
x

z 55°

21. A tree is broken by the wind, as shown in the figure. If the point from where it broke is
5 m above the ground and touches the ground at a distance of 12 m from its foot, find
the total length of the tree before it broke. (3)

252 Mathematics–7
22. Construct a right DABC, right-angled at B in which AB = 4 cm and BC = 5.4 cm. (3)

Section-D
23. 39 packets of 12 pencils each cost ` 374.40. Find the cost of 52 packets of 10 pencils
each. (4)
24. A fruitseller bought 100 bananas for ` 40. Out of them, 20 bananas were spoiled and
thrown away. He sold the remaining bananas at a profit of 10%. Find the selling price of
bananas.  (4)
25. In the figure, find (i) ∠PRS (ii) ∠PTS, (iii) ∠RTS (iv) ∠PRQ. (4)

26. Construct a DABC in which AB = 5.6 cm, ∠A = 60° and ∠B = 30°. (4)
27. (i) Find the median of first 15 odd numbers.
(ii) The marks obtained by 11 students of a class in a test are given below:
23, 2, 15, 38, 21, 19, 23, 23, 26, 34, 23
Find the modal marks. (4)
28. Calculate the area of the quadrilateral ABCD from the measurements given in figure. (4)

29. Multiply using distributive property 7219 × (–1005). (4)


3x 11
30. Solve +1 = and check your answer. (4)
2 2

Test Papers  253


Time: 3 hrs. Annual Test Paper-2  Maximum Marks: 80

Based on Chapters
1. Integers 2. Data Handling 3. Simple Equations
4. Lines and Angles 5. The Triangle and Its Properties 6. Comparing Quantities
7. Practical Geometry 8. Perimeter and Area

General Instructions
1. All questions are compulsory.
2. Question paper consists 4 sections and 30 questions.
3. Questions 1-6 in Section A carry 1 mark each, questions 7-12 in Section B carry 2 marks
each, questions 13-22 in Section C carry 3 marks each and questions 23-30 in Section D carry
4 marks each.

Section-A
1. Express as ratio: a dozen and a score.  (1)
2. Express 0.051 as percents.  (1)
3. Find the angle which is double of its supplement. (1)
4. Find the measure of all the angles of an equilateral triangle. (1)
5. Find 4399 ÷ (–4399). (1)
6. Verify by substitution that z = 4 is the solution of 2z + 7 = 15. (1)

Section-B
7. The cost of 10 tables is ` 7,500. Find the number of tables that can be purchased with
` 9,000. (2)
8. Ram got 80% marks out of 500. How many more marks are needed to secure 90%? (2)
9.
Find the values of x and y in the figure given below. It is given that l || m and transversal
t intersects them. (2)

254 Mathematics–7
10. In the figure, find the values of x and y. (2)

11. The perimeter of a triangle is 64 cm. Two of its sides measure 15 cm and 24 cm
respectively. Find the length of the third side. (2)
12. A piece of wire in the form of a rectangle with dimensions 12 m by 10 m is bent to
form a circle. Find the diameter of the circle. (2)

Section-C
13. Calculate the area of the shaded portion from the measurements given in the figure.(3)

14. Following are the marks obtained by 25 students of class VII in a test (out of 25 marks)
in mathematics. 
5, 18, 18, 2, 16, 13, 8, 17, 18, 17, 13, 18, 16, 8, 5, 13, 20, 8, 16, 8, 19, 13, 2, 18, 5.
Find:
(i) the maximum marks obtained
(ii) the minimum marks obtained
(iii) the mean of the marks obtained. (3)
15. Verify and name the property
[–16 + 134] + (–200) = –16 + [134 + (–200)]  (3)
x
16. Solve: + 1 = x – 3  (3)
5
17. Find the mean proportional between 25 and 64.  (3)
18. By selling an article for ` 300, Pradeep loses ` 60. At what price must he sell the article
to gain 15%. (3)
19. Sarika obtained 432 marks out of 600 and Manohar obtained 525 marks out of 750 in
annual examination. Whose performance is better? (3)

Test Papers  255


20. In the figure, find the values of x and y. Also, write the name of this special type of
polygon. (3)
D C
x
y

30°

40°
A B

21. Two buildings 30 m and 15 m high stand upright on the ground. If they are 36 m apart,
find the distance between their tops. (3)
22. Construct a triangle ABC in which AB = 7 cm, BC = 3 cm and ∠B = 60°.(3)

Section-D
23. A car travels 275 km in 5 hours.
1
(i) How far will it travel in 8 hours?
2
(ii) How long will it take to travel 495 km? (4)
24. A grocer buys eggs at 10 for `  8 and sells at 8 for `10. Find his gain or loss percent. (4)
25. Show that sum of the exterior angles of a triangle is 360°.  (4)
26. Construct a DABC in which BC = 7 cm, ∠B = 120°, ∠C = 30°. (4)
27. Renu recorded temperature at 3 p.m. for a week as follows.
Days Monday Tuesday Wednesday Thursday Friday Saturday Sunday
Temp. (°C) 40 35 45 40 30 48 38
Draw a bar graph for the above data. (4)
28. The perimeter of a right triangle is 12 cm. Its hypotenuse is 5 cm and base is 4 cm.
Find the area of the triangle. (4)
29. Find the value of –761 × 256 + (–761) × 371 + (–761) × 373 (4)
30. Solve 3(x + 6) = 4(2x – 8) and check your answer. (4)

256 Mathematics–7
WORKSHEETS
Chapters
1. Integers
2. Fractions and Decimals
3. Data Handling
4. Simple Equations
5. Lines and Angles
6. The Triangle and Its Properties
7. Congruence of Triangles
8. Comparing Quantities
9. Rational Numbers
10. Practical Geometry
11. Perimeter and Area
12. Algebraic Expressions
13. Exponents and Powers
14. Symmetry
15. Visualising Solid Shapes
WORKSHEET-1
Name: Grade:

Class: Section: Roll No.: Maximum Marks.: Teacher’s Sign

[Chapter — Integers]
1. Evaluate: [–48 ÷ 12] ÷ (–2). 2


.....................................................................................................................................................


.....................................................................................................................................................


.....................................................................................................................................................
2. Replace the blank with an integer to make it a true statement. 2
(i) (–9) × = 108

(ii) ÷ 45 = 5


.....................................................................................................................................................


.....................................................................................................................................................


.....................................................................................................................................................


.....................................................................................................................................................
3. Find the product using suitable property: 2
35 × (–92) + (–35) × 8


.....................................................................................................................................................


.....................................................................................................................................................


.....................................................................................................................................................
4. Do as directed: 2
(i) Write a pair of negative integer whose difference is 7.
(ii) What will be the sign of product of 5 negative integers and 7 positive integers.


.....................................................................................................................................................


.....................................................................................................................................................

  259

.....................................................................................................................................................


.....................................................................................................................................................
5. Write three consecutive numbers in the patterns: 4
(i) –1024, 256, –64, _______, _______, _______.
(ii) –7, –21, –63, _______, _______, _______.


.....................................................................................................................................................


.....................................................................................................................................................


.....................................................................................................................................................


.....................................................................................................................................................
6. The sum of two integers is –78. If one of them is –43, find the other. 2


.....................................................................................................................................................


.....................................................................................................................................................


.....................................................................................................................................................
7. The product of two integers is –729. If one of them is 81, find the other. 2


.....................................................................................................................................................


.....................................................................................................................................................


.....................................................................................................................................................
8. In a test containing 25 questions, 5 marks are given for every correct answer and (–3) marks
for every incorrect answer. A student attempts only 15 questions and scored 35 marks. Find the
correct answers and incorrect answers. Verify your solution. 4


.....................................................................................................................................................


.....................................................................................................................................................


.....................................................................................................................................................


.....................................................................................................................................................


.....................................................................................................................................................
qqq

260
WORKSHEET-2
Name: Grade:

Class: Section: Roll No.: Maximum Marks.: Teacher’s Sign

[Chapter — Fractions and Decimals]


1. Write the next two numbers in the pattern: 2
45, 15, 5, ______, ______.

.....................................................................................................................................................


.....................................................................................................................................................


.....................................................................................................................................................


.....................................................................................................................................................
2. Each side of a regular hexagon is 3.9 cm long. Find the perimeter of the hexagon. 2

.....................................................................................................................................................


.....................................................................................................................................................


.....................................................................................................................................................
3. Find the value of: 3
1 1
+
6 2
3 5
7 5


.....................................................................................................................................................


.....................................................................................................................................................


.....................................................................................................................................................


.....................................................................................................................................................


.....................................................................................................................................................
4. Tarun is cutting ribbons in the shape given alongside. How many ribbons can 3
be cut from the roll of 6 m long ribbon. Find the length of remaining ribbon.

............................................................................................................................


............................................................................................................................

  261

.....................................................................................................................................................


.....................................................................................................................................................
1
5. Sohail read th pages of a magazine of 240 pages. If he reads further 30 pages
4
of the book. What fraction of total number of pages would he have read? 2


.....................................................................................................................................................


.....................................................................................................................................................


.....................................................................................................................................................


.....................................................................................................................................................
6. Simplify and write the result in decimal form 3
3 æ1 20 ö÷ 1
¸ çç of ÷÷ + 3 ¸ (reciprocal of 7)
100 ç
è 3 5 ø 400


.....................................................................................................................................................


.....................................................................................................................................................


.....................................................................................................................................................


.....................................................................................................................................................


.....................................................................................................................................................
æ ö
7.
1
of a number equals çç 3 ¸ 25 ÷÷ . What is the number? 2
5 çè 4 7 ÷ø


.....................................................................................................................................................


.....................................................................................................................................................


.....................................................................................................................................................
8. The size of a floor is 7.2 m long and 5.4 m wide. A 60 cm square shaped tile costs `120. What
will be the cost to cover the floor with these tiles? 3

.....................................................................................................................................................


.....................................................................................................................................................


.....................................................................................................................................................

qqq

262
WORKSHEET-3
Name: Grade:

Class: Section: Roll No.: Maximum Marks.: Teacher’s Sign

[Chapter— Data Handling]


1. Find the mean of the first six whole numbers. 2
.....................................................................................................................................................

.....................................................................................................................................................
2. What is the probability of getting a prime number on throwing a dice? 2
.....................................................................................................................................................

.....................................................................................................................................................
3. The ages (in years) of my ten school friends is as follows: 2
12, 10, 11, 12, 12, 10, 15, 14, 12, 10
(i) What is the range of the ages of my friends?
(ii) What is the mean age of my friends?
.....................................................................................................................................................

.....................................................................................................................................................
4. Find mode of the given set of numbers: 2
3, 3, 4, 5, 6, 4, 3, 5, 7, 3
Also if 4 is the mode of the data then write minimum number of times it should be repeated.
.....................................................................................................................................................

.....................................................................................................................................................
5. Observations arranged in ascending order is as follows: 2
20, 20, 24, 35, 46, a, 49, 50, 50
(i) Find median of the data
(ii) Possible values of a.
.....................................................................................................................................................

.....................................................................................................................................................
6. The runs scored in a cricket match by 11 players is as follows: 3
16, 25, 22, 25, 40, 18, 10, 25, 18, 20, 15
Find the (i) mean; (ii) median; and (iii) mode of the data.
.....................................................................................................................................................

.....................................................................................................................................................

.....................................................................................................................................................

  263
7. Look at the double bar graph carefully and answer the following questions: 4

(i) What does the graph depict?


(ii) In which year is the difference between sale of English book and Hindi book the least?
(iii) What fraction is increase of English book from year 1995 to 1996?
(iv) Find the ratio of sale of Hindi books in 1997 to 1998.
.....................................................................................................................................................

.....................................................................................................................................................

.....................................................................................................................................................

.....................................................................................................................................................
8. Consider this data collected from a survey of a colony. 3
Favourite Sport Cricket Basket Ball Swimming Hockey Athletics
Watching 1240 470 510 430 250
Participating 620 320 320 250 105
(i) Draw a double bar graph choosing an appropriate scale. What do you infer from the bar
graph?
(ii) Which sport is most popular?
(iii) Which is more preferred, watching or participating in sports?
.....................................................................................................................................................

.....................................................................................................................................................

.....................................................................................................................................................

.....................................................................................................................................................

.....................................................................................................................................................

qqq
264
WORKSHEET-4
Name: Grade:

Class: Section: Roll No.: Maximum Marks.: Teacher’s Sign

[Chapter — Simple Equations]


1. If 5x = –15, then find x. 2

.....................................................................................................................................................


.....................................................................................................................................................
2. Solve for x: 3(x – 7) = 15. 2

.....................................................................................................................................................


.....................................................................................................................................................


.....................................................................................................................................................
3. Write equations for the following statements: 2
(i) Four-fifth of a number ‘a’ is 15.
(ii) Quotient of a number x and 7 reduced by 5 gives 45.

.....................................................................................................................................................


.....................................................................................................................................................


.....................................................................................................................................................
4. Write statement for the following equations: 2
6m
(i) =4
11
(ii) 3n – 4 = 23

.....................................................................................................................................................


.....................................................................................................................................................
4
5. If m = , say whether the equations 3m – 7 = 3(m + 8) is satisfied. 2
3

.....................................................................................................................................................


.....................................................................................................................................................


.....................................................................................................................................................

  265

.....................................................................................................................................................


.....................................................................................................................................................
6. Solve the following equation for x and check your solution: 2
5 +8
3x + =
7 7


.....................................................................................................................................................


.....................................................................................................................................................


.....................................................................................................................................................


.....................................................................................................................................................


.....................................................................................................................................................
7. In a herd of elephants, half of the elephants were male, one-third of them were female and
remaining were baby elephants. If the number of baby elephants was 36, what was the total
number of elephants in the herd. 4

.....................................................................................................................................................


.....................................................................................................................................................


.....................................................................................................................................................


.....................................................................................................................................................


.....................................................................................................................................................


.....................................................................................................................................................
8. If tens digit of a two-digit number is one more than twice the unit digit. The number formed by
interchanging digits is 45 less than the original number. Find the number. Verify your answer. 4

.....................................................................................................................................................


.....................................................................................................................................................


.....................................................................................................................................................


.....................................................................................................................................................


.....................................................................................................................................................


.....................................................................................................................................................
qqq

266
WORKSHEET-5
Name: Grade:

Class: Section: Roll No.: Maximum Marks.: Teacher’s Sign

[Chapter — Lines and Angles]


1. In figure given below Ð1 and Ð2 are adjacent, state true or false and justify your answer. 2


.....................................................................................................................................................


.....................................................................................................................................................
2. Fill in the blanks: 2
(i) Supplementary angle of 54° = .................................
(ii) Complementary angle of right angle is ...................................... angle.
3. State True/False: 2
(i) If two parallel lines are intersected by a transversal, then alternate interior angles
are equal.
(ii) If two lines are intersected by a transversal and corresponding angles are equal,
then lines are not always parallel.
4. In the following figure l is a line, find a and give reasons. 2


.....................................................................................................................................................


.....................................................................................................................................................


.....................................................................................................................................................
5. In given figure, AB || CD and a and c are complementary angles, then find b. 2


.....................................................................................................................................................


.....................................................................................................................................................

  267

.....................................................................................................................................................


.....................................................................................................................................................
6. In figure, OR ^ OS, QOS = 2POR. Find QOS and POR. 3


.....................................................................................................................................................


.....................................................................................................................................................


.....................................................................................................................................................
7. In the following figure, find measure of a. 3


.....................................................................................................................................................


.....................................................................................................................................................


.....................................................................................................................................................
8. If PQ || RS and 2 = 3x, 1 = 45°. Find all angles. 4

3x 2 1
P 3 4 Q

6 5
R 7 8 S


.....................................................................................................................................................


.....................................................................................................................................................


.....................................................................................................................................................


.....................................................................................................................................................


.....................................................................................................................................................
qqq
268
WORKSHEET-6
Name: Grade:

Class: Section: Roll No.: Maximum Marks.: Teacher’s Sign

[Chapter — The Triangle and Its Properties]


1. What is the sum of the acute angles of a right triangle? 2

.....................................................................................................................................................
2. Fill in the blanks: 2
(a) In a triangle exterior angle is always equal to the sum of .........................................
(b) In an isosceles triangle, if vertical angle is 60° then it is ................................ triangle also.
3. DABC is an isosceles triangle with AB = AC. Find a. 2

................................................................................................


................................................................................................


................................................................................................


................................................................................................
4. DABC is an equilateral triangle. Comment upon the type of DADE.
Justify your answer. Also, write relation between DE and BC. 2


...................................................................................................................


...................................................................................................................


...................................................................................................................


.....................................................................................................................................................
5. In DABC prove that Ð1 + Ð2 + Ð3 = 360°. 2

..............................................................................................


..............................................................................................


..............................................................................................


..............................................................................................


..............................................................................................


..............................................................................................

  269
6. In given figure of DABC, find x, y, z and a. 4

..............................................................................................


..............................................................................................


..............................................................................................


.....................................................................................................................................................


.....................................................................................................................................................


.....................................................................................................................................................


.....................................................................................................................................................
7. A tree of height 25 m, due to strong wind, breaks at the point A which is 12 m above the ground.
At what distance from its base will the tip of the tree touch the ground? 3

.....................................................................................................................................................


.....................................................................................................................................................


.....................................................................................................................................................


.....................................................................................................................................................


.....................................................................................................................................................


.....................................................................................................................................................
8. In DABC altitudes AD and BE are intersecting at M. If ÐCBE = 3
40°. Find ÐAME and ÐCAD.

..............................................................................................


..............................................................................................


..............................................................................................


.....................................................................................................................................................


.....................................................................................................................................................


.....................................................................................................................................................


.....................................................................................................................................................

qqq

270
WORKSHEET-7
Name: Grade:

Class: Section: Roll No.: Maximum Marks.: Teacher’s Sign

[Chapter — Congruence of Triangles]


1. Mention four congruence criterion to determine triangles are congruent. 1

.....................................................................................................................................................


.....................................................................................................................................................
2. In the given figure, is DRAM @ DPAN? Justify your answer. 2


................................................................................................


................................................................................................


................................................................................................


................................................................................................


.....................................................................................................................................................
3. In the given figure, DPQR and DSTU are right angled at P and S respectively. Also QR = UT and
US || PR. Is DPQR @ DSTU? If yes, write three matching parts. 3


............................................................................................................


............................................................................................................


............................................................................................................


............................................................................................................


.....................................................................................................................................................


.....................................................................................................................................................
4. If DADC @ DCBA then state whether ABCD is a parallelogram. Justify your answer. 2


.........................................................................................................


.........................................................................................................


.........................................................................................................


.........................................................................................................

  271
5. In the given figure, AC and BD bisect each other and ÐA = ÐC = 90°. Using R.H.S. show
DBAO @ DDCO. Can any other congruence criterion be used? 3


.....................................................................................................


.....................................................................................................


.....................................................................................................


.....................................................................................................


.....................................................................................................
6. In DRAG; RP is angle bisector and AR ^ RG. 3
(i) State three pairs of equal parts in the triangles ARP and GRP.
(ii) Is DARP @ DGRP? (iii) Does RP bisects AG?

...........................................................................................................


...........................................................................................................


.....................................................................................................................................................


.....................................................................................................................................................
7. Observe the figure and state three matching equal parts. 3
(i) Is DPQR @ DSRQ? (ii) Is PR = SQ?
(iii) Is ÐP = ÐS?

...........................................................................................


...........................................................................................


.....................................................................................................................................................


.....................................................................................................................................................
8. In DABC, altitudes BD and CE are equal. 3
(i) Is DBEC @ DCDB?
(ii) Is DABD @ DACE?

.....................................................................................................


.....................................................................................................


.....................................................................................................................................................


.....................................................................................................................................................


.....................................................................................................................................................

272
WORKSHEET-8
Name: Grade:

Class: Section: Roll No.: Maximum Marks.: Teacher’s Sign

[Chapter — Comparing Quantities]


1. If S.P. of an article is ` 400 and C.P. is ` 250. Find loss/profit. 2

.....................................................................................................................................................


.....................................................................................................................................................


.....................................................................................................................................................


.....................................................................................................................................................
2. Do as directed: 2
9
(i) Express as % ...........
20
7
(ii) Express % as ratio ...........
13

.....................................................................................................................................................


.....................................................................................................................................................


.....................................................................................................................................................


.....................................................................................................................................................
3. If P = ` 2000, R = 4% p.a., T = 2 yrs, then find I. 2

.....................................................................................................................................................


.....................................................................................................................................................


.....................................................................................................................................................


.....................................................................................................................................................
4. Find x; if 300 : 750 : : x : 1000. 2

.....................................................................................................................................................


.....................................................................................................................................................


.....................................................................................................................................................

  273
5. Find the cost of 12 articles if 9 articles cost ` 135. 2

.....................................................................................................................................................


.....................................................................................................................................................


.....................................................................................................................................................
6. Price of petrol increased from ` 66 per litre to ` 68 per litre. Find increase %. 2

.....................................................................................................................................................


.....................................................................................................................................................


.....................................................................................................................................................


.....................................................................................................................................................
7. Sonia bought a TV for ` 36,000 and later sold it to Amrita at a profit of 5%. Amrita used it for
a period of two years and later sold it to Puneet at a loss of 20%. For how much did Puneet buy
it? 4

.....................................................................................................................................................


.....................................................................................................................................................


.....................................................................................................................................................


.....................................................................................................................................................


.....................................................................................................................................................
8. Sarita lent out some money at the simple interest of 16% p.a. to Monica and received ` 9600
as interest for 2 years. She lent out the amount received at 15% p.a. to Kavita and received
` 17,820 simple interest. Find for how long did Kavita borrowed money. Also find what was
original amount with Sarita? 4

.....................................................................................................................................................


.....................................................................................................................................................


.....................................................................................................................................................


.....................................................................................................................................................


.....................................................................................................................................................


.....................................................................................................................................................


.....................................................................................................................................................
qqq

274
WORKSHEET-9
Name: Grade:

Class: Section: Roll No.: Maximum Marks.: Teacher’s Sign

[Chapter — Rational Numbers]


1. ................ and ................ are reciprocal of their own. 2

.....................................................................................................................................................
-52
2. Express as 2
48
(i) with numerator –39.
(ii) with denominator –108.

.....................................................................................................................................................


.....................................................................................................................................................


.....................................................................................................................................................


.....................................................................................................................................................
3. Simplify: 3
12 çæ -7 ÷ö 12 æç -6 ÷ö æç12 ÷ö 33
´ç ÷ + ´ç ÷ + ç ÷´
11 çè 5 ø÷ 11 èç 5 ÷ø èç11 ÷ø 5


.....................................................................................................................................................


.....................................................................................................................................................


.....................................................................................................................................................


.....................................................................................................................................................
4. Find: 2
7 2
(i) the sum of and .
13 11
27 -30
(ii) the difference of and .
4 7

.....................................................................................................................................................


.....................................................................................................................................................


.....................................................................................................................................................

  275
-7 5 2
5. Arrange the rational numbers , , in ascending order. 3
10 8 -3

.....................................................................................................................................................


.....................................................................................................................................................


.....................................................................................................................................................


.....................................................................................................................................................
4 5
6. Find (i) the product of - and -  2
5 12
4 çæ 28 ÷ö
(ii) ¸ç ÷
7 çè 44 ÷ø

.....................................................................................................................................................


.....................................................................................................................................................


.....................................................................................................................................................


.....................................................................................................................................................
13
7. Represent - on number line. 2
5

.....................................................................................................................................................


.....................................................................................................................................................


.....................................................................................................................................................


.....................................................................................................................................................
8. Find the multiplicative inverse and additive inverse of  4
3 -4 3 5
¸ ´ +
13 65 7 7


.....................................................................................................................................................


.....................................................................................................................................................


.....................................................................................................................................................


.....................................................................................................................................................


.....................................................................................................................................................
qqq

276
WORKSHEET-10
Name: Grade:

Class: Section: Roll No.: Maximum Marks.: Teacher’s Sign

[Chapter — Practical Geometry]


1. Can a triangle be constructed with sides 2.5 cm, 3.0 cm, 6.5 cm? Give reason. 2

.....................................................................................................................................................


.....................................................................................................................................................


.....................................................................................................................................................


.....................................................................................................................................................
2. State True/False: 2
(i) We can draw exactly one triangle with each angle of measure 60°.
(ii) We can draw a right triangle if length of hypotenuse and one of its acute angle is
known.

3. Draw
 a line AB. Take a point M on it. Draw MX such that ÐXMB
 = 60°. Take a point N on
MX such that MN = 5 cm. Draw a line perpendicular to MX and produce it to intersect AB
(or AB produced) at point Q. 3

.....................................................................................................................................................


.....................................................................................................................................................


.....................................................................................................................................................


.....................................................................................................................................................


.....................................................................................................................................................
4. Construct a DPQR with ÐP = 60°, ÐR = 45° and PQ = 5 cm using only scale, compasses and
pencil. 3

.....................................................................................................................................................


.....................................................................................................................................................


.....................................................................................................................................................


.....................................................................................................................................................


.....................................................................................................................................................

  277
5. Construct a right-angled isosceles triangle with one side (other than hypotenuse) of length
4  cm. 3

.....................................................................................................................................................


.....................................................................................................................................................


.....................................................................................................................................................


.....................................................................................................................................................


.....................................................................................................................................................


.....................................................................................................................................................


.....................................................................................................................................................
6. Draw an equilateral triangle of side length 6 cm using scale and compasses. 3

.....................................................................................................................................................


.....................................................................................................................................................


.....................................................................................................................................................


.....................................................................................................................................................


.....................................................................................................................................................


.....................................................................................................................................................


.....................................................................................................................................................
7. Draw a line l. Take a point P at a distance of 4 cm from line l. Through point P draw a line m
parallel to line l. Write steps of construction. 4

.....................................................................................................................................................


.....................................................................................................................................................


.....................................................................................................................................................


.....................................................................................................................................................


.....................................................................................................................................................


.....................................................................................................................................................


.....................................................................................................................................................
qqq

278
WORKSHEET-11
Name: Grade:

Class: Section: Roll No.: Maximum Marks.: Teacher’s Sign

[Chapter — Perimeter and Area]


1. The distance around a circular region is known as its ..................... . 1

.....................................................................................................................................................


.....................................................................................................................................................
2. Find area of a rectangle having length 14.5 cm and breadth 7.5 cm. 2

.....................................................................................................................................................


.....................................................................................................................................................


.....................................................................................................................................................
3. Find radius of protractor with perimeter 108 cm. 2

.....................................................................................................................................................


.....................................................................................................................................................


.....................................................................................................................................................


.....................................................................................................................................................
4. Area of an isosceles right triangle is 18 sq cm. Find the length of smaller side of the triangle. 2

.....................................................................................................................................................


.....................................................................................................................................................


.....................................................................................................................................................


.....................................................................................................................................................


.....................................................................................................................................................
5. If area of a quadrant is 154 cm2. Find its perimeter. 2

.....................................................................................................................................................


.....................................................................................................................................................


.....................................................................................................................................................


.....................................................................................................................................................

  279
6. If side length of a square is doubled, find ratio of area of original square to new square. Also
find increase %. 3

.....................................................................................................................................................


.....................................................................................................................................................


.....................................................................................................................................................


.....................................................................................................................................................


.....................................................................................................................................................
7. DPQR is right angled at P. PM ^ QR. If RP = 6 cm and PQ = 8 cm. Find PM. 4

.....................................................................................................................................................


.....................................................................................................................................................


.....................................................................................................................................................


.....................................................................................................................................................
8. Two cross roads, each of width 5 m, run at right angles through the centre of a rectangular park
of length 70 m and breadth 45 m and parallel to its sides. Find the area of the roads. Also find
the cost of constructing the roads at the rate of ` 105 per m2. 4

.................................................................................................................


.................................................................................................................


.................................................................................................................


.................................................................................................................


.....................................................................................................................................................


.....................................................................................................................................................


.....................................................................................................................................................


.....................................................................................................................................................
qqq

280
WORKSHEET-12
Name: Grade:

Class: Section: Roll No.: Maximum Marks.: Teacher’s Sign

[Chapter — Algebraic Expressions]


1. Simplify the expression by combining the like terms: 2
6p3 – 6p2q + pq2 + p2q – q3

.....................................................................................................................................................


.....................................................................................................................................................


.....................................................................................................................................................
2. Find the value of p2 + q2 – 2pq at p = 1 and q = –2. 2

.....................................................................................................................................................


.....................................................................................................................................................


.....................................................................................................................................................


.....................................................................................................................................................
3. The length of a side of a square is 3x + 5. Write an expressions for its perimeter. 2

.....................................................................................................................................................


.....................................................................................................................................................


.....................................................................................................................................................


.....................................................................................................................................................
4. Manavi works in a shop and gets ` 80 per hour. Last week she worked for 6 hours a day for 6
days and this week she worked for y hours. Write an algebraic expression for the money paid
to him for both the weeks. 3

.....................................................................................................................................................


.....................................................................................................................................................


.....................................................................................................................................................


.....................................................................................................................................................


.....................................................................................................................................................

  281
5. Subtract the sum of –2a3b2 + 3a2b3 and –7a3b2 – 4a2b3 + b4 from –6a3b2 – b4. 3

.....................................................................................................................................................


.....................................................................................................................................................


.....................................................................................................................................................


.....................................................................................................................................................


.....................................................................................................................................................
6. In a rectangular plot of length 8x m and breadth 4x m, 6 square flower beds of side (2x + 3) m
each has been laid as shown in the figure. Find the total cost of laying grass in the area outside
square at the rate of ` 2 per sq m. 4


.....................................................................................................................................................


.....................................................................................................................................................


.....................................................................................................................................................


.....................................................................................................................................................


.....................................................................................................................................................
7. A hostel charges ` x per day per person for food and levies a fixed room charges of ` 5000.
Write an expression if Shalu stays in hostel for January to May. Also find total charges if she
stayed only for 120 days. 4

.....................................................................................................................................................


.....................................................................................................................................................


.....................................................................................................................................................


.....................................................................................................................................................


.....................................................................................................................................................


.....................................................................................................................................................
qqq

282
WORKSHEET-13
Name: Grade:

Class: Section: Roll No.: Maximum Marks.: Teacher’s Sign

[Chapter — Exponents and Powers]


1. Express 243 in exponential form. 2

.....................................................................................................................................................


.....................................................................................................................................................


.....................................................................................................................................................
2. Simplify: 23 × 34 ÷ 62. 2

.....................................................................................................................................................


.....................................................................................................................................................


.....................................................................................................................................................
3. Do as directed: 2
(i) Express 0.0625 × 105 in usual form.
(ii) Express 1678000 in standard form (scientific notation).

.....................................................................................................................................................


.....................................................................................................................................................
4. Simplify and express in scientific notation 2
6.25 × 103 × 15.4 × 10–8.

.....................................................................................................................................................


.....................................................................................................................................................
5. Simplify and express each of the following in exponential form (with positive exponent). 2
éæ 4 ö5 æ 4 ö4 ù æ 4 ö12
(i) êêçç ÷÷÷ ´çç ÷÷÷ úú ¸ çç ÷÷÷
èç 7 ø çè 7 ø ú çè 7 ø
ëê û
3
éæ 3 ö ù æ 5 ö7 æ 3 ö4
2
(ii) êêçç ÷÷ úú ¸ çç ÷÷ ´çç ÷÷
ç ÷ ç ÷ ç ÷
êëè 5 ø úû è 3 ø è 5 ø
.....................................................................................................................................................


.....................................................................................................................................................

  283

.....................................................................................................................................................


.....................................................................................................................................................


.....................................................................................................................................................


.....................................................................................................................................................
6. Find the value of n, where n is an integer such that 3
3n – 4 × 62n – 5 = 216

.....................................................................................................................................................


.....................................................................................................................................................


.....................................................................................................................................................


.....................................................................................................................................................


.....................................................................................................................................................
7. Find m so that 3
8 6
æ 5 ö÷ æ 2 ö÷
çç ÷ ´çç ÷ = (6.25)2 m-5
èç 2 ø÷ èç 5 ø÷


.....................................................................................................................................................


.....................................................................................................................................................


.....................................................................................................................................................


.....................................................................................................................................................


.....................................................................................................................................................


.....................................................................................................................................................
8. If 319 – 2.318 – 317 + 4.316 = 30 × 3k, find the value of k. 4

.....................................................................................................................................................


.....................................................................................................................................................


.....................................................................................................................................................


.....................................................................................................................................................


.....................................................................................................................................................
qqq

284
WORKSHEET-14
Name: Grade:

Class: Section: Roll No.: Maximum Marks.: Teacher’s Sign

[Chapter — Symmetry]
1. Draw a regular hexagon. How many lines of symmetry does this have? Draw all. 4

..................................................................................................................................................


..................................................................................................................................................


..................................................................................................................................................

..................................................................................................................................................
2. Draw the lines of symmetry of 2

(a) (b)

3. Write the number of letters in the word TRAIL that have line symmetry. Also draw the line of
symmetry for these letters. 2


.....................................................................................................................................................


.....................................................................................................................................................


.....................................................................................................................................................
4. Which of the following road signs has a line symmetry? 2

(a) (b)

(c) (d)


.....................................................................................................................................................

  285
5. What do you call a rectangle that has four lines of symmetry? 1

.....................................................................................................................................................
6. Find out which of the following figures have rotational symmetry about the marked point and
also find the order of rotational symmetry. 2

(a) (b)


.....................................................................................................................................................


.....................................................................................................................................................


.....................................................................................................................................................
7. Does a kite have a rotational symmetry? 1

.....................................................................................................................................................
8. Given below is a part of a shape with centre of rotation O. The order of rotational symmetry
is  6. Complete the figure. 2


.....................................................................................................................................................


.....................................................................................................................................................


.....................................................................................................................................................
9. Copy the word Maths onto a grid, as shown. Reflect it about l. 4


.....................................................................................................................................................


.....................................................................................................................................................


.....................................................................................................................................................


.....................................................................................................................................................
qqq

286
WORKSHEET-15
Name: Grade:

Class: Section: Roll No.: Maximum Marks.: Teacher’s Sign

[Chapter — Visualising Solid Shapes]

1. For the solid, count the number of faces, vertices and edges. Check the Euler’s rule
for it. 4


.....................................................................................................................................................


.....................................................................................................................................................


.....................................................................................................................................................

2. Look at the following nets and add the face that is needed to make the net of the named solid.
Shade the added face. 4

(a) (b)

rectangular prism

(c) (d)

square prism


.....................................................................................................................................................


.....................................................................................................................................................


.....................................................................................................................................................
3. Label each net given below. 4

(a) (b)

  287
(c) (d)


.....................................................................................................................................................


.....................................................................................................................................................
4. Sketch the rectangular pyramid as shown as if seen directly from: 3
(a) above (plane)
(b) the front (elevation)
(c) side

.......................................................................................................


.......................................................................................................


.....................................................................................................................................................


.....................................................................................................................................................
5. The solid at the right shows a solid, made of cubes, drawn on a dotted (isometric surface) paper.
Draw its plane, front and side (from the left) elevation. Also write the number of cubes used.4

...............................................................................................................


...............................................................................................................


...............................................................................................................


...............................................................................................................


...............................................................................................................


.....................................................................................................................................................
6. State Euler’s Formula. 1

.....................................................................................................................................................


.....................................................................................................................................................


.....................................................................................................................................................


.....................................................................................................................................................


.....................................................................................................................................................
qqq
288
Answers to Test Papers
Term I
Periodic Test-1 31
1. 6 2. 5.38 × 106 3. –78300 11. 6 x 2 + x + 1 12. 75°
6
4 13. (i) –62500, (ii) –6700
4. 18 5. 0 6.
9 1
14. (i) 25 marks, (ii) 11 marks 15. 2 m
4
17 7 8
7. 8. 16. ` 89.10 17. 18.
30 27 15
19. 2x3 + 12x2 + 6x – 10 20. 7, 9, 11
9. x = 10 10. 13.75 21. (i) (ii) 2
Periodic Test-2
1. 5(y + 3) – 2x3 2. No
3. a = –5 4. 8x = 9 + 7 22.

23. At 11:00 pm; –12° C 24. ` 9000


5.
1
25. (i) − , (ii) –49
49
26. 2.7 × 108­ sec
27. (i) 90, (ii) x –yz + yb – yc
6. , four
28. (i) 2, (ii) 3 29. 8
2149 2149
30. 2.149; (i) 2.149 = , (ii)
1000 1000
7.
Half-Yearly Test Paper-2
40 5
23 1. 14 2. 3. −
8. 9 4
7
4. –128 5. Yes, 8 6. 9
9. (a) Both reflection and rotational symmetry. 7. 9 8. –108500 9. 9
(b) Both reflection and rotational symmetry. 4
(c) Only rotational symmetry of order 1.  9
10.  
10. (i) Cube (ii) Cone  10 
(iii) Square pyramid 11. Degree of p2q2 = 4, Degree of 2pq2 = 3,
(iv) Triangular prism. Degree of –p­2q = 3, Degree of 1 = 0, Degree
Half-Yearly Test Paper-1 of expression = 4.
1. True 2. 20 3. 214 × 32 12. 20 13. Distributive
14. (i) False, (ii) True, (iii) True
4. Three 5. Six 6. 9.2
1
7. –54 15. ` 105.65 16. 58 m
8. 10.65, 12.01, 12.14, 13.10, 30.1, 81.201 6 3 5 2
17. > > 18. 50
1 1 7 14 −4
9. − 10. −
2 8 19. 2x3 – 4x2 – 2x – 2

Answers  289
20. x = 1 1 19
23. –4 24. 25. −
6 25
21. (i) 3, (ii) 3, (iii)
1
26. –1.62 27. −
675
22. No. of dots on A, B and C are 2, 3, 1 28. 4x2 + 6x – 10 29. x = 10
respectively. 30. 36 and 48
Term II
Periodic Test-1 11. 28 m 12. 2600 13. ` 105
1. 9 2. 600 m 3. 15 14. ` 15266.67 15. 3 16. x = 7
1 17. 32 18. SI = ` 1920, A = ` 7920
4. 5. ` 8928 6. Sarika 19. ` 22500 20. x = 125°, y = 55°, z = 55°
8
21. 18 m 23. ` 423.23 24. ` 44
7. ` 505 9. 32
25. (i) 85°, (ii) 130°, (iii) 50°, (iv) 95°
10. (i) No, (ii) Yes
27. (a) 15, (b) 23 28. 56 cm2
Periodic Test-2 29. –7255095 30. x = 3
πd 2 Annual Test Paper-2
1. 2. Eight 1. 3 : 5 2. 5.1% 3. 120°
4
3. y = 120°, z = 60° 4. 60°, 60°, 60° 5. –1 7. 12 tables
8. 50 marks 9. x = 60°, y = 120°
4. x = 65°, y = 50°
10. x = 70°, y = 45° 11. 25 cm
5. 420 m 6. 24 cm 7. 70°
12. 7 m 13. 1050 cm2
9. ∠ACD = 125°, ∠AED = 145°
14. (i) 20, (ii) 2, (iii) 12.56
10. (i) 248 m2, (ii) 2760 m2
15. Associative Property
Annual Test Paper-1 16. x = 5 17. 40 18. ` 414
1. 2 : 5 2. 0.156 3. 45° 19. Sarika’s performance is better
4. 60° 5. –126 6. x = 8 20. x = 40°, y = 30°, parallelogram
7. 4.5 kg 8. 252 21. 39 m 22. 467.50 km 23. 9 hours
9. x = 60°, y = 50° 24. 56.25% 28. 6 cm2 29. –761000
10. x = 115°, y = 35° 30. x = 10

Answers to Worksheets
Worksheet-1 3 1
1. 2 2. (i) –12, (ii) 225 5. 6. 21.04 7. 1
8 20
3. –3500 4. (i) –4, –11, (ii) Negative 8. ` 12960
5. (i) 16, –4, 1, (ii) –189, –567, –1701
6. –35 7. –9 Worksheet-3
8. Correct = 10 and incorrect = 5 5 1
1. 2.
2 2
Worksheet-2 3. (i) 5, (ii) 11.8 4. Mode = 3, 5 times
5 5 4
1. , 2. 23.4 cm 3. 5. (i) 46, (ii) From 46 to 49
3 9 9 6. (i) 21.27, (ii) 20, (iii) 25
4. 4 ribbons, remaining ribbon = 80 cm 7. (i) Sale of English and Hindi books

290 Mathematics–7
1 6 7. ∠PQR = ∠SRQ, ∠PRQ = ∠SQR, QR = QR
(ii) 1998 (iii) (iv)
7 7 (i) Yes (ii) Yes (iii) Yes
8. (ii) Cricket, (iii) Watching 8. (i) Yes (ii) Yes
Worksheet-8
Worksheet-4 1. Profit = ` 150
1. x = –3 2. x = 12 2. (i) = 45%, (ii) 7 : 1300
4 3. I = ` 160 4. x = 400 5. ` 180
3. (i) a = 15 , (ii) (x ÷ 7) – 5 = 45
5 1
4. (i) Quotient of six times m and 11 is four. 6. 3 % 7. ` 30,240
33
(ii) Four less than three times ‘n’ is twenty 8. 3 years; ` 30,000
three. Worksheet-9
1 −39 117
5. No 6. x = 7. 216 1. 1 and –1 2. (i), (ii)
7 36 −108
8. 94
48 103 309
Worksheet-5 3. 4. (i) , (ii)
11 143 28
1. False 2. (i) 126°, (ii) zero −7 2 5 1
5. < < 6. (i) , (ii) 44
3. (i) True, (ii) False 10 −3 8 3 49
4. a = 65° 5. 90° 7.
6. ∠POR = 30° and ∠QOS = 60°
7. a = 35° −28
8. Multiplicative Inverse =
8. ∠5 = ∠1 = 45°; ∠8 = ∠4 = 135°; 25
∠6 = ∠2 = 135°; ∠7 = ∠3 = 45° 25
Additive Inverse =
28
Worksheet-6
1. 90° Worksheet-10
2. (a) Two opposite interior angles 1. No, as 2.5 + 3.0 = 5.6 < 6.5
(b) An equilateral In a triangle, sum of any two sides is always
3. a = 100° greater than the third side.
4. DADE is an equilateral triangle; DE || BC. 2. (i) False, (ii) True
6. x = 50°, y = 130°, a = 20°, z = 30° Worksheet-11
7. 5 m 8. ∠CAD = 40°, ∠AME = 50° 1. Circumference
2. 108.75 sq cm 3. r = 21 cm
Worksheet-7 4. 6 cm 5. 50 cm 6. 1: 4; 300%
1. SSS, SAS, ASA, RHS 2. No 7. PM = 4.8 cm 8. 550 sq m, ` 57750
3. Yes; DPQR ≅ DSTU by AAS; Worksheet-12
∠PRO = ∠SUR, ∠P = ∠S, QR = UT 1. 6p3 – 5p2q + pq2 – q3
4. ABCD is a parallelogram 2. 9 3. 4(3x + 5)
5. Yes, ASA, SAS can be used. 4. 80 × (36 + y)
6. (i) ∠ARP = ∠GRP, RP = RP, ∠RPA = ∠RPG 5. 3a3b2 – 2b4 + a2b3
(ii) Yes (iii) Yes 6. ` (16x2 – 144x –108)

Answers  291
7. Expression = 5000 + 151x 6. (a) has rotational symmetry of order 2.
Total charges for 120 days = ` (5000 + 120x) (b) has rotational symmetry of order 4.
Worksheet-13
1. 35 2. 18 7. No 8.
3. (i) 6250, (ii) 1.678000 × 106
4. 9.625 × 10–4
3 17
 7  3 9.
5. (i)   , (ii)  
 4  5
6. n = 4 7. m = 3 8. k = 15
Worksheet-15
Worksheet-14
1. Faces 7; Vertices 10 and edges 15

1. 2. (a) (b)

(c) (d)
2. (a) (b)
3. (a) cone, (b) cylinder, (c) tetrahedron,
(d) pyramid
3. 4.
4. (a) (b) (c)

5. Plane Front Side

5.

Number of cubes = 9
6. Euler’s formula: F + V – E = 2
qqq

292 Mathematics–7

You might also like